You are on page 1of 114

JEE TRAINER

Preparing You For Both JEE MAIN and ADVANCED

COORDINATE GEOMETRY
By Nitin Jain

Ativeer Publication
A unit of Ativeer Research and Educational Services Pvt Ltd
Our Other Products (for JEE MAIN and ADVANCED)

Mechanics by Vikas Jain


Fluid and Waves by Vikas Jain
Electrodynamics by Vikas Jain
Optics and Modern Physics by Vikas Jain
Trigonometry by V Ramakrishna
Vectors and 3D by Nitin Jain
Permutations and Combinations by V Ramakrishna
Differential Calculus by V Ramakrishna
Integral Calculus by V Ramakrishna
Algebra by Nitin Jain
Mock Tests for JEE MAIN by Vikas Jain, Rajshree and Nitin Jain

Published By :
ATIVEER PUBLICATION
(A Unit of Ativeer Research and Educational Services Pvt. Ltd.)
Regd. Office : H.No. 309,
Dashmesh Nagar, Baghpat Road,
Meerut, UP

Head Office : H. No. 2-10-644,


Teacher's Colony, Phase - I
Warangal, Telangana.

All rights reserved

c Authors

No part of this publication (either in part or full) may be re-produced, stored in a retrieval system or distributed in any
form or by any means, electronic, mechanical, photocopying, recording, scanning or otherwise without the prior written
permission of the publisher and the authors. Ativeer Publication and its authors has collected the information contained
in this book from the sources believed to be reliable and true. However Ativeer and its associates don't take any
responsibility for the absolute accuracy of any information published and the damages or loss suffered thereupon.

Typesetting :
Ativeer DTP Unit at Warangal

Printed at :
Deepak Traders, Meerut
Preface

Dear Students
Ativeer feels privileged to serve you with its best seller JEE TRAINER SERIES for JEE Main and
Advanced. These books have been designed by the expert visionaries of the field and aim at providing the
best assistance to the students.
JEE TRAINER is designed with the vision of providing knowledge and guidance for the JEE preparation
along with excellent performance at the board level.
The book fulfills all tutorial needs of the students and is in itself a complete package which will help the
students to achieve their goal.
The book has been developed in such a way that while going through it, you will feel that teacher is present
in front of you, and sequencing of the topics has been done after intensive research.
We hope you will enjoy the JEE TRAINER SERIES, and this will lead you towards your aim.
Lastly, we have tried our level best to provide you the perfect material but still some errors might have
crept in. If you bring it to our notice, it will be highly appreciated.

Structure of JEE TRAINER


1. The entire Chapter is divided into various topics and sub-topics in a very sequential manner.
2. Topic wise, well explained theory along with Concept Reckoners (practical situations used to explain a
particular concept) have been incorporated.
3. Solved Examples at the end of every topic will help you to boost your knowledge and expertise in
solving questions.
4. Make Concepts Clear (MC2) - This section is developed to give you more practice and gain the
expertise on every topic. Here you will have a number of questions on the topic just covered, whose
solutions are given in last.
5. For more rigorous practice - (PS-1 for JEE MAIN and PS-2 for JEE ADVANCED) are given sepa-
rately including all possible types of questions which are fully solved.

Nitin Jain
Contents
Chapter 1 Straight Line 1.1 – 1.164
1.1 Co-ordinate System 1.1
1.2 Locus and Transformation of Axes 1.33
1.3 Basics of Straight Line 1.43
1.4 Advanced Concepts of Straight Line 1.63
1.5 Family of Straight Lines 1.81
1.6 Pair of Straight Lines 1.94
Practice Session - 1 for JEE MAIN 1.106
Practice Session - 2 for JEE ADVANCED 1.111
BRAIN BOOSTER 1.121
Answer Key 1.124
Explanations to Mc2 1.125
Explanations to PS - 1 1.143
Explanations to PS - 2 1.150

Chapter 2 Circles 2.1 – 2.168


2.1 Equation of Circle in Various Forms 2.1
2.2 Position of point, Equation of chord, Tangent,
Normal to the Circle 2.14
2.3 Power, Chord of Contact, Pair of Tangents 2.27
2.4 Diameter, Inverse Points, Pole and Polar Diameter of a circle 2.40
2.5 Common Tangents, Common Chord, Family of Circles 2.50
2.6 Angle of Intersection between Two Circles, Radical axis,
Co-axial Family, Limiting Points 2.65
2.7 Sphere 2.81
Practice Session - 1 for JEE MAIN 2.87
Practice Session - 2 for JEE ADVANCED 2.94
BRAIN BOOSTER 2.106
Answer Key 2.110
Explanations to Mc2 2.111
Explanations to PS - 1 2.134
Explanations to PS - 2 2.147

Chapter 3 Parabola 3.1 – 3.146


3.1 Basic of Conic Section 3.1
3.2 Standard and General Equation of Parabola 3.8
3.3 Position of point, Chord and Tangent of Parabola 3.21
3.4 Normal of Parabola, Co-normal points, Envelope of Family
of Curve 3.36
3.5 Chord of Contact, Pair of Tangents, Diameter, Pole and Polar 3.52
3.6 Properties of Parabola 3.68
Practice Session - 1 for JEE MAIN 3.87
Practice Session - 2 for JEE ADVANCED 3.92
BRAIN BOOSTER 3.99
Answer Key 3.103
2
Explanations to Mc 3.104
Explanations to PS - 1 3.119
Explanations to PS - 2 3.128

Chapter 4 Ellipse 4.1 – 4.120


4.1 Definition and Equation of Ellipse 4.1
4.2 Position of point, Chord, Tangent of Ellipse 4.18
4.3 Normal, Co-normal Points, Chord of Contact, Pair of Tangents 4.28
4.4 Concylic Points, Pole and Polar, Diameter 4.42
4.5 Properties of Ellipse 4.56
Practice Session - 1 for JEE MAIN 4.75
Practice Session - 2 for JEE ADVANCED 4.79
Answer Key 4.86
2
Explanations to Mc 4.87
Explanations to PS - 1 4.99
Explanations to PS - 2 4.105

Chapter 5 Hyperbola 5.1 – 5.112


5.1 Equation of Hyperbolas in Various Forms 5.1
5.2 Position of Point, Chord, Tangent 5.15
5.3 Co-normal Points, Chord of Contact, Pair of Tangents 5.27
5.4 Pole and Polar, Diameter, Asymptotes 5.35
5.5 Rectangular Hyperbola and Properties of Hyperbola 5.50
5.6 Polar Co-ordinates 5.68
Practice Session - 1 for JEE MAIN 5.76
Practice Session - 2 for JEE ADVANCED 5.79
Answer Key 5.86
Explanations to Mc2 5.87
Explanations to PS - 1 5.94
Explanations to PS - 2 5.100
Coordinate Geometry
Chapter 3
Parabola
3.1 Basic of Conic Section
3.2 Standard and General Equation of Parabola
3.3 Position of point, Chord and Tangent of Parabola
3.4 Normal of Parabola, Co-normal points, Envelope of Family of Curve
3.5 Chord of Contact, Pair of Tangents, Diameter, Pole and Polar
3.6 Properties of Parabola
Parabola
3.1 Basic of conic section As the name suggests, the conic section is a section cut off
from a right circular cone by a plane in various positions.
If we take a pen and randomly draw a curve on paper, The shape of section depends upon the orientation of cutting.
obviously we get some curve (locus), but is this curve
Right circular cone
meaningful ? can this curve represents some mathematical
equation ? does the curve follow any particular pattern ? A right circular cone is locus of a line passing through a
Immediately we are unable to answer these problems or fixed point A and making a fixed constant angle  with
even we can say that only by the help of figure we are fixed line passing through A ( let AB).
unable to say every property of curve.
Here fixed point A is called vertex, fixed line AB is called
Now if we start analysing the statement ‘A point moves in axis and fixed angle  is called semi vertical angle of cone.
such a manner such that the ratio of its distance from a Also the moving line is known as generator of cone.
fixed point to the perpendicular distance from fixed straight
line is constant’. immediately we can answer the above A
A
questions. So, we can say that curve is meaningful only if Vertex
we draw the curve in some mathematical pattern. 
or


e rat
axis

Moreover, if we take a right circular cone and cut it by


Gen

plane, we get some cross-section, which gives some particular


figure which depends upon the orientiation of cutting of B
B
Circular Base
cone by a plane.
The famous greek mathematician Euclid, the creator of Now, right circular cones are divided in two categories.
creative geometry studied various plane sections of a right
1) Single Nappe right circular cone
circular cone and discovered some remarkable curves.
A
After him many mathematical and scientific discoveries
occured, related to section of a cone.  

The sections which are cut from the cone have their own
beauty and they are used a lot in general life as well as in
B
research work.
Let AB is a fixed line and a moving line l intersects fixed
Generally they are used to make different type of mirrors
which are used in ‘ fun show’, to make reflectors, they are line AB at A and also making constant angle  with line
used in researches of outer space and also by the help of AB. If the line l is rotated in such a way that angle  always
these we can even understand the behaviour of atomic remains constant, then the surface generated by line l is
particles. said to be single nappe cone.
Coordinate Geometry

Here A is vertex, AB is axis,  is semivertical angle and line of the cone, and so in this case we have pairs of real and
l is said to be generator of cone. co-incident straight line, and the line co-incide, with generator
of cone.
2) Double Nappe right circular cone
iii)  <  < 900: Under this case the plane does not
A intersect the cone at any other point except vertex. So, under
this case conic section is just a point which co-incides with
 
l
vertex of cone.
V
2) Plane not passing through vertex of cone
 
If plane does not passe through vertex of the cone, then on
the basis of value of  the following conic sections are
B
possible.
Let AB is a fixed line and a moving line l intersects fixed i)  = 900
line AB at V and also making constant angle  with line
AB. If the line l is rotated in such a way such that angle  V
always remains constant, then the surface generated by
line l is said to be double nappe cone. Plane

Here V is vertex, AB is axis,  is semivertical angle and line Circle


l is said to be generator of cone.
The vertex V seperates the surface generated into two parts O
P Q
called nappes.
Section of single Nappe right circular cone by a plane Circular Base

As we know that from cone we get different cross sections If plane is parallel to the base of cone or plane is
which depends upon the orientation in which plane cut the perpendicular to the axis of the cone then the resulting conic
cone. In single Nappe cone, conic section may be point, section is circle.
pair of straight lines, circle, parabola, ellipse which depends
upon the position of plane. ii)  = 

Let  is semi vertical angle of cone and  is angle between V


the plane and axis of the cone. Generator

1) Plane passess through vertex of cone 

If plane passess through vertex of cone, then also there can 


be various curves which depend upon the value of angle  parabola
or position of plane w.r.t cone. O Q
P
Plane
V

 
If  =  then plane is parallel to the generator of the cone,
and the resulting conic section under the case is parabola
A
Q
iii)  < < 900
P O

B V Plane
Plane

Ellipse
i)  <  : Under this case section of right circular cone is
pair of real and distinct straight lines. And two lines co-
P O Q
incide with two lines VA and VB.
Circular Base
ii)  =  : Under this case the plane co-incides with generator

3.2
Parabola

Here plane is not parallel to generator or axis of cone and


also not perpendicular to axis of cone, then the resulting
conic section under this case is Ellipse.
Sections of a double Nappe right circular cone by

plane
In double nappe sections of cone may be point, pair of
straight lines, circle, parabola, ellipse or hyperbola which
depend upon the angle which the plane made with the axis
of cone.
Let  is semi vertical angle of cone and  is angle which
the plane makes with axis of cone. Here plane does not intersect the cone at only one point
(i.e. vertex). hence the conic section is just a point which
Plane passess through vertex of the cone co-incides with vertex of cone.
If plane passess through vertex of the cone then following 2) Plane not passing through vertex of cone
different cases arises which depend upon the value of .
If the plane cuts any one or both nappe of cone (other then
i) 0     vertex), then then following cases arise.
i)  = 900
A

 O
B1
=900
A1

Under this case the section of right circular cone is pair of


real and distinct intersecting straight lines. The two lines
intersect at vertex of cone. The two intersecting lines are
AA1 and BB1. Section of a cone cut by plane which is parallel to the base
ii)  =  of cone is circle.
ii)  = 
Under this case plane is parallel to generator of the cone
and resulting conic is parabola.
 iii)     900

Here the plane co-incides with the generator of the cone


and so the conic section is pair of real and co-incident
0
straight lines, which co-incide with generator. <<90

iii)     900

3.3
Coordinate Geometry

Here plane is neither parallel to generator or axis of cone said to be directix of conic section on which moving point P
nor perpendicular to the axis of cone, then the resulting lies.
section of is ellipse.
vi) 0     :
M P

L=0

 PS
Also this constant ratio i.e. is known as ecentricity of
PM
conic denoted by e.
Equation of conic from focus directrix property

Let S () be the focus and L  ax  by  c  0 be the


directrix and e be the eccentricity of a conic. Let P(x, y) be
any point on the conic. Let M be the projection of P on L.
Section of right circular cone cut by a plane which intersect
both nappes such that angle of intersection lies between 00
and , is a hyperbola. If  = 00 then axis of the cone is M P(x,y)
parallel to the plane.

note S( )
ax+by+c=0
In double nappe cone, for the case of circle, parabola
and ellipse plane cuts only one nappe of the cone while
for hyperbola plane must cut both nappes of cone.
Then, by the definition of the conic
3D view of conic sections SP
 e  SP 2  e 2  PM 2
PM

(ax  by  c)2
 (x  )2  (y  )2  e 2
(a 2  b2 )
CIRCLE  The locus of P is

(a 2  b 2 ) (x  )2  (y  )2   e 2 (ax  by  c)2

On simplification, we get the equation of a conic as a second


HYPERBOLA degree equation is x and y. Therefore, the equation of a
ELLIPSE PARABOLA
conic can be taken as
ax2 + 2hxy + by2 + 2gx + 2fy + c = 0
Locus of a point which moves in a plane such that the ratio
of its distance from a fixed point to its perpendicular distance note
from a fixed straight line is always constant, is known as Ecentricity of conic can also be defined as ratio of
conic or conic section. cos  to cos  where  is angle between the plane

PS and axis of cone while  is semivertical angle of cone.


i.e.  constan t
PM cos 
i.e. e 
cos 
Here, fixed point S is said to be focus, fixed line L = 0 is

3.4
Parabola

Discrimination of conic due to eccentricity Hence, we can say that degenerate conic will always
represent a pair of straight lines through focus.
PS  cos  
By the help of ratio  or  i.e. ‘e’ we are able to Now, if ax 2  2hxy  by 2  2gx  2fy  c  0 represents a
PM  cos  
degenerate conic, then its discriminant
determine which conic the equation represents.
If value of  and  are known we will be able to determine D  abc  2fgh  af 2  bg 2  ch2  0
the orientation of plane w.r.t cone and so easily we can
Now, on the basic of eccentricity degenerate conic is divided
determine which conic it represents.
in following categories.
i) if e = 1 ( i.e.  =  ), so plane is parallel to the generator i) If e > 1 i.e. h2 – ab > 0 then conic equation will represents
and hence the resulting conic is parabola. pair of real and distinct intersecting lies which intersect at
ii) If 0 < e < 1 (i. e.  <  < 900), hence resulting conic is focus.
ellipse. ii) If e = 1 i.e. h2 – ab = 0 then the pair of lines are real and
iii) if e > 1 ( i.e. 0 <  < ), then resulting conic is co-incident.
hyperbola. iii) If e < 1 i.e. h2 – ab < 0 then the pair of lines are
iv) If e = 0 ( i.e.  = 900), so plane is perpendicular to axis imaginary intersecting at a real point (i.e. focus), or in real
system it represents just a point.
of cone and hence resulting conic is circle.
2) Non-Degenerate conic
v) If e   ( i.e.  = 900), then conic represents pair of
straight lines. If focus does not lie on directrix or if plane does not pass
through vertex of the cone then the conic is said to be non-
Discrimination of conic by location of focus w.r.t
degenerate conic.
directrix
For existence of non degenerate conic, the discriminant of
On the basis of location of focus conic is divided into two
conic equation must be non zero. i.e. if
categories.
ax 2  2hxy  by 2  2gx  2fy  c  0 is equation of non-
Degenerate conic
degenerate conic then its discriminant i.e
If focus lies on directrix then the conic is said to be be
degenerate conic. D  ab  c  2fyh  af 2  hg 2  ch 2  0

(OR) if plane passess through vertex of the cone, then the Now, on the basis of eccentricity non-degenerate conic is
resulting conic is said to be degenerate conic. divided in following categories.

Let S lies on directrix L = 0 and is taken as origin i.e. S (0, i) If e = 1 i.e. h2 – ab = 0 then conic is parabola
0) and directrix by Y-axis i.e. X = 0. Let P(x, y) is moving ii) If 0 < e < 1 i.e h2 – ab < 0 then conic is ellipse (also
PS known as emtpy set)
point, such that k
PM iii) If e = 0 i.e. a = b and h = 0 then conic is circle. (circle
is limiting case of ellipse)
iv) If e > 1 i.e. h2 – ab > 0 then conic is hyperbola
M P(x,y)

v) If e  2 i.e. h2 – ab > 0 and a + b = 0 then the conic


is rectangular hyperbola.
Definitions of various terms related to conic
L=0
Axis : The straight line passing through the focus and
perpendicular to the directrix is called the axis of the conic
x2  y2 section.
k
|x|
Vertex : The point of intersection of the conic section and
the axis is (are) called vertex (vertices) of the conic section.
 x 2  y 2  k 2 x 2 x 2 (k 2  1)  y 2
Focal distance : The distance of any point on the conic
which is pair of straight line through origin (i.e. focus). from the focus.

3.5
Coordinate Geometry

Double ordinate : A straight line drawn perpendicular to Solved Examples


the principal axis and terminated at both ends of the curve Example 1
is a double ordinate of the conic section provided the Which conic is represented by
principal axis of conic is x-axis. 25 (x2 + y2 – 2x + 1) = (4x – 3y + 1)2
Tangent at vertex : A line perpendicular to axis and Solution
passing through vertex. The given equation is
25 (x2 + y2 – 2x + 1) = (4x – 3y + 1 )2
Latus Rectum : The double ordinate chord perpendicular 2
to principal axis of conic and passing through the focus is  4x  3y  1 
25(4x  3y  1)2  25  
called the latus rectum of the conic section.  (4 2  3 2 ) 
 
Centre : The point which bisects every chord of the conic
passing through it is called the centre of the conic section. |4x  3y  1|
or (x  1)2  (y  0)2 
4 2  32
Method to determine centre of conic
Here e = 1
2 2
If S  ax  2hxy  by  2gx  2fy  c  0 Thus the given equation represents a parabola. It may
be noted that (1, 0) is the focus and 4x – 3y + 1 = 0
Partially differentiating w.r.t x and y, we get is the directrix of the parabola.
Example 2
S S Find the conic represented by the equation
 2ax  2hy  2g ;  2hx  2by  2f
x y
ax  by  1 ?
Solution
S S
For centre,  0 and 0
x y The given equation is ax  by  1
Squaring both sides, we get
 2ax + 2hy + 2g = 0 and 2hx + 2by + 2f = 0
ax  by  2 abxy  1  ax  by  1  2 abxy
or ax + hy + g = 0 and hx + by + f = 0
Again squaring both sides, we get
Solving these equations we get the centre (ax + by – 1)2 – 4abxy
 hf  bg gh  af   a 2 x 2  b 2 y 2  1  2abx  2by  2ax  4abxy
(x, y)   2
, 2 
 ab  h ab  h   a 2 x 2  b 2 y 2  2abxy  2ax  2by  1  0
How to Remember  a 2 x 2  2abxy  b 2 y 2  2ax  2by  1  0 ....(1)
Comparing the equation (1) with the equation
a h g Ax2 + 2Hxy + By2 + 2Gx + 2Fy + C = 0,
A = a2, H = –ab, B = b2, G = –a, F = –b, C = 1
Since   h b f
   ABC  2FGH  AF 2  BG2  CH 2
g f c
Write first two rows,  a 2 b 2  2a 2 b2  a 2 b 2  a 2 b 2  a 2 b 2
 4a 2 b 2  0 and H 2  a 2 b 2  AB
a h g a
i.e. h (Repeat 1st column) So we have   0 and H2 = AB.
b f h Hence the given equation represents a parabola.
Example 3
 ab  h 2 ,hf  bg,gh  af If the equation x2 – y2 – 2x + 2y += 0 represents a
degenerate conic then find the value of .
 hf  bg gh  af   C13 C 23  Solution
or points  2
, 2  or 
,  For degenerate conic  = 0
 ab  h ab  h   C 3 C33 
Comparing the given equation of conic with
OR ax 2  2hx  by 2  2gx  2fy  c  0
According to first two rows ax + hy + g = 0 and  a  1, b  1h  0, g  1, f  1,c  
hx + by + f = 0
   abc  2fgy  af 2  bg 2  ch2  0
After solving the above equations we will be able to
determine the centre of the conic. (1)(1)()  0  1  (1)2  1  (1)2  (0)2  0

3.6
Parabola

or   1  1  0   0 Solution
Comparing this equation with ax2 + by2 + 2hxy +
Example 4
2gx + 2fy + c= 0 we have a = 9, b = 4, c = 3,
Find the centre of the conic
h = , g = 2, f = – 1
14x2 – 4xy + 11y2 – 44x – 58y + 71 = 0
If the equation 9x2 + 4y2 + 4xy + 4x – 2y + 3 = 0
Solution
represents the parabola then its second degree terms
Let f(x, y)  14x 2  4xy  11y 2  44x  58y  71  0 must form the perfect square.
Differentiating partially w.r.t to x and y then   2  36 (using h2 – ab = 0)    6
f Also for these values of ,   0 .
 28x  4y  44
x Example 7
Find the equation of a conic section whose focus is at
f
and  4x  22y  58 (–1,0), directrix is the line 4x – 3y + 2 = 0 and eccen-
y
1
f f tricity is .
 0 and 0 2
For centre y
x Solution
 28x  4y  44  0 Let P(h, k) be a point on the conic. Then by definition
SP = e PM
7x  y  11  0 ...(1)
4x  22y  58  0 ...(2) 1 4h  3k  2
 (h  1)2  (k  0)2 
Solving (1) and (2) we get x = 2 and y = 3 2 4 2  (3)2
Centre is (2, 3)
Example 5 1
 (h  1)2  k 2  (4h  3k  2)2
For what value of  the equation of conic 2xy + 4x – 50
6y +  = 0 represents two intersecting straight lines ? So, the locus of (h, k) is
If  = 17 then, what does this equation represent ?
Solution 1
(x  1)2  y 2  (4x  3y  2)2
Comparing the given equation of conic with 50
ax2 + 2hxy + by2 + 2gx + 2fy + c = 0, we get
 50 (x  1)2  y 2   (4x  3y  2)2
 a  0, b  0,h  1,g  2,f  3,c  
34x2 + 24xy + 41y2 + 84x + 12y + 46 = 0 is the
for two intersecting lines
required conic.
h2 – ab > 0,  = 0
Example 8
 ab  0, h  1,  h 2  ab  1  0 Determine the name of the curve described parametri-
and   abc  2fgh  af 2  bg 2  ch 2 cally by the equations x  t 2  t  1, y  t 2  t  1
Solution
 0  2(3)  2  1  0  0  (1)2  0    12
We have,
For   17 , the given equation of conic becomes 2xy
x  t 2  t  1 and y  t 2  t  1
+ 4x – 6y + 17 = 0
 a = 0, b = 0, h = 1, g = 2, f = –3, c = 17  x  y  2(t 2  1) and x  y  2t
   abc  2fgh  af 2  bg 2  ch 2  0  x  y 2 
2
 x  y  2    1
 0  2  3  2  1  0  0  17  (1)  2  
 12  17  29  0
 2(x  y)  (x  y)2  4
  0
and h2 – ab = 1 > 0  x 2  y 2  2xy  2x  2y  4  0
Comparing this equation with the equation
So we have   0 and h2 – ab > 0; and a + b = 0
Hence the given equation represents a rectangular hy- ax 2  2hxy  by 2  2gx  2fy  c  0 , we get
perbola. a = 1, b = 1, c = 4, h = –1, g = –1 and f = –1.
Example 6
 abc  2fgh  af 2  bg 2  c 2  4  2  1  1  4  0
Find the value of  if equation 9x2 + 4y2 + 4xy + 4x
– 2y + 3 = 0 represents parabola. and, h 2  ab  1  1  0
Thus, we have   0 and h 2  ab .

3.7
Coordinate Geometry

So, the given equations represent a parabola. Example 11


Example 9 If the equation of conic 2x2 + xy + 3y2 – 3x + 5y +
Determine the name of the conic represented by each = 0 represents a single point, then find the value of .
of the following equations : Solution
(i) 13x 2  18xy  37y 2  2x  14y  2  0 For the equation to represent a single point h2 < ab
and  = 0. Comparing the given equation with
(ii) px  qy  1 , where p,q  R, p,q  0 ax 2  2hx  by 2  2gx  2fy  c  0 ,
Solution
(i) Comparing the given equation with the equation 1 3 5
a = 2, h  , b  3, g  ,f  ,c  
ax2 + 2hxy + by2 + 2gx + 2fy + c = 0 2 2 2
we get
1 23
a = 13, b = 37, c = –2, f = 7, g = 1 and h = –9  ab  h2  6   0
4 4
   abc  2fgyh  af 2  bg 2  ch 2
and   abc  2fgh  af 2  bg 2  ch 2
 962  126  637  37  162  0
5 3 1
and, h 2  ab  81  13  37  0  (2)(3)()  2   
2 2 2
Thus, we have   0 and h 2  ab
25 9 1
So, the given equation represents an ellipse. 2   3  
4 4 4
(ii) We have,
px  qy  1  { px  qy}2  1 15 25 27  23
 6       23  0
4 2 4 4 4
 px  qy  2 pq xy  1  (px  qy  1)2  4pq xy 4
2 2 2 2
 p x  2pq xy  q y  2px  2qy  1  0 Make Concepts Clear 3.1
Comparing this equation with the equation 1. Find the value of a for which the equation ax2 + 4xy
+ y2 + ax + 3y + 2 = 0 represents a parabola.
ax 2  2hxy  by 2  2gx  2fy  c  0 , we get
2. Find centre of conic whose equation is 4x2 + 12x –
a  p 2 , b  q 2 , c  1, g   p, f  q and h   pq 20y + 67 = 0
3. For what value of k the equation 2xy+4x – 6y + k=0
   abc  2fgh  af 2  bg 2  ch 2 represents pair of intersecting straight lines. If k = 12
then which conic the given equation represents
 p2q 2  2p 2q 2  p 2q 2  p2q 2  p 2q 2
4. Find name of locus which moves in such a way that
 4p 2q 2  0 the ratio of its distance from (1, 2) to its perpendicu-
and, h 2  ab  p2q 2  p 2q 2  0 lar distance from 4x – 3y + 2 = 0 is 3.
Thus, we have   0 and h2 = ab, so,the given equa- 5. Find the centre of xy – 2x – 4y + 7 = 0
tion represents a parabola. 6. Show that the locus of a point whose sum of the
Example 10 distances from the origin and the line x = 2 is 4 units
If the equation x2 + y2 – 2x – 2y + c = 0 represents is a parabola. Sketch the curve.
an empty set then find the value of c.
Solution Answer Key
For emtpy set   0 and h2 < ab
 3 
Now comparing the given equation of conic with 1. 4 2.   , 0 
 2 
ax 2  2hxy  by 2  2gx  2fy  c '  0
3.k = –12, Rectangular hyperbola.
then a = 1, h = 0, b = 1, g = –1, f = –1, c '  c 4.Pair of intersecting straight lines.
2
h2 = 0, ab = 1  h  ab is satisfied 5.(4, 2)
Solutions Are On Page No. 3.104
  abc ' 2fgh  af 2  bg 2  c ' h2  0
 (1)(1)(c)  0  1  (1)2  1  (1)2  0  0
 c  2  0 c  2 3.2 Standard and General
Hence c  R  {2}
Equation of Parabola
Parabola is the locus of a point which moves in a plane

3.8
Parabola

such that its distance from a fixed point (i.e. focus) is equal (ii) (x, y)  C and y  0  x  0 . Thus the curve meets
to its distance from a fixed straight line (i.e. directrix).
the x-axis at only one point (0, 0). Hence the parabola has
Standard Equation of Parabola only one vertex.

Let S be the focus, ZZ ' be the directrix of a parabola. (iii) If x < 0 then there exists no y  R as y 2  4ax Thus
Draw SK perpendicular from S on the directrix and bisect the parabola does not lie on the left of y-axis (i.e., in the
SK at A. Then second and third quadrants).
AS = AK
(iv) If x > 0 then y 2  4ax  y has two real values which
 Distance of A from the focus = Distance of A from the are equal in magnitude but opposite in sign. Hence the
directrix parabola lies in the first and fourth quadrants.
 A lies on the parabola (v) (x, y)  C, x  0  y 2  0  y  0,0 . Thus y axis meets
Let SK = 2a. Then, the parabola in two coincident points and hence y-axis
AS = AK = a. touches the parabola at (0, 0).

Now, let us choose A as the origin, AS as x-axis and AY a  x  0 is the tangent to the parabola at the vertex
line perpendicular to AS as y-axis. Then, the coordinates of A (0, 0).
S are (a, 0) and the equation of the directrix ZZ' is
(vi) x    y 2    y   . Therefore the curve is
x + a = 0 , Then,
not bounded (not closed) on the right side of the y-axis.
PM  NK  AN  AK  x  a
Standard results related to parabola y2 = 4ax

Z y
y
L P Q (h, 2 ah )
P M
M Focal chord
Latus
x’ K A rectum
x (0,0)
Double ordinate
N S(a,0)
Directrix

S(a,0) N
x
focus Z A
a 2
y =4ax
x=-a
Focus Axis
Z’ y’ P’
L’ Q’
Directrix
Now, P lies on the parabola

 SP  PM  SP 2  PM 2 1. Axis
The straight line which passes through focus of the parabola
 (x  a)2  (y  0)2  (x  a)2  y 2  4ax
and perpendicular to the directrix of the parabola is called
This is the required equation of the parabola in its standard its axis.
form. For the parabola y2 = 4ax, x-axis is its axis.
note In y2 = 4ax, power of y is even, so it is symmetrical about
A parabola has two foci situated on its axis. One of its axis (i.e., x-axis)
which is at focus S and the other lies at infinity and its If the point (x, y) lie on the parabola y2 = 4ax then the
corresponding directrix is also at infinity. point (x, – y) also lies on it. Hence the parabola is
Tracing of parabola y2 = 4ax symmetrical about axis of parabola.

Let C be the curve represented by y2 = 4ax where a > 0. 2. Vertex


Then The point of intersection of the parabola with its axis is
called its vertex. For the parabola y2 = 4ax.
(i) (x,y)  C  (x, y)  C . Thus the curve is symmetric
about the x-axis. The x–axis is the principal axis of the A (0, 0) i.e., the origin is the vertex.
parabola.

3.9
Coordinate Geometry

3. Double ordinate Parametric Equation of standard parabola y2 = 4ax


If Q be the point on the parabola, draw QN perpendicular
y 2x
to the axis of parabola and produced to meet the curve Equation of parabola y2 = 4ax can be written as 
2a y
again at Q' then QQ' is called a double ordinate.
....(1)
If abscissa of Q is h then ordinate of Q is 2 ah , and Let ‘t’ is the parameter, then equation (1) can also be written
ordinate Q' is y  2 ah y 2x
as   t which gives y  2at
2a y
Hence co-ordinates of Q and Q' are (h, 2 ah) and
2x = ty  2x  t.2at  x  at 2
(h, 2 ah) respectively..
Therefore, the equations x = at2 and y = 2at are called
4. Latus rectum
parametric equation of parabola y2 = 4ax.
The double ordinate LL ' passes through the focus called
And (at2, 2at) are parametric co-ordinates of y2 = 4ax.
the latus rectum of the parabola.
note
Since focus S(a, 0) the equation of latus rectum of the
parabola is x = a. Parametric co-ordinates for a parabola are not unique,

Hence the co-ordinates of the extremities of the latus rectum  at 2 at   at 2 


it can be  4 , 2  , 2 ,at  etc., but in standard case
are L(a, 2a) and L '(a, 2a) respectively..    
we have to take (at2, 2at).
Since LS  L ' S  2a
Other Standard forms of Parabola
Length of latus rectum LL '  2(LS)  2(L ' S)  4a .
1) Right handed parabola
5. Focal chord
The parabola which opens towards right side (i.e. towards
A chord of a parabola which is passing through the focus is positive X-axis) is said to be right handed parabola. Equation
called a focal chord of the parabola. of this parabola is y2 = 4ax where a > 0.

In the given figure PP ' and LL ' are the focal chords. (i) Vertex A = (0, 0),
(ii) Focus S = (a, 0)
note
(iii) Equation of the directrix is x + a = 0
In objective questions use LL ' as focal chord and in
subjective questions use PP ' as focal chord. (iv) Equation of the axis is y = 0

6. Focal distance (v) Equation of the tangant at the vertex is x = 0

The focal distance of any point P on the parabola is its (vi) Length of the latus rectum LL '  4a
distance from the focus S i.e., SP
(vii) Extremities of latus rectum are L(a, 2a) and L '(a, 2a)
SP  PM  distance of P from the directrix
(viii) Equation of latus rectum is x = a
Let S be (x ,y), then
(ix) parametric co-ordinates (at2, 2at)
SP = PM = x + a
note L
1) Two parabolas are said to be equal if they have
same length of latus rectum. x
Z A S
2) Point of intersection of axis and directrix of parabola
is said to be foot of directrix i.e. foot of perpendicular L’
Y’
from focus on directrix is foot of directrix. x=0
3) Perpendicular distance from focus to directrix is 2) Left handed parabola
equal to semi latus rectum. The parabola which opens towards left side (i.e. towards

3.10
Parabola

negative x-axis). Equation of this parabola is y2 = – 4ax 4) Downwards parabola


when a > 0
Parabola which is opens downwards. Equation of this
(i) Focus S = (–a,0), parabola is x2 = – 4ay where a > 0.
(ii) Vertex A = (0, 0) (i) vertex A = (0, 0)
(iii) Equation of the directrix is x – a = 0 (ii) Focus S (0, –a)
(iv) Equation of the axis is y = 0 (iii) Equation of the directrix is y – a = 0
(v) Equation of the tangant at the vertex is x = 0 (iv) Equation of the axis is x = 0

(vi) Length of the latusrectum is LL '  4a (v) Equation of the tangant at the vertex is y = 0

(vii) Extremities of latusrectum are L(–a, 2a) and (vi) Length of the latus rectum is LL '  4a
L '(a, 2a) (vii) Extremities of latus rectum are L (2a, –a) and
(viii) Equation of latusrectum is x = – a L '(2a, a)
ix) Parametric co-ordinates (–at2, 2at) (viii) Equation of latusrectum is y = –a.

Y (ix) Parametric co-ordinates (2at, –at2)

L
Y
(0,-a)
S A Z x Z
A (0,0)
X y=0
L’ L’ L
S
x=0
3 ) Upward parabola
x=0
Parabola which is open upwards. Equation of this parabola
is x2 = 4ay where a > 0.
Non-Standard forms of Parabola
(i) Vertex A = (0, 0)
Parabola havings its vertex at () and axis parallel to
(ii) Focus S = (0, a)
any of the co-ordinate axes.
(iii) Equation of the directrix is y + a = 0
These are of two types
(iv) Equation of the axis is x = 0
1) Parabola opening upwards
(v) Equation of the tangent at the vertex is y = 0
This type of parabola is as shown below :
(vi) Length of the latusrectum is LL '  4a
y
(vii) Extremities of latusrectum are L(2a, a) and L '(2a,a)
S
(viii) Equation of latusrectum is y = a L’ L

(ix) parametric co-ordinates (2at, at2) A()


x
Y O

S(a,0)
L’ L Its equation is given by (x –)2 = 4a(y –) ...(iii)

x and other related terms are as given below.


A(0,0) y=0

Z (i) Focus S  ,   a 

x=0 (ii) Vertex A  ,  

3.11
Coordinate Geometry

(iii) Equation of axis of parabola : x = or x–  = 0 2


 b 2  4ac   b 
(iv) Equation of tangent to parabola at vertex  y  2   a x  
 4a   2a 
y =  or y –  = 0
2
(v)Extremities of latus rectum LL '  (  2a,   a) ;  b  1 b2  4ac 
 x    y  
(  2a,   a)  2a  a 4a 2 

(vi) Equation of directrix : y    a   b (b2  4ac) 


If we shift the origin at  2a ,  4a 2  then above
(vii) Parametric equation of parabola : x    2at  
y    at 2 2 1
equation becomes x  y
2) Parabola opening downwards a

This type of parabola is as shown below : Therefore, the above equation represents a parabola whose
axis is parallel to Y axis. If a > 0 then parabola open
y upwards while if a < 0 then parabola opens downwards.
Also we can determine all the parameters of such parabola
in same manner as we do for above two types.
A()
P(x,y) 2
L’ L 4) Equation of type x=ay +by+c also represents a
S parabola
x
O
 b c
 x  a  y2  y  
 a a
Its equation is given by (x  )2  4a(y  ) ...(iv)
2
And other related terms are as given below  b  (b2  4ac) 
 x  a  y   
 2a  4a 2 
(i) Focus S  ,   a 
2
(ii) Vertex A  ,    b  1 b2  4ac 
 y    x  
 2a  a 4a 2 
(iii) Equation of axis of parabola : x =  or x–  = 0
(iv) Equation of tangent to parabola at vertex :  (b  4ac)  b 
If we shift the origin at   ,  then the above
y =  or y –  = 0  4a 2 2a 
(v)Extremities of latus rectum 1
2
L,L '  (  2a,   a) (  2a,   a) equation becomes y  x
a

(vi) Equation of directrix : y    a Therefore, the above equation represents a parabola whose
axis is parallel to x-axis. If a > 0 then parabola opens
(vii) Parametric equation of parabola : x    2at rightwards while if a < 0 then parabola opens leftwards.
y    at 2 note
Second degree general equation of conic i.e.
3) Equation of type y=ax 2+bx+c also represents a
parabola ax 2  2hxy  by 2  2gx  2 fy  c  0

will represent above forms of parabola if


 b c
y  a x2  x   h = 0 and out of a and b one must be zero i.e. either
 a a
a = h = 0 or b = h = 0

 2
b  (b2  4ac)  General form of Parabola
 y  a  x    
 2a  4a 2  Let S(a, b) be the focus, and lx + my + n = 0 is the

3.12
Parabola

equation of the directrix. Let P(x, y) be any point on the


y
parabola. Then by definition SP = PM

P
Y M L
A
M

)
x, y
0

P(
A L
=

x
+n

Z O
my

S(a,b)
lx+

L’ X
O Let the length of latus rectum = 4a
Then equation of parabola is given by (PL)2 = 4a(PM)
|lx  my  n|
 (x  a)2  (y  b)2   lx  my  n   mx  ly  k 
(l 2  m 2 )    4a  
2 2 2 2
 l m   m +l 
2
|lx  my  n|
 (x  a)2  (y  b)2   (lx  my  n)2   4a(mx  ly  k) l 2  m 2
(l 2  m 2 )
which gives us two possible equations of parabola as shown
 m 2 x 2  l 2 y 2  2lmxy  x term in the above diagram.
 y term  cons tan t  0
Applications of Parabola
This equation is the general form of parabola. Parabola is a very important curve and it has very wide
applications in our day to day life.
note
1) So, from above equation, we note that in general Following are some examples to illustrate the role of parabola
form of parabola second degree terms will form a is several fields.
perfect square 1) The path of a projectile is a parabola knowing the
2) Equation of line inside the bracket of second degree equation of the path of a projectile, we can find horizontal
terms of equation of parabola i.e. range, greatest height attained and velocity at a particular
time
mx – ly = 0 is a line parallel to axis of parabola.
3) Equation of axis of above parabola is a line parallel
to mx – ly = 0 and passing through focus (a, b). S(focus)
Therefore equation of axis is
m(x – a) – l(y – b) = 0
Oblique Parabola
2) Parabolic reflectors have the property that light rays or
In standard parabola y2 = 4ax. Let P is any moving point sound waves coming parallel to its axis converges at the
on parabola, and PM and PN are length of perpendicular focus and light rays or sound waves coming from the focus
from P on axis and tangent at vertex respectively of parabola. are reflected parallel to the axis. Because of this property.
Then PM = y and PN = x parabolic reflectors are used in cars, automobiles,
Therefore, equation of parabola whose length of latus loudspeakers, solar cookers, telescopes etc.
rectum is 4a can also be written as PM2 = 4a (PN) 3) If the roadway of a suspension bridge is loaded uniformly
In the similar way we can write the equation of oblique per horizontal metre, the suspension cable hangs in the form
parabola i.e. the parabola whose axis is not parallel to co- of arcs which closely approximate to parabolic arcs.
ordinate axes. Therefore, parabolic arcs are used in suspension cable bridge
construction.
Let the equations of axis of parabola and tangent at vertex
be lx + my + n = 0 and mx – ly + k = 0 respectively. Let 4) If a particle moves under the influence of inverse square
A() be the vertex and P(x, y) be any point on the field, then its path may be parabolic.
parabola as shown :

3.13
Coordinate Geometry

Solved Examples 67 67
 y 2  3x  5y   0  y 2  5y  3x 
Example 12 4 4
Find the equation of the parabola whose focus is at
2 2
(–1, –2) and the directrix is x – 2y + 3 = 0. 5 67  5 
 y 2  5y     3x   
Solution 2
  4  2
Let P (x, y) be any point on the parabola whose focus
2 2
is S (–1, –2) and the directrix x – 2y + 3 = 0. Draw  5 42  5  7
PM perpendicular to directrix x – 2y + 3= 0. Then by   y    3x    y    3  x   ...(i)
 2 4  2  2
definition,
2 2 7 5
SP = PM  SP  PM Let x  X  , y  Y  ....(ii)
2 2
2
 x  2y  3  Using these relations, equation (i) reduces to Y 2  3X
 (x  1)2  (y  2)2   
 1 4  ...(iii)

y
M P(x,y)
7 ,5
x-2y+3=0

S(-1,-2) 22 y= 5
A 2

x’ x
0
2 2
 4x  y  4xy  4x  32y  16  0 x= 7
2 y’
This is the equation of the required parabola.
Example 13 This is of the form Y 2  4aX . On comparing, we
Find the equation, axis, Latus rectum and vertex of
the parabola whose focus is (0, 0) and directrix is x get 4a  3  a  3 / 4
+ y + 1 = 0. Vertex
Solution The coordinates of the vertex are (X = 0, Y = 0) So,
Let (x, y) be a point on the parabola. Then by definition  7 5
the coordinates of the vertex are   , 
 x  y 1
2
 2 2
(x  0)2  (y  0)2   
 2  Axis
The equation of the axis of the parabola is Y = 0. So,
 x 2  y 2  2xy  2x  2y  1  0
5
which is the equation of the parabola. the equation of the axis is y 
2
Axis : It is a line passing through (0, 0) and
perpendicular to the directrix x + y + 1 = 0. Hence Focus
its axis is x = y. The coordinates of the focus are (X = –a, Y = 0)
LR : It is a line through (0, 0) and parallel to i.e. (X = –3/4, Y = 0)
x + y + 1 = 0. Hence its equation will be x + y = 0 So, the coordinates of the focus are (–17/4, 5/2)
Length of LR Directrix

 2  (dis tan ce of S from x  y  1  0)  2 The equation of the directrix is X = a i.e. X  3


4
Vertex :
The point of intersection of axis x – y = 0 and directrix 11
So, the equation of the directrix is x  
x + y + 1 = 0 i.e. foot of directrix is 4
Z  (1 / 2, 1 / 2) Latus rectum
Now vertex is the mid point ZS, so it will be (–1/4, –1/ The length of the latus rectum of the given parabola
4). is 4a = 3.
Example 14 Example 15
Find the vertex, axis, focus, directrix, latusrectum of Find the coordinates of the points on the parabola y2
the parabola 4y2 +12x – 20y + 67 = 0
5
Solution = 2x whose focal distance is .
The given equation is 2

4y 2  12x  10y  67  0

3.14
Parabola

Solution
x1  4
4  x1  4
1 1 2
y 2  2x  y 2  4   x  a 
 2 2
y1  3
1   y1  1 so Z  (4,1)
1 2  2
Let P  t , t  represent the required points. Directrix is parallel to x-axis and passes through
2 
(4 , 1), so equation of directrix is y = 1 or y – 1 = 0
The focal distance of P is Let P(x, y) be any point on the parabola. Join SP and
1 2 1
2
25 draw PM perpendicular from P on directrix. Then by
2
 t   t   t2  4 definition SP = PM
 2 2  4
(SP)2 = (PM)2
So, the required points are (2, –2) and (2,2)
2
Example 16 | y  1|
If Q is the foot of perpendicular from a point P on (x  4)2  (y  3)2   
2
 1 
the parabola y2 = 8(x – 3) to its directrix. S is the
focus of the parabola and if SPQ is an equilateral (x  4)2  (y  3)2  (y  1)2
triangle then find the length of the side of the triangle.
Solution x 2  8x  8y  24  0
Example 18
y
Q P Find the equation of the parabola whose axis is parallel
60
0
0
60 to x-axis and which passes through points (0, 0) ,
O 600 (1, 1) and (2, 3).
x Solution
Z A S
Let O  (0, 0), A  (1,1), B  (2, 3)
As the axis of the parabola is parallel to x-axis, so its
equation may be taken as
Given parabola is y2 = 8(x – 3) (y – )2 = 4a(x – ) ...(1)
Vertex A = (3, 0) Focus S = (5, 0) As parabola (1) passes through O (0, 0), A (1, 1) and
SPQ is an equilateral triangle B(2, 3)
  PSQ  60 0  PQS  SQZ  30 0 2 = – 4a ...(2)
(1 – )2 = 4a (1 – ) ...(3)
SZ 4 and (3 – )2 = 4a (2 – ) ...(4)
From the SQZ , sin 300  
SQ SQ On solving above equations we get
 SQ  8 which is the length of the side 49 7 3
 ,   and a  
Example 17 24 2 2
Find the equation of the parabola whose focus is So, required equation of parabola is
(4, –3) and vertex is (4, –1).
2
Solution  7  49 
Let A(4, –1) be the vertex and S(4, –3) be the focus.  y    6  x  
 2  24 
3  1 Example 19
Slope of AS  
Find the equation of the parabola whose latus rectum
44
So, axis is parallel to y-axis and directrix is parallel to is 4 units, axis is the line x – 4y – 4 = 0 and the
x-axis. tangent at the vertex is the line 4x – 3y + 7 = 0
Solution
Z y-1=0 Axis of the parabola is
Y M
3x + 4y – 4 = 0 ...(1)
O Equation of tangent at the vertex is
A(4,-1) 4x – 3y + 7 = 0 ...(2)
Let A be the vertex, S the focus, BC the directrix DE
P(x,y)
the latus rectum of the parabola.
we know that standard equation of a parabola is
S(4,-3)
y2 = 4ax ...(3)
Y’

Let(x1 ,y1) be foot of directrix, then mid point of SZ

3.15
Coordinate Geometry

2
B L  1 7 29 
N K D P i.e.  x      y   ...(1)
 2 3 28 
H
Q A S
29 3
If we choose y    t2 ...(2)
C E 28 7
M
where 4a is the length of latus rectum. From equation it makes the right hand side of equation (1) a perfect
(3), we conclude that if P(x, y) be any point on the square and gives
parabola, then equation of parabola can be obtained 1
by x t ....(3)
2
PH2 = (length of latus rectum). PK
Equation (2) and (3) are therefore a convenient choice
 3x  4y  4 
2
 4x  3y  7  for the parametric equations of the parabola.
   4  Example 22
2
 3 4 
2  (4 2  (3)2 
  A variable line passing through a fixed point P cuts a
(3x + 4y – 4)2 = 20(4x – 3y + 7) given parabola at A and B. Now a point Q moves on
This is the required equation of the parabola. the line PAB such that PA, PQ and PB are in harmonic
Example 20 progression. Prove that the locus of Q is a straight
Find equation to the parabola whose vertex and focus line.
Solution
are on the x-axis at distances a and a ' respectively
Let us choose the equation of the parabola as
from the origin
y2 = 4cx ...(1)
Solution
and the fixed point as P(a, b).
Since the vertex and focus of the parabola are on x-
Let  be the angle that the variable line through P
axis, therefore, axis of the parabola is x-axis.
makes with the axis of the parabola .
Let A be the vertex and S the focus of the parabola.
Since vertex and focus of the parabola are at distance B
Y
a and a ' from the origin, therefore P

A  (a,0) and S  (a ',0) x
O
L A

A x
(a,0) S (a’,0) A point on the variable line can be chosen as
(a+r cos, b+r sin) where r denotes the distance of
M
the chosen point measured from P. As this point also
We know that equation of a parabola whose vertex is lies on the parabola represented by equation (1), then
() and axis is parallel to x-axis is we have
(y  )2  4b(x  ) (b + 4 sin  )2 = 4c(a + r cos )
where, b = distance between focus and vertex. r 2 sin 2   r(2b sin   4c cos )  b2  4ac)  0 ...(2)
Here   a,   0 and b  AS  a ' a If r1, r2 be the distances of A, B from P respectively
 Required equation of the parabola will be and r be the distance of the point Q from P which
satisfies the given condition, then we have
y 2  4(a ' a)(x  a) .
Example 21 2 1 1 2 r1  r2
  i.e. 
Find the parametric equations for the parabola r r1 r2 r r1r2
3x2 + 3x + 7y + 8 = 0. Since r1, r2 are the roots of equation (3), therefore we
Solution have
The given equation can be written as
2b sin   4c cos  b 2  4ca
r1  r2   and r1r2 
7 8 2
sin  sin 2 
x2  x  y 0
3 3 Putting the values of r1 + r2 and r1r2 in equation (3),
2 we have
 1 1 7 8
i.e.  x     y   0 2 4c cos   2b sin 
 2 4 3 3 
r b 2  4ac
i.e. b 2  4ca  2c(r cos )  b(r sin ) ...(4)

3.16
Parabola

let h(h , k) be the coordinates of Q, then we have Example 24


h  a  r cos  and k  b  r sin  QQ ' is a double ordinate of a parabola y2 = 4ax.
Putting these values in equation (4), we have Find the locus of its point of trisection.
b2 – 4ca = 2c(h – a) – b(k – b) Solution
i.e 2ch – bk + 2ca = 0. Let the double ordinate QQ ' meet the axis of the
Putting (x, y) in place of (h, k) gives the equation of parabola in N.
the required locus as 2cx – by + 2ca = 0. Let co-ordinate of Q be (x1, y1) then co-ordinate of
Example 23
Q ' be (x1 – y1) since Q lies on parabola. So
A focal chord of the parabola y 2 = 4ax, whose
perpendicular distance from vertex is b is of length c. y 12  4ax 1 ...(1)
Show tha cb2 = 4a3. Let R and T be the points of trisection of QQ ' . Then
Solution
the co-ordinates of R and T are
Equation of the given parabola is y2 = 4ax ...(1)
We have  1  x1  2  x1 1  ( y1)  2  y1   y1 
 ,  or  x 1, 
B  12 1 2   3

  2  x1  1  x1 2  ( y1)  1  y1   y1 
V S(a,0) and  ,  or  x1,  
P  21 21   3
A
respectively.
Since R divided QQ ' in 1 : 2 (internally)
VS = a , VP = b, AB = c
and T divide QQ ' in 2 : 1 (internally)
VP b
and sin    ...(2) For locus, Let R (h, k) then
VS a
Hence, equation of the focal chord AB is y1
x 1  h and  k or y1  3k
y  tan (x  a) ...(3) 3
Substituting the values of x1 and y1 in (1), then
Solving for the intersection points of the parabola and
(3k)2 = 4a(h) or 9k2 = 4ah
the chord, we have tan2 (x  a)2  4ax
Q
Y
i.e. x 2 tan 2   2a(tan 2   2)x  a 2 tan 2   0 ...(4)
R
If x1, x2 be the roots of equation (4), then
A L
x
2a(tan 2   2)
x1  x 2  ...(5)
tan2  T

2
and x 1x 2  a ...(6) Q’

Now the projection of AB along the axis is  The required locus is 9y2 = 4ax
AB cos   |x 2  x1 | Similarly, let T(h ', k ')

i.e. c 2 cos 2   (x 1  x 2 )2  4x 1x 2 y1
then x1  h' and   k ' or y1  3k '
2
3
 2a(tan2   2)  2 Substituting the values of x1 and y1 in (2) then
   4a
 tan2   (3k ')2  4a(h')
2 2 2 2 2 2
i.e. c cos   4a (1  2 cot )  4a or 9k ' 2  4ax i.e. locus is 9y 2  4ax
2 2 4 2
 4a (1  4 cot   4 cot )  4a Alternative method
Let R and T be the points of trisection of double
 16a 2 cot 2 (cot 2   1)  16a 2 cot 2  cos ec 2
ordinate QQ ' . Let (h, k) be the co-ordinates of R
i.e. c cos   4a cot  cos ec  then AL = h and RL = k
i.e. c sin 2   4a RT = RL + LT = k + k = 2k.
Since RQ  TR  Q ' T  2k
2
b  LQ  LR  RQ  k  2k  3k
i.e. c    4a [ using result (1)]
a thus the co-ordinates of Q are (h, 3k) which satisfy
i.e. cb2  4a 3
curve. So locus is 9y 2  4ax

3.17
Coordinate Geometry

Example 25
(y  k)2  4a(x  h) ...(1)
The x and y co-ordinates of any point P are expression
1 2 Since latus rectum  (3  3)2  (6  2)2  8
x = (V cos )t, y = (V sin )t  gt , where t is a
2  4a  8 i.e. a = 2
parameter and V, , g are constant Show that the
locus of the point P(x, y) is a parabola.  From(1),(y  k)2  8(x  h)
Solution Since (3, 6) and (3, –2) lie on the parabola, then
Given, x  (V cos )t (6  k)2  8(3  h)

1 2 and (2  k)2  8(3  h)


and y  (V sin )t  gt
2 Solving (2) and (3) we get k = 2
From (2) 16  8(3  h) ,
x
From (1) t   h  3  2  h = 5,1
V cos 
Hence values of (h, k) are (5, 2) and (1, 2). The
Substituting this value t in (2) then
required parabolas are
gx 2 2
y  x tan   (y  2)2  8(x  5) and (y  2)  8(x  1)
2V 2 cos 2 
Example 27
or 2V 2 cos 2 y  2V 2 sin  cos  x  gx 2 Show that the locus of a point that divides a chord of
slope 2 of the parabola y2 = 4x internally in the ratio
2V 2 cos 2  2V 2 sin  cos  1 : 2 is a parabola. Find the vertex of this parabola.
y x  x2
g g Solution
Let P(t 12 , 2t1 ) and Q(t 22 , 2t 2 ) be extremities of a chord
2V 2 sin  cos  2V 2 cos 2 
x2  x y with slope 2.
g g
2t1  2t 2
2 2
 V 2 sin  cos   V 4 sin 2  cos 2  2V 2 cos 2  t12  t 22
x    2
 y
 g  g g
2
(t1 , 2t1)
2 2 2 2 2 2
Y P
 V sin  cos   2V cos   V sin  
x    y   1 R(h,k)
 g  g  2g 
x’ A
x
2
V 2 sin  cos  V 2 sin2  (t22, 2t2)
Let x   X, y  Y Q
g 2g
Y’
2V 2 cos 2 
X2  Y, which is a parabola  t1  t 2  1 ...(1)
g
Example 26 Let R(h, k) be co-ordinates of the point which divides
Find the equation of the parabola with latus rectum PQ ratio 1 : 2 then
joining the points (3, 6) and (3, –2). 2t12  t 22 4t  2t 2
Solution h and k  1
3 3
2  6
Slope of (3, 6) and (3, –2) is   3h  2t 12  (1  t1 )2 and 3k  4t1  2(1  t1 )
33
since latus rectum is perpendicular to axis. Hence axis [from equation (1) ]
parallel to x-axis. The equation of the two possible 2
3h  3t  2t1  1 and 3k  2t1  2
1
parabolas will be of the form
Eliminating t1
Y L(3,6) 2
 3k  2   3k  2 
3h  3    2  1
 2   2 
A 9k 2  16k  4h  8  0
(1,2) S(3,2)
2
16k 4h 8  8 4 2
X  k2     0  k    h  
O
L’(3,-2) 9 9 9  9 9 9

3.18
Parabola

2
 8 4 2 (h2  k 2 )  (h2  k 2  c)  r ...(4)
Locus of R (h, k) is  y     x  
 9 9 9 If (3) touches (2), length of the perpendicular from (h,
k) on (2) must be equal to the radius
8 2
is a parabola whose vertex is  ,  . i.e. h  a  (h2  k 2  c) ...(5)
9 9
The required locus will be obtained by eliminating c
Example 28
Prove that the locus of the centre of a circle, which from (4) and (5). So putting the value of (h2  k 2  c)
intercepts a chord of given length 2a on the axis of x
from (5) in (4), we get
and passes through a given point on the axis of
y distant b from the origin, is the curve. (h 2  k 2 )  (h  a)  r
x2 – 2yb + b2 = a2. Squaring both sides,
Solution
Let the centre of the required circle be (h, k); so its h 2  k 2  h 2  a 2  r 2  2ah  2rh  2ar
equation can be written as Generalising and simplifying, we get the required locus
x2 + y2 – 2hx – 2ky + c = 0. ...(1) as y = 2x (r – a) + (a – r)2 which is clearly a parabola.
The circle passes through a point on y-axis which is at Example 30
a distance of b from the origin the co-ordinates of the A line PQ meets the parabola y2 = 4ax in R such that
point will be (0, b). These co-ordinates will satisfy (1); PQ is bisected at R. If co-ordinates of P are (x1, y1).
hence Find locus of Q.
0 + b2 – 0 – 2kb + c = 0 Solution
or b2 – 2kb + c = 0 ...(2) Let the co-ordinates of Q be (h, k) since R is the mid
Again, solving (1) with x-axis i.e. y = 0, we get point of PQ and R lies on parabola.so,
x2 – 2hx + c = 0 ...(3) Y
If the circle cuts x-axis at A and B, the abscissae of A
and B say x1 and x2 are given by (3). The distance AB Q
2 A
x
will be (x 1  x 2 )  {(x 1  x 2 )  4x 1x 2 }  2a
R
2a  {(2h)2  4c} P

or 4a 2  4h 2  4c or h 2  c  a 2  0 ...(4)
x1  h x1  h
The required locus will be obtained by eliminating c at 2  2
i.e. t  ...(1)
2 2a
from (2) and (4). So adding these two, we get
b2 – 2kb + h2 – a2 = 0. y1  k y1  k
Generalising, the required locus is 2at  i.e. t  ...(2)
2 4a
x2 – 2yb + b2 – a2 = 0
From equations (1) and (2),
Example 29
2
If a circle be drawn so as always to touch a given  y1  k   x 1  h 
straight line and also a given circle, prove that the    
 4a   2a 
locus of its centre is a parabola.
Solution  (y1  k)2  8a(x1  h)
Consider the centre of the fixed circle as origin, the Hence locus of Q (h, k) is (y + y1)2 = 8a(x + x1)
axes being perpendicular and parallel to the given line Example 31
respectively, the equation of the given circle will be P(h, k) is a fixed point in the plane of the parabola y2
x2 + y2 = r2 ...(1) = 4ax. Through P a variable secant is drawn to cut
and of the line x = a ...(2)
the parabola in Q and R  T is a point on QR such
where r is the radius of the circle and a is the distance
of the centre of the given circle from the line. that PQ  PR  PT 2 . Find locus of T..
Let the centre of the other circle be (h, k). So its Solution
equation may be written as Let line through P makes angle  with positive direction
x2 + y2 – 2hx – 2ky + c = 0 ...(3)
of X-axis, then equation of the line passing through P
Radius of (3) is (h2  k 2  c) is
If (1) touches (3), the distance between the centres xh yk
 r ...(i)
must be equal to the sum of the radii. So cos  sin 

3.19
Coordinate Geometry

Thus x  h  r cos  and y  k  r sin  .  P lies on (2) so we have


This general point satisfy the parabola  9   3
h  16(0.75)2  16(0.75)  16    16  
2
 (k  r sin )  4a(h  4 cos )  16  4
= – 9 + 12 = 3
 r 2 sin 2   2r(k sin   2a cos )  k 2  4ah  0
 Required height PD = 3m
which is a quadratic equation in r. Consider r1, r2 be Example 33
its two roots corresponding to PQ and PR. The cable of a uniformly loaded suspension bridge
k 2  4ah hangs in the form of a parabola. The roadway which
Thus, r1r2  PQ  PR  is horizontal and 100 m long is supported by vertical
sin 2 
wires attached to the cable, the longest wire being
k 2  4ah 30m and the shortest being 6m. Find the length of the
 PT 2  ...(ii) supporting wire attached to the roadway 18m from
sin 2 
the middle.
k Solution
From (i)  PT ...(iii)
sin  The bridge is hung by supporting wires in a parabolic
From (ii), (iii) arc with vertex at the lowest point and the axis vertical.
We take the coordinate axes as given in the figure.
(  k)2 k 2  4ah
:  , i.e.(  k)2  k 2  4ah y
sin 2  sin 2 
A S us (50,24) B
2 pe
So, locus is  y  k   k 2  4ah ns
ion
ca P
Example 32 b le O L x
A water jet from a fountain reaches its maximum
height of 4 m at a distance 0.5 m from the vertical Roadway M Q N
passing through the point O of water outlet. Find the
height of the jet above the horizontal OX at a distance The equation of the parabola may be taken as
of 0.75 m from the point O. x2 = 4ay, a > 0 ..(1)
Solution Shortest supporting vertical wire = OM = 6m
The path of the water jet is parabolic. Let the equation longest supporting vertical wire = NB = 30m
of the water jet being parabola is  LB  30m  6m  24m
y = ax2 + bx + c ...(1)
the path is symmetrical to the line AB, (the maximum
1
Also OL  .100m  50m
height) so it strikes the x-axis at E such that 2
AE = OA = 0.5 m  B  (50, 24)
i.e, OE = 2OA = 2(0.5 ) = 1 m Since B (50, 24) lies on the parabola (1)
Co-ordinates of B and E are (0.5, 4) and (1, 0)
respectively 2500
 (50)2  4a.24  a 
96
Y B(0.5,4)
Let PQ be the vertical supporting wire at a distance of
P
18m from M. If PQ = k metres, then P  (18, k  6)
4
Since P lies on parabola (1),

E(1,0) X 2500 324  24


O A D  18 2  4. (k  6)  k 6
96 2500
Since O, B, E lie on (1)
324  24
1 1 k   6  9.11 metres (approx.)
 0  c, 4  a  b  c,0  a  b  c 2500
4 2
Hence the length of the required wire = 9.11 metres
Solving these we get a = – 16, b = 16, c = 0
From (1), the equation of the parabola is Make Concepts Clear 3.2
1. The girder of a railway bridge is a parabola with its
y  16x 2  16x ...(2)
vertex at the highest point, metres above the ends. If
Let P be a point on the parabola (2), such that P is at the span is 100 metres, find its height at 20 metres
a distance 0.75 m from y-axis and let P is at a distance from the mid-point.
h from x-axis 2. Two parabolas have a common axis and concavities
 co-ordinates of P is (0.75, h) in opposite directions; if any line parallel to the

3.20
Parabola

common axis meet the parabolas in P and P' , prove


3.3 Position of point, Chord and
that the locus of the middle point of PP' is another
parabola, provided that the latus rectums of the given Tangent of Parabola
parabolas are unequal. Position of point w.r.t parabola
3. An equilateral triangle is inscribed in the parabola
y2 = –8x, where one vertex of triangle coincides with Here we have to determine wheather the given point is inside
vertex of the parabola. Find the length of the side of the parabola, outside the parabola or on the surface of
the triangle. parabola.
4. If a parabolic reflector is 20 cm in diameter and 5 cm A parabola divides the plane into three regions.
deep find the focus.
5. An arch is in the form of a parabola with its axis i) The region containing the focus called interior of the
vertical. The arch is 10 m high and 5m side at the parabola.
base. How high is it 2m from the centre ?
ii) The region consisting of the curve itself.
6. A variable circle passes through (1,0) and touches the
curve y = tan (tan–1x). Find the locus of centre of the iii) The region containg the remaining portion of the plane,
circle called the exterior of the parabola.
7. Find the equation of the parabola whose axis is parallel
to the y-axis and also passes through the points (0, 4) Parabola
(1, 9) and (4, 5) and determine its latus rectum.
8. The vertex A of a parabola is joined to any point P on
the curve and PQ is drawn at right angles to AP to Exterior(outside) Interior (inside)
meet the axis in Q. Prove that the projection of PQ
on the axis is always equal to the latus rectum.
9. A double ordinate of the curve y2 = 4px is of length
8p; prove that the lines from the vertex to its two ends Let P(x1, y1) be a point in the plane of the parabola
are at right angles. y2 = 4ax. Let PN be the ordinate of P meeting the curve in
10. Prove that the locus of the circumcentre of the variable
triangle having sides y-axis, y = 2 and lx + my = 1,  
Q (figure). Then N  (x1 ,0),Q  x1 , 4ax1 and P(x1, y1)
where (l, m) lies on the parabola y2 = 4ax, is also a
parabola.  NP  y1 and NQ  4ax1
11. A variable parabola is drawn to pass through A and
B, the ends of a diameter of a given circle with centre
at the origin and radius c and have directrix as tangent P(x1,y1)
to a concentric circle of radius ‘a’ (a > c); the axes
being AB & a perpendicular diameter, prove that the Y
locus of the focus of the parabola is the standard Q
x2 y2
ellipse 2  2  1 where b 2  a 2  c 2
a b
12. Find the area of triangle formed by lines joining vertex O S N(x1,0)
of x2 = – 36y to ends of latus rectum.
13. Find the equation of circle, described on line segment
joining foci of x2 = 4ay and y2 = 4a(x–a) as diameter.
14. If on a given base, a triangle is described such that
(i) The point P(x1, y1) lies outside the parabola i.e., P is an
sum of tangents base angles is constant. Prove that
external point of the parabola if
locus of third vertex is parabola
NP  NQ  NP 2  NQ 2
Answer Key
5 i.e, if y 12  4ax 1  y 12  4ax 1  0  S11  0
1.8.4 m 3. 16 3 4.(5, 0) 5.
2
(ii) The point (x 1, y 1 ) lies on the parabola if
12
6.Parabola 7. 12.162 NP  NQ  NP 2  NQ 2
19
2 2
13. x + y – 2ax – ay = 0
Solutions are on Page No. 3.104 i.e., if y 12  4ax 1  y 12  4ax 1  0  S11  0

(iii) The point P(x1, y1) lies inside the parabola i.e., P is an

3.21
Coordinate Geometry

internal point of the parabola if NP  NQ  NP 2  NQ 2 On subtracting (2) from (1) we get y12  y 22  4a(x1  x 2 )

i.e., if y 12  4ax 1 y1  y 2 4a
  ...(3)
x 1  x 2 y1  y 2
 y12  4ax 1  0  S11  0

Thus P(x1, y1) is a point in the plane of the parabola S = 0 y1  y 2


Slope of chord AB  ...(4)
then x1  x 2
(i) P lies outside the parabola  S11>0
By using equation (3) & (4), equation of chord AB is
(ii) P lies on the parabola  S11 =0 4a
y  y1  (x  x 1 )
(iii) P lies inside the parabola  S11 < 0 y1  y 2

Position of line w.r.t Parabola  yy1  y12  yy 2  y 1 y 2  4ax  4ax 1


Here we have to determine that wheather the given line
intersects the parabola, touches the parabola or neither  (yy1  2ax)  (yy 2  2ax)  y12  4ax1  y1 y 2 ...(5)
intersects nor touches the parabola.
On adding –2ax1 and –2ax2 on both sides of equation (5)
Let the parabola be y2 = 4ax ...(1) we get
and given line y = mx + c ...(2) (yy1  2ax  2ax1 )  (yy 2  2ax  2ax 2 )
...(6)
On substituting y from equation (2) in equation (1) we get  (y12  4ax1 )  (y1 y 2  2ax 1  2ax 2 )
(mx + c)2 = 4ax
Now equation (6) becomes S1  S 2  S12 which is required
 m2 x 2  2x(mc  2a)  c 2  0 ...(3)
equation of chord AB.
This equation is quadratic in x, gives two values of x, shows
that every straight line cuts the parabola at two points which y (at22,2at2)
may be real and distinct, real and co-incident or imaginary, (x2,y 2)
which can be determined by discriminant (say D) of B
equation (3) (at21,2at1)
(x 1,y 1)
D  4(mc  2a)2  4m2 c 2 A
x
2 2 2 2 2
O
 4  m c  4a  4amc  m c 
y2=4ax
D  16a(a  mc) ...(4)
As ‘a’ is always positive, so it depends upon the value of
a–mc that whether the line intersect or touches the circle. 2) Parametric form
i) If a – mc > 0 then the line intersects the parabola at two
Let A(at 12 , 2at1 ) and B(at 22 , 2at 2 ) are extremeties of chord
real and distinct points i.e. line will act as a chord.
w.r.t parabola y2 = 4ax.
ii) If a – mc = 0 then the line touches the parabola at one
point i.e. line will act as a tangent. 2at1  2at 2
Slope of chord AB 
iii) If a – mc < 0 then line passess outside the parabola. at12  at 22

Equation of chord whose extremities are


2
known Slope of chord AB = t  t ...(1)
1 2
2
1) Cartesian form : Let the parabola be y = 4ax, and
A(x1, y1) , B(x2, y2) are extremities of chord. Then 2
Equation of chord AB is y  2at1  (x  at12 )
S11  y12  4ax 1  0 ...(1) t1  t 2

S 22  y 22  4ax 2  0 ...(2)  y(t1  t 2 )  2at12  2at1 t 2  2x  2at12

3.22
Parabola

 y(t1  t 2 )  2at1 t 2  2x the chord in parametric form.


Relation between parametric co-ordinates of ends of
 2x  y(t1  t 2 )  2at1 t 2  0 focal chord
is required equation of chord AB. Let y2 = 4ax be a parabola, if PQ be a focal chord.

note P  (at12 , 2at1 ) and Q  (at 22 , 2at 2 )


Parametric form of chord whose extremities are A(t1)
Since PQ passes through the focus S(a, 0) Q, S, P are
and B(t2) w.r.t parabola x2 = 4ay is
collinear
2y – x(t1 + t2) + 2at1t2 = 0
Slope of PS = Slope of QS
Length of chord of parabola y2 = 4ax intercepted on
y = mx + c: 2at1  0 0  2at 2
 
Let A(x1, y1) and B(x2, y2) are extremities of chord. So at12  a a  at 22
y1  mx1  c ...(1)
2t1 2t
 2
 2 2
y 2  mx 2  c ...(2) t  1 t2  1
1

 y1  y 2  m(x1  x 2 ) ...(3)
y
On substituting y from chord in parabola we get P(at2 1,2at 1)
B
(mx  c)2  4ax  0

m2 x 2  2x(mc  2a)  c 2  0 ...(4) x


A S(a,0)
Roots of above equation are x 1 and x 2. Therefore
Q (a
t 22,2
2(mc  2a) c2 at2 )
x1  x 2  2
and x1 x 2  2
m m

From above (x 1  x 2 )2  (x1  x 1 )2  4(x 1 x 2 )  t1 (t 22  1)  t 2 (t12  1)

 t1 t 2 (t 2  t1 )  (t 2  t1 )  0
4(mc  2a)2 4c 2 2
 (x 1  x 2 )   2  2 4a 2  4amc ...(5)
m4 m m t 2  t1  0  t1t 2  1  0

Length of chord AB  (x 2  x1 )2  (y 2  y1 )2 1
t1 t 2  1 or t 2  
t1
| AB | (x 2  x1 )2  m 2 (x1  x 2 )2
which is the required relation.

| AB |  | x 2  x 1 | 1  m 2 note
1) From above we can say that if one extremity of
4 focal chord is (at2, 2at) then its other extremity is
 | AB |  (a 2  mac)(1  m 2 ) is required length of
m2  a 2a 
chord.  2, .
t t 
Length of chord in parametric form
2) Above result is valid for all four standard parabolas.
2 2
Let A(at , 2at1 ) and B(at , 2at 2 ) are extremities of chord
1 2 Corollary
AB. Then | AB |  a 2 (t12  t 22 )  4a 2 (t1  t 2 )2 If chord AB and CD where A(t1), B(t2), C(t3) and D(t4)
intersect at (c, 0) on the axis of parabola then
t1t2 = t3t4 = –c/a
| AB | a| t1  t 2 | (t1  t 2 )2  4 is the required length of

3.23
Coordinate Geometry

Proof 4a. Hence, the latus rectum of a parabola is the smallest


focal chord.
Equation of chord AB is 2x  y(t1  t 2 )  2at1 t 2  0 ...(1)
Corollary
As it passess through (c, 0), so equation (1) satisfies by
Length of focal chord of parabola varies inversely as the
(c, 0)
square of its distance from vertex.
2c  0(t 1  t 2 )  2at1 t 2  0 Proof

 t1 t 2   c / a ...(2) Let P(at2, 2at) be one end of a focal chord of the parabola
y2 = 4ax. The focus of this parabola is S(a, 0).
Similarly, for chord CD
Equation of focal chord is (i. e, equation of PS)
t3t4 = –c/a
2at  0
Therefore, from equation (2) and (3), we have y0  (x  a)
at 2  a
t1t2 = t3t4 = –c/a
Length of focal chord in parametric form 2t
y 2
(x  a)
(t  1)
Let P(at2, 2at) is one end of focal chord PQ of parabola
 a 2a   (t 2  1)y  2tx  2at
y2 = 4ax, then its other end is Q  2 ,   .
t t 
 2tx  (t 2  1)y  2at  0
If d be the distance of this focal chord from the vertex (0, 0)
P of the parabola y2 = 4ax, then
M

Y
A S(a,0) P
M
x+a=0
x+a=0

N Q

Z A d S(a,0) X

 Length of focal chord = PQ N Q


= SP + SQ ( SP = PM and SQ = QN)
= PM + QN

a |0  0  2at | 2at 2a
 at 2  a  a d  
t2 2 2 2 2
(t  1)  1 
(2t)  (t  1)
t  
2
 t
 1  1
 a  t 2  2  2   a  t  
 t   t
4a 2
d2  2
Remark  1 ...(1)
t  
 t
1
 t  2 for all t  0 ( AM  GM )
t
 a 2a 
The other end of the focal chord is Q  2 ,  
2 t t 
 1
 a  t    4a
 t If length of focal chord = PQ = (l say)

 Length of focal chord  latus rectum  1


2

l=a  t+ 
i.e., The length of smallest focal chord of the parabola is  t

3.24
Parabola

2
l  1
= t+  ...(2) Y
a  t
P
M L

x+a=0
4a 2 4a 3
From (1) and (2), d 2 =  l1
(l / a) l
X
Z A l2 S(a,0)
4a 2 1 N
l or l  2 Q L
d2 d
i.e., the length of the focal chord varies inversely as the
square of its distance from vertex.
 Length of latus rectum LL '  4a
Length of focal chord making an angle  with axis
of parabola 1
 Semi latus rectum  (4a)  2a
2
Let P(at12 , 2at1 ) and Q(at 22 , 2at 2 ) be the extremities of focal
chord of parabola y2 = 4ax, which makes angle  with If sections of focal chord are l1 and l2
axis of parabola (i.e. x – axis). Then l1 = SP = PM = a + at2 = a(1 + t2)

PQ  a(t 2  t1 )2 ...(1) a a(1+ t 2 )


l2 = SQ = QN = a + =
t2 t2
 a((t 2  t1 )2  4t1 t 2 )
2l1l 2
 a((t 2  t 1 )2  4) ( t1t2= –1) Harmonic mean of l1 and l2 
l1  l2

2at 2  2at1
tan   slope of PQ  2 2 2
at 22  at12     2a
1 1 t 2
1 1
   
l 2 l1 a(1  t 2 ) a(1  t 2 ) a
2
tan    t 2  t1  2cot  ...(2)
t 2  t1 = Semi latus rectum.
Substituting the value of t2 + t1 from (2) in (1) then note
PQ = a(4 cot2  + 4) = 4a cosec2 . The above result is valid for all parabolas, ellipses,
note and hyperbolas.

For the parabola x2 = 4ay, the length of focal chord Tangent


which makes angle  with axis of parabola is 4a sec 2  Tangent at any point on the curve is limiting case of secant
passing through that point.
Theorem
Let S = 0 be a parabola and P is any point on the parabola.
If l1 and l2 are length of segements of focal chord of
Let Q is any other point on the parabola. If secant line
parabola, then harmonic mean of l1 and l2 is semi latus 
PQ is drawn to the parabola then as Q  P or |PQ|  O
2l1l 2
rectum. i.e. 2a  l  l then secant PQ is converted into tangent line at point P.
1 2
where P is known as point of contact of tangent.
Proof
Let parabola be y2 = 4ax
P(x1,y1)
2
If PQ be the focal chord then if P  (at , 2at) then X

 a 2a  Q(x2,y2)
Q 2, 
t t 

3.25
Coordinate Geometry

Point form of tangent is required equation of parametric form of tangent.


This form of tangent is applicable only if given point is on Slope form of Tangent
the surface of parabola.
Let the parabola be y2 = 4ax ...(1)
Method 1
and given line be y = mx+c ...(2)
Let the equation of parabola be y2 = 4ax ...(1)
given line is tangent to parabola, if there is only one solution
Let P(x1, y1) and Q(x2, y2) be any two points on the surface of line and parabola.
of parabola. Then equation of secant PQ is
On putting y from equation (2) to equation (1), we get
S1 + S2 = S12 ....(2) (mx + c)2 = 4ax
Now, for the existance of tangent at P, the distance  m2 x 2  2x(mc  2a)  c 2  0 ...(3)
PQ  O or Q  P. Hence S12  S11 and S2  S1
Given line is tangent, if equation (3) has only one root, i.e.
Now, put these limiting values in equation (2), we get
discriminant of above equation is zero.
S1 + S1 = S11
S1 = 0 (As S11 = 0) D  4(mc  2a)2  4m2 c 2  0
is required equation of point form of tangent .  4(4a 2  4amc)  0
Method 2
 a  mc
The equation of tangent ax (x 1, y1) to the parabola
S = y2 – 4ax = 0 is S1 = 0. a
Hence, the given line is y  mx  is required equation
Proof m
of slope form of tangent.
Let P(x1, y1) be a point on the parabola y2 = 4ax ...(1)
Point of contact of tangent in slope form
dy 2a It is the point where tangent touches the parabola.
Differentiating (1) w.r.t x we get 
dx y
Let P(x1, y1) is a point where slope form of tangent i.e.
2a a
The slope of the tangent at P(x1, y1)  y (y  0) y  mx  ...(1)
1
m

Now, the equation of the tangent at P(x1, y1) touches the parabola y 2  4ax ...(2)

2a Point form of tangent at P(x1, y1) is


i.e. y  y1  (x  x 1 )
y1 yy1  2ax  2ax1 ...(3)

i.e. yy1  y 12  2ax  2ax 1 Therefore, equation (1) and (3) are identical, so on
y1 2a 2ax 1
 yy1  2a(x  x1 )  0  S1  0 comparing   gives
1 m a
Parametric form of Tangent m

Let y2 = 4ax be the parabola and P(x1, y1) be any point on


parabola at which equation of tangent has to be determined.  a 2a 
(x1 , y1 )   2 ,  is required point of contact tangent.
m m 
As (at2, 2at) are parametric co-ordinates of parabola
y2 = 4ax, so (x 1 , y1 )  (at 2 , 2at) note
In the same way we can determine the equation of
Point form of tangent to parabola at P(x 1, y 1) is
tangents for other parabolas.
yy1  2a(x  x1 ) replace x1 by at2 and y1 by 2at in above
Corollary
equation. We have y  2at  2a(x  at 2 )
(1) If m = 0 and c  0 in slope form of tangent then equation
 ty  x  at 2 y = mx + c reduces to y = c, which intersect the parabola

3.26
Parabola

Method to determine equation of tangent from


 c2  external point
at only one point whose co-ordinates is  4a ,c  , Hence
 
Let the parabola be y2 = 4ax and from any external point
we can say that line parallel to axis of parabola can’t be its P(x1, y1) two tangents PA and PB are drawn to parabola.
tangent.
Let, us assume equation of tangent be
(2) If c = 0 and m  0 is slope form of tangent then its
equation becomes y = mx, which intersect the parabola at a
y  mx  ...(1)
m
 4a 4a 
(0, 0) and  2 ,  and so any line through origin can’t
m m  a
As it passess through point P, so it satisfy it y 1  mx 1 
be tangent for parabola y2 = 4ax m
(3) If m = c = 0 is slope form of tangent then its equation
m 2 x1  my1  a  0 ...(2)
reduces to y = 0 (i.e x axis) which can’t be its tangents as it
is axis of parabola. resulting equation is quadratic in m, from where we get two
Point of intersection of two tangents in parametric values of m which are slopes of PA and PB. Now by using
form slope point form we can determine equation of tangents.

Let the parabola be y2 = 4ax two points on the parabola Equation of tangents and their related terms for
are different parabolas
i) y2 = 4ax
Y a) Point form of tangent : yy1 = 2a(x + x1)
t)
2
t , 2a 2 b) Parametric co-ordinates : (at2, 2at)
P(a
1

c) Parametric form of tangent : ty = x + at2


X’ X
O a
R d) Slope form of tangent : y = mx +
Q{at1t2, a(t1+t2)} Q (a t 2 m
2 , 2 at )
2

Y’  a 2a 
e) Point of contact in slope form :  2 , 
m m 
P  (at12 , 2at1 ) and Q  (at 22 , 2at 2 )
a
f) Condition of tangency in slope form : c 
Equation of tangents at P(at12 , 2at1 ) and Q(at 22 , 2at 2 ) m
ii) y2 = - 4ax
2
t1 y  x  at 1
...(1)
a) Point form of tangent : yy1 = – 2a(x + x1)
and t 2 y  x  at 22 ...(2) b) Parametric co-ordinates : (– at2, 2at)
c) Parametric form of tangent : ty = –x + at2
Solving these equations we get x  at1 t 2 , y  a(t1  t 2 )

Thus, the co-ordinates of the point of intersection of tangents a


d) Slope form of tangent : y = mx –
m
at (at12 , 2at1 ) and (at 22 , 2at 2 ) are (at1 t 2 ,a(t1  t 2 ))

note
e) Point of contact in slope form :
Hence, we can say that x and y co-ordinate of point of
intersection of tangents at P(t1) and Q(t2) to the
parabola y2 = 4ax is geometric mean of x-coordinate a
and arithmatic mean of y-coordinate respectively of f) Condition of tangency in slope form : c  
m
points P and Q.

3.27
Coordinate Geometry

iii) x2 = 4ay
a
f) Condition of tangency in slope form : c  mh  k 
a) Point form of tangent : xx1 = 2a(y + y1) m
b) Parametric co-ordinates : (2at , at2) vii) (x - h)2 = 4a(y - k)
c) Parametric form of tangent : tx = y + at2
a) Point form of tangent :  x  x1  x1  h   2a  y  y1 
d) Slope form of tangent : y = mx – am2
b) Parametric co-ordinates : (h+2at, k+at2)
e) Point of contact in slope form :  2am,am 
2
c) Parametric form of tangent : t(x-h) = (y-k)+ at2

f) Condition of tangency in slope form : c  am 2 d) Slope form of tangent : y = mx - mh + k - am2

iv) x2 = –4ay e) Point of contact in slope form :  h  2am, k  am 2 


a) Point form of tangent : xx1 = – 2a(y + y1)
f) Condition of tangency in slope form : c  mh  k  am 2
b) Parametric co-ordinates : (2at , – at2)
viii) (x - h)2 = - 4a(y - k)
c) Parametric form of tangent : tx = –y + at2
d) Slope form of tangent : y = mx + am2 a) Point form of tangent :  x  x1  x1  h   2a  y  y1 

e) Point of contact in slope form :  2am, am 2  b) Parametric co-ordinates : (h+2at, k- at2)
c) Parametric form of tangent : t(x-h) = - (y-k)+ at2
2
f) Condition of tangency in slope form : c  am
d) Slope form of tangent : y = mx - mh + k + am2
2
v) (y – k) = 4a(x – h)
e) Point of contact in slope form :  h  2am,k  am 2 
a) Point form of tangent :  y  y1  y  k   2a  x  x1 
f) Condition of tangency in slope form : c  mh  k  am 2
b) Parametric co-ordinates : (h + at2, k + 2at)
Solved Examples
c) Parametric form of tangent : t(y-k) =(x-h)+ at2
Example 34
Find condition such that the line lx + my + n = 0
a
d) Slope form of tangent : y = mx - mh + k + l   touches y2 = 4ax.
m
Solution
a 2a  Let the given line lx + my + n = 0 ...(1)

e) Point of contact in slope form :  h  2 , k   touch the parabola y2 = 4ax at P(x1, y1)
 m m
 Equation of the tangent at P to the parabola is
yy1  2a(x  x1 )  0 ...(2)
a
f) Condition of tangency in slope form : c  mh  k  (1) & (2) represent the same line
m
l m n
vi) (y - k)2 = -4a(x - h)  =
2a y1 2ax 1
a) Point form of tangent :  y  y1  y  k   2a  x  x1  n
 x1 =
l
b) Parametric co-ordinates : (h-at2, k+2at)
c) Parametric form of tangent : t(y-k) = - (x-h)+ at2 2am  n 2am 
y1 =  P = , 
l l l 
a But P lies on
d) Slope form of tangent : y = mx - mh + k -
m
4a 2 m2 n
y 2 = 4ax  2
 4a    ln = am 2
a 2a  l l

e) Point of contact in slope form :  h  2 ,k   Example 35
 m m
If the end point t1, t2 of a chord satisifies the relation
t1 t2 = k(constant) then prove that the chord always

3.28
Parabola

passes through a fixed point. Find the point ? described on LL ' and MM' as diameters passes
Solution through the vertex of the parabola.
Equation of chord joining (at12 , 2at1 ) and (at 22 , 2at 2 ) is Solution
Equation of LL ' 2x – (t1 – t2) y + 2at1t2 = 0
2 passes through (c, 0)
y  2at1  (x  at12 )
t1  t 2
M(t3) L(t1)
(t 1  t 2 )y  2at12  2at1 t 2  2x  2at 12

2 x
y (x  ak) (  t 1 t2 = k ) A O (C,0)
t1  t 2
 This line passes through a fixed point (–ak, 0) (T2)L’ M’(t4)
Example 36
A tangent to the parabola y2 = 8x makes an angle of c c
450 with the straight line y = 3x + 5. Find its equation  t1 t 2   ; similarly t 3 t 4  
a a
and its point of contact.
now circle with LL ' as diameter
Solution
3 1  x  at  x  at    y  2at  y  2at   0
2
1
2
2 1 2
Slope of required tangent’s are m 
1 3
x 2  y 2  a  t 12  t 22  x  2a  t1  t 2  y  c 2  4ac  0 ...(1)
1
m1 = – 2 and m2   as a t t 2 2 2
 c 2 & 4a 2 t 1t 2  4ac 
2 1 2

 Equation of tangent of slope m to the parabola similarly circle on MM' ad diameter


a x 2  y 2  a  t 23  t 24  x  2a  t 3  t 4  y  c 2  4ac  0 ...(2)
y2 = 4ax is y  mx 
m
radical axis of above two circles is
1  x  t 12  t 2 2  t 3 2  t 4 2   2y  t1  t 2  t 3  t 4   0
 tangent’s y   2x  1 at  ,  2 
2 
which always passes through the origin.
1 Example 39
y x  4 at (8, 8)
2
Prove that the line x + y =1 touches the parabola
Example 37 l m
Find the equation to the tangents to the parabola y = 4a(x + b) if m (l + b) + al2=0.
2 2

y2 = 9x which goes through the point (4, 10). Solution


Solution The given parabola is
Equation of tangent to parabola y 2 = 9x is y2 = 4a(x + b) ...(1)
9 Equation of tangent to the parabola is
y  mx 
4m a
y  M  x  b  ...(2)
Since it passes through (4, 10) M
9 x y
 10  4m   16m2 – 40m + 9 = 0 given line,  1 ...(3)
4m l m
1 9 touches the parabola. So, equation (1) and (2) are
m , identical. On comparing we get
4 4
a
x Mb 
 equation of tangent’s are y   9 and M 1 M
4 1
 1 
l m 1
9 On eliminating M we get m2 (l  b)  al 2  0
y x  1.
4 Example 40
Example 38 Find the equations of the straight lines touching both
LOL ' and MOM ' are two chords of parabola the curves x2 + y2 = 2a2 and y2 = 8ax.
y2 = 4ax with vertex, A passing through a point O on Solution
its axis. Prove that the radical axis of the circles The given curves are

3.29
Coordinate Geometry

x2 + y2 = 2a2 ...(1) 2
1 
or k 2  ah  m  
t  m
n tangen
Co mmo
k 2  ah(m1/ 2  m 1/ 2 )2

 Required locus is y 2  ax(m1/ 2  m 1/ 2 )2


Example 42
A quadrilateral is inscribed in a parabola y2 = 4ax
and three of its sides pass through fixed points on the
axis. Show that the fourth side also passes through
and y2 = 8 ...(2)
fixed point on the axis of the parabola.
Equation of tangent to parabola is
Solution
2a Let AB, BC, AD of quadrilateral passess through fixed
y  mx 
m points (c , 0), (d , 0) and (e , 0) respectively then

or m2 x  my  2a  0 ...(3) c d
Given t1t 2   .....(1) ; t 2 t 3   ...(2)
It is also tangent of (1), then the length of perpendicular a a
from centre of (1) i.e. on (3) must be equal to the
A(t1)
radius of (1) i.e. a 2 . B(t2)

|0  0  2a| (e,0)
a 2 (c,0)
x

(m 2 )2  ( m)2
C(t3)
4a 2
or 4  2a 2 D(t4)
m  m2
or (m2 + 2)(m2 – 1) = 0 e
2
t 1t 4   ...(3)
 m 1  0  m  1 a
Hence from (3) the required tangents are now from above equations
x  y  2a  0  t 2 t 3  t1t 4   de
Example 41 t 3t 4 
 t 1t 2  ac
Tangents to the parabola y2 = 4ax are drawn at points
whose abscissaes are in the ratio m2 : 1. Find locus of which is constant.
their point of intersection. Example 43
Solution For what real values of a, the point (–2a, a + 1) will
be an interior point of the smaller region bounded by
Let two points be P(at12 , 2at 1 ) and Q(at 22 , 2at 2 ) on the the circle x2 + y2 = 4 and the parabola y2 = 4x.
parabola y2 = 4ax Solution
The point P(–2a, a + 1) will be an interior point of
at12 m2
then,  both the circle x2 + y2 – 4 = 0 and the parabola
at 22 1 y2–4x = 0.
t1 = mt2 ...(1)  (2a)2  (a  1)2  4  0 ,
Point of intersection of tangents at P and Q is
{at1t2, a(t1 + t2)} i.e., 5a2 + 2a – 3 < 0 ...(i)
and (a + 1)2– 4(–2a) < 0,
Let h = at1t2 and k = a(t1 + t2) using (1),
i.e., a2 + 10a + 1 < 0 ...(ii)
2 h
h  amt 22 t  ...(2)
2
am y2=4x
k p
k  a(mt 2  t 2 )  t 2  ...(3)
a(m  1)
From equations (1) and (3), 2
x +y =4
2

k2 h (m  1)2
2 2
 or k 2  ah The required values of a will satisfy both (i) and (ii).
a (m  1) am m From (i), (5a – 3) (a + 1) < 0

3.30
Parabola

If M is the mid-point of PN, then


3
 by sign-scheme we get, 1  a  5 ...(ii) NM = (1/2)PN = at.
As QM is parallel to x-axis, its equation is y = at....(2)
Solving (ii), the corresponding equation is To solve (2) with (1), putting the value of y from (2) in
a2 + 10a + 1 = 0
2 2 1 2
10  100  4 (1), we get a t  4ax or x  at
or a   5  2 6 ...(iv) 4
2
The set of values of a satisfying (ii) and (iv) is 1 2 
So the co-ordinates of Q are  at , at 
4 
1  a  5  2 6 .
Equation of NQ will be
Example 44
Prove that two straight lines, one a tangent to the at  0 4
parabola y2 = 4a (x + 1) and the other to the parabola y0 (x  at 2 ) or y    x  at 2  ...(4)
1 2 2 3t
at  at
y 2  4a '(x  a ') , which are at right angles to one 4
another, meet on the straight line x  a  a '  0 , also If NQ meets the tangent at the vertex i.e. x = 0 at T,
show that this straight line is common chord of the 4at
two parabolas : then putting x = 0 in (3), we get y  .
3
Solution
Parabolas y2 = 4a(x + a) ...(i)  4a  4a
2 So the co-ordinates of T are   0, t  or AT  t
y  4a '(x  a ') ...(ii)  3  3
Equation to the tangents of (i) & (ii) are
2 2
y = m (x + a ) + a/m ...(iii) Clearly AT  .2at  .PN
3 3
and y  m '(x  a ')  a '/ m ' ...(iv)
Example 46
mm'  1 ...(v)
If perpendiculars be drawn on any tangent to a
Point of intersection of (iii) and (iv) parabola from two fixed points on the axis, which are
a a' equidistant from the focus, prove that the difference
(m  m ')x  am  m ' a '  0 of their squares is constant.
m m'
Solution
(am '  a ' m)
 (m  m ')x  am  m ' a '  0 Let the equation to the parabola be y2 = 4ax ...(1)
1 Its axis is x-axis and focus is (a, 0). So any point on
 (m  m ')x  a(m  m ')  a '(m  m ')  0 the axis equidistant from the focus may be taken as
(a + k, 0) and (a – k, 0) say P and Q respectively.
 x  a  a'  0 Equation to any tangent to (1) is
To get common chord subtract (ii) from (i),
a
We get x  a  a '  0 y  mx  ...(2)
Example 45 m
PN is an ordinate of the parabola, a straight line is m(a  k)  a / m
drawn parallel to the axis of parabola which bisect Distance of P from (2), say PN 
NP and meets the curve in Q; prove that NQ meets (1  m 2 )
the tangent at the vertex in a point T such that AT = and the distance of Q from (2) say
(2/3) NP.
m(a  k)  a / m
Solution QM 
Let the equation of the parabola be y2 = 4ax. ....(i) (1  m 2 )
Tangent at the vertex is y-axis or x = 0 ....(2) Hence
Let P be any point (at2, 2at) on the parabola. So
PN = 2at and co-ordinate of N are (at2, 0). [m(a  k)  a / m]2 [m(a  k)  a / m]2
PN 2  QM 2  
1  m2 1  m2
t)
(a t , 2a
2

T P 1 a2 a
Q  m 2 (a  k)2  2  2m  (a  k)  m 2 (a  k)2
x’
N
M
X 1  m 
2
m m
A
a2 a 
 2
 2  m(a  k)
m m 

3.31
Coordinate Geometry

Let AP = r1 and AQ = r2
1
[4m 2ak  4ak] If PAX  
(1  m 2 )
= 4ak which is constant as k and a are constant. then QAX  900  
Example 47  Co – ordinates of P and Q are (r1 cos , r1 sin )
Prove that on the axis of any parabola there is a certain
point K which has the property that if a chord PQ of and (r2 sin , r2 cos ) respectively..
1 1 Since P and Q lies on y2 = 4ax
the parabola be drawn through it, the  is
PK 2 QK 2  r12 sin 2   4ar1 cos  and r22 cos 2   4a r2 sin 
same for all positions of the chord. 4/3 4/3
Solution 4/3  4a cos  4a sin    16a 2 
 (r1r2 )  2
   
Let the point on the axis be (c , 0). So, equation of  sin  cos 2    sin  cos  
chord is and
xc y0  4a cos   2 / 3  4a sin   2 / 3 
 r
cos  sin  16a 2  (r12 / 3  r22 / 3 )  16a 2  2   2  
 sin    cos   
Hence general point on this chord is
x  r cos   c ; y  r sin ;  (cos )2 / 3 (sin )2 / 3 
 16a 2  (4a)2 / 3  4 /3
 
Which satify parabola, on substituting it in y2 = 4ax  (sin ) (cos )4 / 3 
r 2 sin 2   4a(r cos   c)  0
 cos 2   sin 2  
2 2  16a 2  (4a)2 / 3  4/3 
r sin   (4a cos )r  4ac  0 4/3
 (sin ) (cos ) 
4a cos  4ac 4 /3
r1  r2  : r1r2   2 16a 2  (4a)2 / 3  16a 2 
sin2  sin   4/3
  ...(2)
(sin  cos )  sin  cos  
now,
Hence from equation (1) and (2) we have
1 1 r12  r22 (r1  r2 )2  2r1r2
   (r1r2 )4 / 3  16a 2 (r12 / 3  r22 / 3 )
PK 2 QK 2 (r1r2 )2 (r1r2 )2
Example 49
 16a 2 cos 2  8ac  4 Let C1 and C2 be respectively the parabolas
   sin 
sin4  sin 2   x2 = y – 1 and y2 = x – 1. Let P be any point on C1

16a 2c 2 and Q be any point on C2. Let P1 and Q1 be reflections
of P and Q respectively with respect to the line y = x.
 16a 2 cos 2   8ac sin 2   Prove that P1 lies on C2, Q1 lies on C1 and PQ  min
  which is a constant if c
 16a 2c 2  {PP1, QQ1}. Hence or otherwise determine points P0
and Q0 on the parabolas C1 and C2 respectively such
1
= 2a and constant  that P0 Q 0  PQ for all pairs of points (P, Q) with P
4a 2
on C1 and Q on C2.
Example 48
Solution
If r1, r2 be the length of the perpendicular chords of
Let co-ordinates of P and Q are P(t, t2 + 1) and Q(s2
the parabola y2 = 4ax, drawn through the vertex, then
+ 1, s) which lie on x2 = y – 1 and y2 = x – 1
show that (r1r2 )4 / 3  16a 2 (r12 / 3  r22 / 3 ) respectively
Solution  P1 and Q1 be reflections of P and Q respectively
Since chords are perpendicular, therefore if one makes with respect to the line y = x then
an angle  with the axis then other will make an angle P1  (t 2  1, t) and Q1  (s, s 2  1)
(900–) with x – axis.
we have (PQ1 )2  (t  s)2  (t 2  s 2 )2  (P1Q)2
Y
P  PQ1  P1Q and also PP1 ||QQ1
r1
X’

X thus PP1QQ1 is an isosceles trapezium
A r2
0
90 -
Q we have PQ  min{PP1 ,QQ1}

Y’

3.32
Parabola

(2x  a)(2x  9a)  0


2
x =y-1
Y a 9a
y=x x  or x   (rejected)
2 2
P
Q1 Hence common chord bisects centre and focus.
(0,1) P1 Example 51
Q X
O A triangle ABC of area  is inscribed in the parabola
y2 = 4ax such that the vertex A lies at the vertex of
y2=y-1 the parabola and BC is a focal chord. Then find
differences of the distances B and C from the axis of
Let us take min {PP1, QQ1} = PP1 the parabola.
Solution
then (PQ)2  (PP1 )2
Let A(0, 0), B(at12 , 2at 1 ),C(at 22 , 2at 2 )
2 2 2 2 2 2
 (t  1  t)  (t  t  1)  2(t  t  1)  f(t)(say)
Since BC is a focal chord of the parabola  t1t 2  1 .
2
 1 3 
 f '(t)  4(t 2  t  1)(2t  1)  4  t     (12t  1) 1 1
2 4    (x1y 2  x 2 y l )  (at12  2at 2  at 22  2at1 )
 2 2
Now, f '(t)  0  t  1 / 2 1
  2a 2 t1t 2 (t1  t 2 )  a 2 (t1  t 2 ) .....(1)
1 2
Also f '  t   0 for t < 1/2 and f '(t)  0 for t  Distance of B and C from axis are 2at and 2at2
2
Hence f(t) is least when t = 1/2, respectively
 2
1 5  D  2a(t 2  t1 )  2a  2 [ using (1) ]
D
Point P0 on C1 is  ,  and P1 (which we take as Q0) a a
2 4
Example 52
5 1 The parabola y2 = kx makes an intercept of length 4
and C2 are  ,  . Note that P0 Q 0  PQ for all pairs on the line x – 2y = 1. Then find value of k.
 4 2
Solution
of (P,Q) with P on C1 and Q on C2. Solving y2 = kx, x – 2y = 1. we get
Example 50
A circle is described whose centre is the vertex and y2 = k(1 + 2y).  y 2  2yk  k  0
whose diameter is three quarters of the latus rectum  y1  y 2  2k and y1 y 2  k
of a parabola, prove that the common chord of the
Now, length of intercept = 4 units given
circle and parabola bisects the distance between the
vertex and the focus.  (x 1  x 2 )2  (y1  y 2 )2  4 2 ,
Solution 2
 y2 y2 
3   1  2   (y1  y 2 )2  16
Diameter of the circle  4a  3a  k k 
4

2 2 9a 2  (y  y )2 
 equation of the circle is x  y  4  (y1  y 2 )2  1 2 2  1  16
 k 
solving it with y2 = 4ax  4k 2 
 (y 1  y 2 )2  4y1 y 2   2  1  16
y  k 
a
2  (2k)2  4( k)  5  16,  20k 2  20k  16  0
x
A S
5  25  80
 5k 2  5k  4  0  k 
25

5  105
k 
2 9a 2 10
x  4ax  0
4 Example 53
4x 2  16ax  9a 2  0 Two perpendicular chords are drawn from the origin

3.33
Coordinate Geometry

O to the parabola y = x2 – x, which meet the parabola


of P and Q. Rectangle POQR is completed. Find the i.e. x 4  64ab 2 x i.e. x(x 3  64ab2 )  0
locus of vertes R. 1 2
Solution gives x  0, 4a 3 b 3
Let y = m1x and y = m2x be the equations of chords Putting the values of x in either of equations (1) or (2)
OP and OQ respectively such that OP  OQ . gives the coordinates of the intersection point of the
given parabolas as
y
1 2 2 1
Q (m2+1 O(0, 0) and P(4a 3 b 3 , 4a 3 b 3 )
m22+m2)
x
y=
m 2
Slope of a tangent to the parabola y2 = 4ax is
R(h,k)
x
dy 2a
O y=m1x y=x2-x 
dx y
P(m1+1,m12+m1)
and to the parabola x2 = 4by is given by
 m1m2  1 dy x

Solving y = m1x and y = x2 – x, we get dx 2b
P   m1  1, m12  m1  Hence, the slopes of the two tangents at the
intersection point P are
Similarly, Q   m 2  l, m 22  m 2  1 1
2a 3 a3
Let the co-ordinates of R be (h, k) m1  1
and m 2  1
 Diagonals of a rectangle bisect each other.. b 3
2b 3
h m1  m 2  2 k m 2  m 22  m1  m 2 The angle between the two tangents is given by
  and  1
2 2 2 2  m  m2 
  tan 1  1 
 1  m1m 2 
2 2
 m1  m 2  h  2 and m  m  m1  m 2  k
1 2

Putting the values of m1 and m2 gives


 m1  m 2  h  2 and
1 1
(m1  m 2 )2  2m1m 2  (m1  m 2 )  k
1 3a 3 b 3
  tan 2 2
 (h  2)2  2   h  2  k  k  h2  3h  4 2(a 3  b 3 )
Hence, the locus of (h, k) is y = x2 – 3x + 4. Example 55
Example 54 Find locus of centre of circle which cut y2 = 4ax
Find angle between the parabolas y2 = 4ax and x2 orthogonally and passess through (0, 0)
= 4ay Solution
Solution The figure shows a circle cutting the parabola
The equations of the given parabola are orthogonally at P(at2, 2at). Hence, the tangent to the
y2 = 4ax ...(1) parabola at P must pass through the centre of the
and x2 = 4by ...(2) circle C(h, k). Equation of the tangent to the parabola
Solving equations (1) and (2), we have at P is
Y x
y  at ...(1)
t

P
x
O 2
P(at ,2at)
C

O X

2
y =4ax

2
 x2 
   4ax h
 4b  Therefore, we have k   at
t

3.34
Parabola

i.e. at 2  kt  h  0 ...(2) y 12 y2
Also, since the circle passes through the origin O, Putting x 1  and x 2  2 in equation (3), it
4a 4a
therefore
CO = CP  y1  y 2 
reduces to h  k 0 ...(4)
2 2 2 2
i.e. h  k  (h  at )  (k  2at) 2
 4a 
i.e. at 3  2(2a  h)t  4k  0 ...(3) y1 y 2 2k
i.e.   0
equation (3) t  equation (2) gives 2a 2a h
According to the given condition, we have
at 3
 2(2a  h)t  4k  t at 2  kt  h  0
2a
slope of the tangent at P   tan 1
kt 2  4(a  3h)t  4k  0 ...(4) y1
Applying Cramer’s rule on quadratic equations (2) and
(4), we have 2a
slope of the tangent at P   tan  2
y2
t2 t 1
 
4k  h(4a  3h) hk  4ak a(4a  3h)  k 2
2
k
slope of the AR   tan 
Eliminating t from above, we have h
Putting the above results in equation (4), it reduces to
(hk  4ak)2  {4k 2  h(4a  3h)}{k 2  a(4a  3h)}
cot 1  cot 2  2 tan   0
i.e. 4k 4  2k 2h(h  12a)  ah(3h  4a)2  0
which is the desired result.
Putting (x, y) in place of (h, k) gives the equation of Example 57
the desired locus as If the tangent to the parabola y2 = 4ax meets the axis
4y 4  2y 2x(x  12a)  ax(3x  4a)2  0 in T and tangent at vertex A in Y and the rectangle
Example 56 TAYG is completed. Find locus of G
Through the vertex A of the parabola y2 = 4ax, two Solution
chords AP and AQ are drawn, and the circles on AP Let P(at 2 , at) be any point on the parabola y2 = 4ax.
and AQ as diameter intersect in R. Prove that if 1, 2
and  be the angles made with the axis by the tangents Then tangent at P(at 2 , at) is ty = x + at2
at P, Q and by AR, then Since tangent meets the axis of parabola in T and
cot 1  cot 2  2 tan   0 . tangent at the vertex in Y.

Solution P
Y
Let the coordinates of the points P, Q and R be G (a
(x1, y1), (x2, y2) and (h, k) respectively. X’ T X
A
Since AP is the diameter of a circle, therefore
AP2 = AR2 + PR2
i.e. x 12  y 12  (h  x 1 )2  (k  y1 )2  (h 2  k 2 )
 Co-ordinates of T and Y are (–at2, 0) and (0, at)
i.e. h 2  k 2  hx 1  ky1  0 respectively.
Let co-ordinates of G be (x1, y1)
Similarly, since AQ is a diameter, therefore
Since TAYG is rectangle.
AQ2 = AR2 + QR2
 Mid-points of diagonals TY and GA is same
i.e. x 22  y 22  (h  x 2 )2  (k  y 2 )2  (h 2  k 2 )
x 1  0 at 2  0
   x 1  at 2
i.e. h 2  k 2  hx 2  ky 2  0 ...(2) 2 2
subtracting equation (1) from equation (2), we have
y 1  0 0  at
and   y1  at
h(x1  x 2 )  k(y1  y 2 )  0 ...(3) 2 2
Eliminating t from (1) and (2) then we get
P
2
y 
R x 1  a  1  or y12  ax 1  0
A
a
 The locus of G(x1, y1) is y2 + ax = 0
Q

3.35
Coordinate Geometry

Make Concepts Clear 3.3 If t varies, prove also that the locus of its centre is the
curve 27a2 = (2x – a)(x – 5a)2
1. Show that the two parabolas
12. The tangent at any point P of the parabola y2 = 4ax
x2 + 4a(y – 2b – a) = 0 and y2 = 4ab(x – 2a + b)
is met in Q by a line through the vertex A, drawn at
intersect at right angles at the common end of the
right angles to AP; and R is the foot of the
latus rectum of each.
perpendicular from A to the tangent at P. Show that
2. Find common tangent to y2 = 4x and x2 = – 32y.
there are three positions of the point P for which R lies
3. A pair of tangents are drawn which are equally inclined
on the line lx + my + na = 0 and the corresponding
to a straight line y = mx + c whose inclination to the
point Q lies on the line (2l – n)x + 4my + 2na = 0.
axis is . prove that the locus of their point of
13. Find the centre and radius of the smaller of the two
intersection is the straight line
circles that touch the parabola 75y2 = 64(5x – 3) at
y = (x – a) tan 2.
(6/5, 8/5) and the x-axis.
4. The focal chord of y2 = 16x is tangent to (x – 6)2+ y2
14. Let P be any point on parabola y2 = 4ax between its
= 2, then find possible values of the slope of this
vertex and extremity of latus rectum (with positive y
chord.
co-ordinate). M is the foot of perpendicular from focus
5. The two parabolas y2 = 4a(x – l) and x 2  4a(y  l ') S on tangent at P. Find maximum value of area of
always touch one another, the quantities l and l ' both triangle PSM.
being variables; prove that the locus of their point of 15. If a, b and c are fixed real numbers and l and m are
contact is the curve xy = 4a2. variable real numbers satisfying aml + cl2 – bm2 + l
6. Tangents drawn from P to the parabola y2 = 4ax are = 0, then prove that the variable staright line lx + my
inclined at angle 1, 2 to the axis of the parabola + 1 = 0 always touches a fixed parabola, whose
(angles being measured in the anti-clockwise sense), axis is parallel to the x-axis.
16. The inclinations  and  of two tangents to the parabola
such that tan 1  tan  2  b (constant)
Find the locus of P. 1
y2 = 4ax with the axis are given by tan   and
7. Let PQ be a variable focal chord of the parabola m
y2 = 4ax whose vertex is A. Prove that the locus of
m
the centroid of APQ is a parabola whose latusrectum tan   . Show that, as m varies, the point of
2
4a intersection of these tangents traces a line parallel to
is . the directrix of the parabola.
3
8. If from the vertex of the parabola y2 = 4ax a pair of 17. Show that the locus of the point of intersection of the
points to y2 = 4ax which intercept a constant length d
chords be drawn perpendicular to each other and with
on the directrix is
these chords as adjacent sides a rectangle is
(y2 – 4ax) (x + a)2 = d2 x2.
completed, then prove that the locus of the vertex of
the farther vertex of the rectangle is the parabola y2 =
4a(x – 8a). Answer Key
9. The tangent at point P to y2 – 2y – 4x + 5 = 0 intersects 2. x – 2y + 4 = 0 4. 1, – 1 6. y = bx.
the directrix at Q. Find locus of a point R such that it 9.2(x – 2)(y – 1)2 = (3x – 4)2
divides PQ externally in ratio 1 : 2 13. 1, (2, 1) 14.a2
10. Through a point P tangents PQ and PR are drawn to Solutions are on Page No. 3.106
a parabola and circles are drawn through the focus to
touch the parabola in Q and R respectively. Prove
that the common chord of these circles passes through 3.4 Normal of Parabola, Co-
the centroid of the triangle PQR.
11. Prove that the equation to the circle, which passes normal points, Envelope of Family
through the focus and touches the parabola of Curve
y2 = 4ax at the point (at2, 2at) is
Normal
x 2  y 2  ax(3t 2  1)  ay(3t  t 3 )  3a 2 t 2  0
If S = 0 is a parabola and P is any point on the surface of
P(t)
parabola, then line passing through P and perpendicular to
the tangent to parabola at point P is called normal to
S parabola at point P.
(OR) line perpendicular to tangent at point of contact is
known as its normal.

3.36
Parabola

Different forms of normal. Now, equation of normal at P(x1, y1) in point form is
Point form of normal y1
y  y1   (x  x1 ) ...(1)
2a
This form of normal is applicable only if given point is on
surface of parabola. Now, on replacing x1 by at2 and y1 by 2at in equation (1) we
get its parametric form.
Let the equation of parabola be y2 = 4ax and the point be
P(x1, y1) 2at
y  2at 
2a
 x  at 2 
Equation of tangent at P is yy1  2a(x  x1 ) ...(1)

2a y  tx  2at  at 3 is required form of parametric form of


Slope of tangent at P  normal.
y1
Slope form of normal
 y1
Slope of normal at P  The equation of normal to the parabola y2 = 4ax at (x1, y1)
2a
y1
y is y  y1   (x  x1 )
2a
 Equation of normal at point P is y  y1  (x  x 1 )
2a
let m is the slope of the normal then
Corollary
y1
The equation of normal at (x1, y1) can be obtained by this m i.e. y1  2am
2a
method
(x1, y1) lies on y2 = 4ax therefore
x  x1 y  y1
 ...(1)
a ' x 1  hy1  g hx 1  by1  f y 12  4ax 1  4a 2 m 2  4ax 1  x1  am 2

a’, b, g, f, h are obtained by comparing the given parabola According the values of x1 and y1 in (1) we get
2 2
with a ' x  2hxy  by  2gx  2fy  c  0 ...(2) y + 2am = m (x – am2)
and denominators of (1) can be easily found by the first y = mx – 2am – am3 ...(2)
two rows of this determinant 3
y = mx – 2am – am is a normal to the parabola
y2 = 4ax where m is slope of the normal. The co-ordinates
a' h g of the point where normal of slope m intersects the parabola
h b f is (am2,–2am) , and c  2am  am 3 is the condition of
i.e.
g f c normality when y = mx + c the normal to the parabola
y2 = 4ax.
Since first row a '(x1 )  h(y1 )  g(1) and second row,, Point of intersection of normals in parametric form
h(x1 )  b(y1 )  f(1) Let the points on the parabola y2 = 4ax are
Here, parabola y2 = 4ax 2
P  (at12 , 2at1 ) and Q  (at 2 , 2at 2 )
or y2 – 4ax = 0 ...(3)
Comparing (2) and (3) then we get 2 2a
t 1)
y
(at1,
P
a '  0, b  1,g  2a, h  0, f  0
x’ 2
From (1), equation of normal of (3) is A x

x  x1 y  y1 y1 Q(
at
 or y  y1   (x  x1 ) 2
2
,2 a
0  0  2a 0  y 1  0 2a y’
t2 )

Parametric form of normal :


Let the point where parametric form of normal has to be Equations of normals at P(at12 , 2at1 ) and Q(at 22 , 2at 2 ) are

find be P(at 2 , 2at) . y   t 1x  2at1  at13 ...(1)

3.37
Coordinate Geometry

and y   t 2 x  2at 2  at 23 ...(2) a


5) Condition of normality : c  2a 
m2
Solving (1) and (2), we get x  2a  a(t12  t 22  t1 t 2 ) ,
6) Point of intersection of normals :
y  at1 t 2 (t1  t 2 ) (at1 t 2 (t1  t 2 ), 2a  a(t12  t 22  t1 t 2 ))
If R is the point of intersection then iv) x2 = – 4ay

R   2a  a(t12  t 22  t 1 t 2 ), at1 t 2 (t1  t 2 ) 2a


1) Point form of normal : y  y1  x (x  x 1 )
1
Equation of normals and their related terms for
different parabolas
2) Parametric form of normal : x  ty  2at  at 3
2
i) y = 4ax
a
 y1 3) Slope form of normal : y  mx  2a 
1) Point form of normal : y  y 1  (x  x 1 ) m2
2a
2a a
2) Parametric form of normal : y  tx  2at  at 3 4) Point of intersection : ( , )
m m2
3) Slope form of normal : y  mx  2am  am3
a
5) Condition of normality : c  2a 
4) Point of intersection : (am , 2am) 2 m2
6) Point of intersection of normals :
5) Condition of normality : c  2am  am 3
(at1 t 2 (t1  t 2 ), 2a  a(t12  t 22  t1 t 2 ))
6) Point of intersection of normals :
(2a  a(t12  t 22  t1 t 2 ), at1 t 2 (t1  t 2 ))
v) (y – k)2 = 4a(x – h)

ii) y2 = - 4ax   y1  k 
1) Point form of normal : y  y 1  (x  x 1 )
2a
y1
1) Point form of normal : y  y1  (x  x 1 ) 2) Parametric form of normal :
2a

2) Parametric form of normal : y  tx  2at  at 3  y  k   t  x  h  2at  at 3


3) Slope form of normal :
3) Slope form of normal : y  mx  2am  am3
 y  k   m  x  h   2am  am3
4) Point of intersection : (am2 , 2am)
4) Point of intersection : (h  am2 ,k  2am)
5) Condition of normality : c  2am  am 3
6) Point of intersection of normals : 5) Condition of normality : c  k  mh  2am  am 3
(2a  a(t12  t 22  t1 t 2 ),at1 t 2 (t1  t 2 )) vi) (y – k)2 = – 4a(x – h)
iii) x2 = 4ay  y1  k 
1) Point form of normal : y  y 1  (x  x 1 )
2a
2a
1) Point form of normal : y  y1   (x  x1 ) 2) Parametric form of normal :
x1

2) Parametric form of normal : x  ty  2at  at 3  y  k   t  x  h   2at  at 3


3) Slope form of normal :
a
3) Slope form of normal : y  mx  2a 
m2  y  k   m  x  h   2am  am 3
2a a 4) Point of intersection : (h  am 2 , k  2am)
4) Point of intersection : ( , )
m m2

3.38
Parabola

5) Condition of normality : c  k  mh  2am  am 3 E (y, x, a) = 0 ...(3)


2
vii) (x – h)2 = 4a(y – k) and for the parabola x = – 4ay will be
E (y, x,–a) = 0 ...(4)
2a
1) Point form of normal : y  y 1   x  h  (x  x 1 ) If the coordinates of the vertex be (), then substitute
1
(x – ) and (y – ) for x and y respectively.
2) Parametric form of normal : For example the equation of a general tangent to the
 x  h  t  y  k   2at  at 3 a
parabola y2 = –4ax is y  mx  then to the parabola
m
a (y – )2 = 4a(x –) is
3) Slope form of normal :  y  k   m  x  h   2a 
m2
a
(y  )  m(x  )  ...(6)
2a a m
4) Point of intersection : (h  ,k  2 )
m m and to the parabola x2 = 4ay is

a a 1
5) Condition of normality : c  k  mh  2a  x  my  (where is slope of tangent) ...(7)
m2 m m
viii) (x – h)2 = – 4a(y – k) Similarly, equation of a general normal to the parabola
y2 = – 4ax is
2a
1) Point form of normal : y  y 1   x  h  (x  x 1 ) y = mx + 2am + am3 ...(8)
1
2
to the parabola x = 4ay is
2) Parametric form of normal :
1
 x  h  t  y  k   2at  at 3 x = my – 2am – am3 (where is slope of normal) ...(9)
m
3) Slope form of normal :
and so on.
a Normal chord
 y  k   m  x  h   2a 
m2 Chord which is normal at one point of intersection is said
to be normal chord of parabola.
2a a
4) Point of intersection : (h  ,k  2 ) If normal at one point P intersects the parabola again at Q
m m
then chord PQ is known as its normal chord.
a Relation between ‘t1’ and ‘t2’ if normal at (t1) intersect
5) Condition of normality : c  k  mh  2a 
m2 parabola again at t2
Rule for transforming an Equation for the various Let the parabola be y2 = 4ax, equation of normal at
forms of the parabola P(at12 , 2at1 ) is y   t1 x  2at1  at13 ...(1)
In all the previous derivations on the parabola, all the related
propositions have been proved and derived for the particular
t 1)
parabola y2 = 4ax. y 2
, 2a
at 1
P(
However, all the results with slight transformations are valid
for any parabola. Here we learn those transformations.
x’ x
If any equation derived for the parabola y2 = 4ax, (a > 0) A
is given by
E(x, y, a) = 0 ...(1)
y’ Q(a
then the same equation t 2
2
, 2at
2 )
for the parabola y2 = – 4ax will be E(x, y, –a) = 0 ...(2)
for the parabola x2 = 4ay will be Since it meet the parabola again at Q(at 22 , 2at 2 ) then

3.39
Coordinate Geometry

Proof
equation (1) passes through Q(at 22 , 2at 2 )
PR is normal to parabola y2 = 4ax
 2at 2   at1 t 22  2at 1  at13
2
 t 3   t1  ...(i)
2
 2a(t 2  t1 )  at1 (t  t )  0 2 t1
2 1

 a(t 2  t1 )[2  t1 (t 2  t1 )]  0
y

 a(t 2  t1 )  0 (As t1 and t2 are different)


P y2 =4ax
So, 2 + t1(t2 + t1) = 0 Q

2 G
t 2   t1 
t1
R
above results is true t  R  {0}
Hence, we can say that normal at vertex would never
intersect the parabola again. Also QR is normal to parabola y2 = 4ax
note
2
As in parabola two tangents can’t be parallel, therefore  t 3  t 2  ...(ii)
t2
any of the normal chords can’t be normal at both
point of intersection of chord.
2 2
Corollary 1 from (i) and (ii) we have t1   t 2 
t1 t2
Length of normal chord
Let equation of parabola be y2 = 4ax on which normal at 2 2 t t 
 t1  t 2    (t1  t 2 )  2  1 2 
P(t1) intersect parabola again at Q(t2). Then t 2 t1  t1 t 2 

|PQ|  a 2 (t12  t 22 )2  4a 2 (t1  t 2 )2  t 1 t 2  2 (t1  t 2 )


Also from (i)
 a | t1  t 2 | (t 1  t 2 )2  4
2 t 
2
t 3   t1    t1  2  2   (t  t )
 2  2 t1 2 1 2

 a t1    t1    t1  t1    4
 t 1   t 1 
 t 3  (t1  t 2 )

2 Corollary 3
(by using t 2  t1  )
t1 If normal at P(t1) and Q(t2) to the parabola y2 = 4ax
intersect at a point which lies on parabola, then line joining
P and Q passess through a fixed point whose co-ordinates
1 1
 4a t1  1 are (–2a, 0)
t1 t12
Proof
4a(t12  1)3 / 2 Equation of line passing through P(at12 , 2at1 ) and
 |PQ| 
t12
Q(at 22 , 2at 2 ) is
Corollary 2
2a(t 2  t1 )
If normal to parabola y2 = 4ax at P(t1) and Q(t2) intersect y  2at1  (x  at12 ) ...(1)
at a point R(t3) which lies on parabola then a(t 22  t12 )

i) t1t2 = 2 ii) t1 + t2 + t3 = 0  (y  2at1 )(t1  t 2 )  2(x  at12 )

3.40
Parabola

 y(t1  t 2 )  2x  2at1 t 2 ...(2)  2


 t   t    4  (t 2  2)  4  t 2  2
As t1t2 = 2 (from previous result) so equation (2) becomes  t
2x  y(t1  t 2 )  4a  0 Corollary 2
which can be written as Normal chord to y2 = 4ax at a point whose ordinate and
abscissa are equal subtends 900 at focus.
(2x  4a)  (t1  t 2 )y  0 ...(3)
Proof
As equation (3) is of the form L1 + L2 = 0, so by family
of lines concept we can say that it passess through a fixed let the normal at P(at12 , 2at1 ) meet the curve at Q(at 22 , 2at 2 )
point which point of intersection of L1 and L2.
 PQ is a normal chord
Hence the fixed point is (–2a, 0)
2
t 2   t1  ...(1)
Chord joining P(t1) and Q(t2) subtends  2 at vertex t1
of parabola if t1t 2   4
given condition 2at1  at12
Proof
As t1 = 2 so from equation (1), t2 = –3
Let the chord PQ subtends 900 at vertex C(0, 0) of the
parabola. P(4a, 4a) and Q(9a, –6a)
As focus is S(a, 0)
Therefore, slope of CP  2
t1 4a  0 4
Slope of SP  
4a  a 3
2
slope of OQ  6a  0 3
t2 Slope of SQ  
9a  a 4
As OP is perpendicular to OQ,
4 3
Slope of OP  slope of OQ = –1 Slope of SP  Slope of SQ     1
3 4
4 PSQ   / 2
 1  t1 t 2  4
t1 t 2
Hence PQ subtends a right angle at the focus S.
Corollary 1 Hence, we can also say that if normal chord at P(t) subtends
2
A normal chord at point P(t) to the parabola y = 4ax 900 at focus of parabola then t2 = 4.
subtends right angle at vertex if t2 = 2 Number of normals from a point to the parabola
Proof Let y2 = 4ax be the given parabola and P(h, k) be the given
Let the normal at P(t) to parabola y2 = 4ax meets parabola point.
2 The equation of any normal to the parabola y2 = 4ax
again at Q(t ') , then t '   t  ...(i)
t y = mx – 2am – am3

2at  0 2 If it passes through the point P(h, k), then


Now slope of AP   k = mh – 2am – am3
at 2  0 t
 am 3  m(2a  h)  k  0
2
And slope of AQ 
t' This is a cubic equation in m. So, it gives maximum three
normals. Corresponding to each value of m there is a normal
2 2 passing through the point P (h, k).
 AP  AQ    1  tt '  4
t t' Hence, from a fixed point maximum three normals can be
drawn the parabola and these number of normals will
depends upon number real roots of above cubic equation.

3.41
Coordinate Geometry

Co-Normal Points normals PA, PB and PC respectively.


From any point P maximum 3 normals can be drawn to  A(am12  2am1 ),B(am 22 , 2am 2 ) and c(am 23 , 2am 3 )
the parabola and the feet of normals where these normals
intersect parabola are known as co-normal points of parabola Also, m1  m2  m3  0 ...(3)
w.r.t point P.
2a  h
y m1 m 2  m 2 m 3  m 3 m1  ...(4)
a
A
B
k
x’
m1 m 2 m 3  ...(5)
O x a
P(h,k)
1) Algebraic sum of ordinates of conormal points is zero.
C Proof
y’
Let the ordinates of A, B, C be y1, y2, y3 respectively then

Let parabola be y2 = 4ax and P(h, k) is any point. y1 = – 2 am1, y2 = – 2am2 and y3 = –2am3
 Algebraic sum of these ordinates is
Equation of normal is y  mx  2am  am3 ...(1)
y1 + y2 + y3 = –2am1 – 2am2 – 2am3
As above equation passess through P(h, k), so
= – 2a(m1 + m2 + m3)
k = mh – 2am – am3
= –2a  0 = 0
3
 am  m(2a  h)  k  0 ...(2) 2) Centroid of triangle formed by three co-normal points lie
Which is cubic in m, having maximum three real roots. on axis of parabola.
Roots of equation (2) are slopes of normals from point P. Proof
Let roots of above equations are m1, m2, and m3 which are Centroid of three co-normals points is
slopes of normals PA, PB and PC respectively. Then co-
ordinates of Co-normal points are a 2a 
G   m12 ,  m1 
2
3 3 
A(am12 , 2am1 ) , B(am 22 , 2am 2 ) and c(am 3 , 2am 3 )
2
Also, Now, m 2
1    m1   2 m1 m 2

 2a  h  2h  4a and
m1  m 2  m 3  0
  0  2  m
1 0
  a  a

2a  h 
m1 m 2  m 2 m 3  m 3 m1  
a  ...(3)  2h  4a 
Co-ordinates of centroid are  ,0 
k   3 
m1 m 2 m 3  
a
3) Necessary condition for existance of three real normals
through the P(h, k), is h > 2a (if a > 0) and h < 2a
results of equation (3) are most useful in determining the (if a < 0)
properties of related to co-normal points.
Proof
Some standard results of co-normal points
Three normals are real, if all three roots of equation (2) are
Equation of normal to the parabola y2 = 4ax is real i.e.
y  mx  2am  am3 ...(1)
m12  m 22  m 23  0
As it passess through P(h, k), hence
 (m1  m 2  m 3 )2  2(m1 m 2  m 2 m 3  m 3 m1 )  0
3
am  m(2a  h)  k  0 ...(2)
 2a  h 
Let m1 , m 2 ,m 3 are roots of above equations which are slopes  0  2 0
 a 

3.42
Parabola

2a  h 4) If S is focus of parabola then SA  SB  SC  a SP 2


 0
a (Where A,B,C are co-normal points of parabola wrt point
P)
 if a > 0 then 2a < h
Proof
a < 0 then 2a > h
By definition of parabola as PS = PM i.e. distance between
note focus and moving point is equal to length of perpendicular
But if h > 2a (a > 0), we can’t say that all three from moving point on the directrix x + a = 0. Therefore
normals are real.
SA  a(m12  1)
4) Sufficient condition for three real and distinct normals
from point P(h, k) to parabola y 2 = 4a x is SB  a(m 22  1)
2 3
27ak  4(h  2a)
SC  a(m 23  1)
Proof
f(m) = am3 + (2a – h)m + k, it has 3 real and distinct SA  SB  SC  a 3 (1  m12 )(1  m 22 )(1  m 23 )
roots then

f '(m)  3am 2  2a  h  0 has 2 real and distinct roots  a 3 1   m12   (m1 m 2 )2  (m1 m 2 m 3 )2 

h  2a 1  ( m1 )2  2  m1 m 2 
3

i.e. m   (say)  are real roots a  2

3a    m1 m 2   2 m1 m 2 m 3  m1    m1 
 Sufficient condition for 3 real slopes is f().f()  0 .
2
  2a  h   2a  h  k2 
 a 3 1  0  2  
    0  
(,f()) y=f(m)
  a   a  a 2 


a 3 [h 2  k 2  a 2  2ah]
(m1,0) (m ,0) (m3,0)   a[(h  a)2  k 2 ]  aSP 2
a2
2

( ,f()) Circle through three co-normal points

 f()  f()  0  f()  f()  0


B
3 3 A
 [a  k  (2a  h)][a  k  (2a  h)]  0
( )
 k 2  (a 3  (2a  h))2  0 O

3 2
 
 h  2a  2 h  2a
 k  a
2
 (h  2a)  0 C
  3a  3a 
 
Let P  ,   is a point of intersection of three normal to the
3 3 2
  parabola.
(h  2a) (h  2a) 2 2
 k 2
  0
 3 3a 3a  If A(am12 , 2am1 ) , B(am 22 , 2am 2 ) and C(am 23 , 2am 3 ) are
 
 
three co-normal points of the parabola wrt point P, then
2
 2  am 3  (2a  )m    0 ...(E)
 k 2  (h  2a)3   
 3 3a  m1 + m2 + m3 = 0 ...(1)

 27ak 2  4(h  2a)3 (2a  )


m1 m 2  m 2 m 3  m 3 m1  ...(2)
a

3.43
Coordinate Geometry

Corollary 2

m1 m 2 m 3   ...(3)
a Common chords of circle and parabola exist in pairs and
each pair is equally inclined to the axis of parabola.
Let the equation of the circle through three co-normal point
Proof
be x 2  y 2  2gx  2fy  c  0 ...(4)
If the point (am 2, –2am) lies on it then y A
2 2 2 2
(am )  (2am)  2g(am )  2f(2am)  c  0 B

or a 2 m4  (4a 2  2ag)m2  4afm  c  0 ...(5) x

This is a biquadratic equation in m. Hence there are four


values of m i.e. m1, m2, m3 and m4 such that the circle
passes through the points. C
D
2 2 2
A(am , 2am1 ),B(am , 2am 2 ),C(am , 2am 3 )
1 2 3
and
2
D(am , 2a 4 ) Let A, B, C, D be the points of intersection of the circle and
4

the parabola with A  am1 , 2am1  ;B  am 2 , 2am 2  ;


2 2

 m1  m2  m3  m4  0
C  am 32 , 2am 3  and D  am 42 , 2am 4  then equation of
0 + m4 = 0 {from equation (1)}
 m4 = 0 AC and BD are y  m1  m 3   2x  2am1 m3 and
y(m2  m4 )  2x  2am 2 m4 respectively
 (am 24 , 2am 4 )  (0,0)

Hence the circle which passes through three co-normal points  Slopes of the chords AC and BD are
must also pass through the vertex of the parabola.
2 2
 and  m  m respectively
 c=0 m1  m 3 2 4

From (5), a 2 m4  (4a 2  2ag)m 2  4afm  0


2 2
Slope of AC   
3
 am  (4a  2g)m  4f  0 .....(6) m1  m 3 m 2  m 4

Now equations (E) and (6) are identical


 m1  m2  m 3  m4  0
4a  2g 4f
1   2 
2a         slope of BD
 m 2  m 4 
 2g  (2a  ), 2f   / 2
 Their slopes are equal in magnitude and opposite in
 The equation of the required circle is sign.

  The chords of AC and BD are equally inclined to the


x 2  y 2  (2a  )x  y0 axis.
2
Corollary 3
Corollary 1
Centroid of points where circle intersects the parabola lies
Algebraic sum of ordinates of four points of intersection of on axis of parabola
circle and parabola is zero.

sum of ordinates = 2am1  2am 2  2am3  2am 4  4 2


4

 am i  (2am i ) 
i 1
Centroid  , i 1 
 2a( m1 )  0  4 4 
 
 
This result is valid even if circle not passing through co-
normal points. a a 
  (( m1 )2  2  m1 m 2 ),  ( m1 ) 
4 2 

3.44
Parabola

So that the locus of (h, k), i.e. the required envelope, is the
a 2(4a 2  2ag)   parabola y2 = 4ax.
   0   ,0 
4 a2  
Hence, more similarly, the envelope of the straight line (1)
is the curve whose equation is obtained by writing down the
 (2a  g,0) condition that the equation (1), considered as a quadratic
Here y = 0 (which is axis of the parabola y2 = 4ax) equation in m, may have equal roots.

Envelope of Family of Curve a


By writing (1) in the form y  mx  ,
A curve C is said to be envelope of family of curves, if C m
touches each and every member of family of curves. Which is clear that it always touches the parabola
(OR) The curve which is touched by series of lines, which y2 = 4ax.
all satisfy some given condition, is called envelope of these Find the envelope of a straight line whose equation
lines. involves a second degree variable parameter
General Method to evaluate Envelope The equation to the straight line is of the form
(i) Differentiate the given family w.r.t parameter keeping x
and y constant.  2 P  Q  R  0 ...(1),

(ii) Then eliminate the parameters using equation of given where  is a variable parameter and P, Q, and R are
family of lines and equation obtained after differentiating expressions of the first degree in x and y.
equation of family.
Equation (1) may be looked upon as an equation involving
2
Let ty  x  at is family of lines (t is parameter on the two values of  corresponding to any given point T.
differentiating w.r.t ‘t’ keeping x and y constant, we get Through this given point two straight lines to touch the
y = 2at required envelope may therefore be drawn.
On eliminating t from above two equations, we get If the point T be taken on the required envelope, the two
2 tangents that can be drawn from it coalesce into the one
y.y  y  y2 y2
 x  a    x tangent at T to the envelope.
2a  2a  2a 4a
Conversly, if the two straight lines given by (1) coincide, the
 y 2  4ax is required envelope. resulting condition will give us the equation to the envelope.
Curve touched by a variable straight line whose But the condition that (1) shall have equal roots is
equation involves a second degree variable parameter Q2 =4PR ...(2)
As an example, let us find the envelope of the straight lines This is therefore the equation to the required envelope
given by the equation
Since P, Q and R are all expressions of the first degree the
m2x – my + a = 0 ....(1) equation (2) is, in general an equation of the second degree,
where m is a quantity which, by its variation, gives the and hence, in general, represents a conic section.
series of straight lines. The envelope of any straight line, whose equation contains
If (1) pass through the fixed point (h, k), we have an arbitrary parameter and square thereof, always
reperesents a conic.
m2h – mk – a = 0 ...(2)
Solved Examples
This is an equation giving the values of m corresponding to
Example 58
the straight lines of the series which pass through the point
Find the condition for line lx + my + n = 0 be normal
(h,k). There can therefore be drawn two straight lines from to the parabola y2 = 4ax.
(h, k) to touch the required envelope. Solution
As (h, k) moves nearer and nearer to the required envelope lx + my + n = 0 ...(1)
these two tangents approach more and more nearer to is a normal to y2 = 4ax ...(2)
coincidence, until, when (h, k) is taken on the envelope, the Let P(t) be the foot of the normal then equation the
two tangents concide. normal is y + xt = 2at + at3
 xt  y  (2at  at 3 )  0 ...(3)
Conversely, if the two tangents given by (2) coincide, the
point (h,k) lies on the envelope. Now the root of (2) are (1) & (3) represent the same line
equal it k2 = 4ah.

3.45
Coordinate Geometry

For this to be normal to y2 = 4ax, we must have


l m n – mb – 2cm – cm3 = – 2am – am3
  
t 1 (2at  at 3 )  b  2c  cm 2  2a  am 2
l n  b  2c  2a  (a  c)m 2
t and 2at  at 3  
m m
On eliminating t from above equations b  2(a  c) b
 m2  m 2
a c ac
 l  l3 n
 2a    a 3  Since m is real, therefore,
 m m m
b b
 al 3  2alm2  m 2n  0 20 2.
a c ac
Example 59 Example 62
Prove that the point on the parabola y2 = 4ax (a > 0) Find the locus of the point N from which 3 normals
nearest to the focus is its vertex. are drawn to the parabola y2 = 4ax are such that
Solution (i) Two of them are equally inclined to x–axis in
Let P(at2, 2at) be a point on the parabola y2 = 4ax, opposite direction
which is nearest to the focus S(a, 0) then (ii) Two of them are perpendicular to each other
SP 2  (at 2  a)2  (2at  0)2 Solution
Equation of normal to y2 = 4ax is
Let f(t)  a 2 (t 2  1)2  4a 2 t 2
y = mx – 2am – am3
f '(t)  a 2 2(t 2  1)(2t)  4a 2 (2t) Let the normal passes through N(h, k)
 k = mh – 2am – am3
 4a 2 t(t 2  1  2)  4a 2 t(t 2  1)
 am 3  (2a  h)m  k  0 ...(1)
for extremum value of f(t), f '(t)  0  t  0
For given value’s of (h, k) it is cubic in ‘m’.
f ''(t)  4a 2[3t 2  1], f ''(0)  4a 2  0 Let m1, m2 & m3 are root’s of above equation
 At t = 0, f(t) is minimum, then P = (0, 0)  m1  m 2  m 3  0 ...(2)
 The point on the parabola y2 = 4ax, which is nearest
to the focus is vertex A (0, 0). 2a  h
m1m 2  m2 m3  m3 m1  ...(3)
Example 60 a
Find the equations of the normals at the ends of the
k
lauts-rectum of the parabola y2 = 4ax. Also prove m1m 2 m3   ...(4)
that they are at right angles on the axis of the parabola. a
Solution (i) If two normal are equally inclined to x–axis, then
The coordinates of the ends of the latusrectum of the m1 + m2 = 0  m3  0
parabola y2 = 4ax are (a, 2a) and (a, –2a).
Equation of normal at (a, 2a) is As m3 is root of equation (1) so locus of N is y = 0.
2a (ii) If two normal’s are perpendicular
y  2a   (x  a) or x  y  3a  0 ...(i)
2a  m1m 2  1
The equation of the normal at (a, –2a) is k
from (3) m3  ...(5)
 2a  a
y  2a    (x  a) or x  y  3a  0 ...(ii)
 2a  k
On putting m3  in equation (1) we get
Clearly, (i) and (ii) are perpendicular as the product of a
their slopes is –1. The point of intersection of (i) and k 2  a(h  3a)
(ii) is (3a, 0), which lies on the axis of the parabola.
Example 61 So, locus is y 2  a(x  3a)
Prove that two parabola y 2 = 4ax and y2 = 4c Example 63
(x – b) can not have a common normal unless Prove that the locus of points such that two of the
three normals from them to the parabola y2 = 4ax
b
2. coincide is 27ay2 = 4(x – 2a)3.
ac Solution
Solution In the previous example, we have from (i)
The equation of any normal of slope m to the parabola
m1  m 2  m3  0 ...(i)
y2 = 4c (x – b) is
y = m (x – b) –2cm – cm3 (2a  h)
or, y = mx – mb – 2cm – cm3 m1m 2  m 2m 3  m 3 m1  ...(ii)
a

3.46
Parabola

(–4, 0) and which cuts the parabola y2 = 8x at A and


k B such that its common chord AB subtends a right
and m1m 2 m3   ...(iii)
a angle at the vertex of the parabola is equal to “4”.
But here two of the three normals are given to be Solution
coincident i.e. m1 = m2 Putting m1 = m2 in (i) and Let r be the radius of the circle. Then its equation is
(iii), we get (x + 4)2 + y2 = r2 ..(1)
This cuts the parabola y2 = 8x at points A(x1, y1) and
2m1  m3  0 ...(iv) B(x2, y2)
2 k The abscicsae of A & B are the roots of the equation
and, m1 m 3  ...(v) (x+4)2 + 8x = r2 (or) x2 + 16x + 16 – r2 = 0,
a
Putting m3 = –2m1, from (iv) in (v), we get x 1  x 2  16 and x 1x 2  16  r 2 ...(2)
k k the ordinates of A and B are given by y12  8x 1 and
2m13    m13 
a 2a
y 22  8x 2 respectively..
Since m1 is a root of the equation
am3  m(2a  h)  k  0  y1  2 2x 1 and  y 2  2 2x 2
Since AB subtends a right at the vertex of the parabola.
 am13  m1 (2a  h)  k  0
So, slope of OA x Slope of OB = –1
1/ 3
k  k  y1 y 2
   (2a  h)  k  0    1  x 1x 2  y1 y 2  0
2  2a  x1 x 2
1/ 3
 k  3k  x 1x 2  8 x 1x 2  0  x 1x 2  0
  (2a  h)  
 2a  2
 16  r 2  0  r  4
k 27k 3 Example 66
(2a  h)3   27ak 2  4(h  2a)3 Prove that the locus of the point of intersection of the
2a 8
normals at the ends of a system of parallel chords of
Hence, the locus of (h, k) is 27 ay2 = 4(x – 2a)3
a parabola is a line which is normal to the given
Example 64
parabola.
The normal at any point ‘P’ meets the axis in G and
Solution
the tangent at the vertex in G ' ; If A be the vertex and Let the given parabola be
the rectangle AGQG ' be completed. Find locus of Q. y2 = 4ax ...(1)
Solution Any normal at ‘t’ is
Normal at P = (am2, –2am) y = –tx + 2at + at3 ...(2)
Let PQ be any chord where
y  mx  2am  am3 cuts x-axis at
P  (at12 , 2at 1 ) and Q  (at 22 , 2at 2 )
y  0, x  2a  am2
Let slope of chord PQ be m

Q 2at 2  2at1 2
G’  m  ...(3)
at 22  at12 t 2  t1
P Now normals at P and Q intersect at R(x1, y1)
then x 1  2a  (t12  t1 t 2  t 22 ); y1  at 1t 2 (t1  t 2 )
A G x
or (x 1  2a)  a{(t 1  t 2 )2  t1 t 2 ); y 1   at1 t 2 (t1  t 2 )

 4 
or x1  2a  a  2  t1 t 2  ...(4)
2
G  (2a  am , 0) cuts y-axis at  m 
x = 0, y = –2 am – am3
Let co ordinates of Q be (h , k) then  2
y1  at1 t 2   ...(5)
...(i) m
h  2a  am 2
k = – 2a m – am3 ...(ii) From (4) and (5), we get
Eliminating m from (i) & (ii), we get  4 my1 
x1  2a  a  2  
h 3  2ah 2  ak 2 m 2a 
So, locus of Q is x3 = 2ax2 + ay2.
Example 65 my1 4a
or  x1  2a  2
Prove that the radius of the circle whose centre is 2 m

3.47
Coordinate Geometry

3
2 2 2 1 
or y 1    x 1  2a    a      tan1  tan  
m m m 2 
 The locus of R(x1, y1) is Example 68
 2  2  2
3 If a2 > 8b2, prove that a point can be found such that
y      x  2a     a    the tangent from it to the parabola y2 = 4ax are
 m  m  m
normals to the parabola x2 = 4by
which is normal to (1) at the point whose parameter Solution
is t = –2/m Two parabola are given as
Example 67 y2 = 4ax ....(1) and x2 = 4by ...(2)
If the normal to a parabola y2 = 4ax, makes an angle
 with the axis of the parabola then it will cut the a
Tangent to (1) y  mx  ...(3)
m
1  Equation to any normal to (2) is
curve again at an angle tan1  tan   .
2 
x  m ' y  2bm ' bm' 3
Solution
1
Let the normal at P(at12 , 2at 1 ) be or y  x  2b  bm ' 2 ...(4)
m'
y   t1x  2at1  at 13 . If the equations (3) and (4) are the same, then the
coefficients must be identical. As the coefficients of y
 tan   t1  slope of the normal ...(1)
are equal in both, so coefficients of x and constant
It meet the curve again Q say (at 22 , 2at 2 ) terms will also be equal. Hence,
1 1
2 m or m '  ...(5)
 t 2   t1  ...(2) m' m
t1
a
and  2b  bm' 2 ...(6)
Y m
) Putting the value of m' from (5) in (6), we get
2
,2at 1
P(at 1 a b
 2b  2 or 2bm 2  am  b  0 ...(7)
m m
If the roots of (7) are real, then its discriminant must
X’ X be greater than zero.
A
 Therefore a 2  4. 2b.b  0 or a 2  8b2 which is the
Q(at2 2,2at given condition.
2 ) Example 69
Y’ From the point where any normal to the parabola y2
= 4ax meets the axis a line is drawn perpendicular to
Now angle between the normal and parabola this normal. Prove that this line always touches an
= Angle between the normal and tangent at Q equal parabola.
Solution
(i.e., t 2 y  x  at 22 ) The equation of any normal to the parabola is
If  be the angle, then
y  mx  2am  am3
1 This meets the axis in the point (2a + am2,0).
t1  The equation to the straight line through this point
t2 t t 1
m1  m 2    12 perpendicular to the normal is
tan   1 t 2  t1
1  m1m 2 1  ( t1 )   y  m1 (x  2a  am 2 ) where m1m  1
 t2 
The equation is therefore

 2  a 
t1   t1    1 y  m1  x  2a  2 
t1   m 1 
   t tan 
2   1 
 t1   t 1 2 2 a
t1 i.e. y  m1(x  2a) 
m1
This straight line, always touches the equal parabola
y2 = – 4a (x – 2a),

3.48
Parabola

whose vertex is the point (2a, 0) and whose concavity


is towards the negative end of the axis of x. A
Example 70 B
Prove that from the point (at2, 2at) two normals can
be drawn to the parabola y2 = 4ax and the parameters ()
of their feet Q and R will satisfy the equation 2 + t
+ 2 = 0. C
Solution D
Equation of a normal to the parabola y2 = 4ax can Let t1, t2, t3 be the parameters of the points which
be written as
form an equilateral triangle. Then we have
y  mx  2am  am3 ...(1)
a(t12  t 22  t 23 )  3 ...(6)
2
Since this normal passes through (at , 2at), therefore
and 2a(t1 + t2 + t3)= 3 ...(7)
we have
Using results (2) and (7), we have
m3 + (2 – t2) m + 2t = 0 ...(2)
One of the roots of this equation is equal to the slope 3
t 4    t i   (t1  t 2  t 3 )  
of the normal at the point (at2, 2at) itself which is 2a
equal to –t. Hence, the cubic equation (2) can be gives 2at4 = –3
written as (m  t)(m 2  mt  2)  0 ...(3) Also, we have
The other two roots of this equation given by the 2  2 
equation t12  t 22  t 23  t 24    ti   2   ti t j   2  4  
 a 
(m2 – mt + 2) = 0 ...(4)
denote the slopes of the other two normals. gives at 24    8a ...(9)
If  be the parameter of the feet of a normal, then
Hence, the coordinates of the fourth point are
slope of the normal = –
Hence, putting m = –, the above quadratic equation (at 24 , 2at 4 )  (  8a, 3) .
in m becomes Example 72
 2  t  2  0 ...(5) A variable chord PQ of the parabola y2 = 4ax subtend
whose roots denote the parameter of the feet of the a right angle at the vertex. Show that the locus of the
normals. point of intersection of the normals at P and Q is y2 =
Example 71 16a (x – 6a).
A circle is drawn to cut the parabola y2 = 4ax at four Solution
points. If three of those points form an equilateral Let P   at12  2at1  and Q   at 22  2at 2  .
triangle show that the coordinates of the fourth point
will be ( – 8a, –3) where () are the coordinates 2at1 2
Now, slope of OP   and slope of
of the centre of the circle. at12 t1
Solution
Let ( ) be the centre of the circle and r be the 2at 3 2
OQ    As OP  OQ 
radius. at 22 t2
A point on the given parabola can be chosen as (at2
,2at). If this point must also lie on the circle, then we  t1 t 2   4 ...(i)
have Again, let the normals at P and Q intersect at R().
(at 2  )2  (2at  )2  r 2 Thus the equation of normal at P(t1) is
 y  t 1x  2at 1  at13 ...(ii)
i.e. a 2 t 4  (4a 2  2a)t 2  4at  ( 2  2  r 2 )  0
Similaly, the equation of normal at Q(t2) is
...(1)
If ti, i = 1, 2, 3, 4 be the roots of this bi-quadratic y  t 2 x  2at 2  at 32 ...(iii)
equation, then
P(t1)
t i 0 ...(2)

 2 
t t i j  4 
 a 
 ...(3)
0
R()
4
t t t
i j k 
a
...(4)
Q(t2)
 2  2  r 2
and t i 
a2
...(5)

3.49
Coordinate Geometry

On solving equation (2) and (3) we get Example 74


2 2
x  a(2  t  t1 t 2  t ) and y  at1 t 2 (t1  t 2 )
From any point (h, k), three normals are drawn to the
1 2
parabola y2 = 4ax and the tangents at their feet are
  a(2  t12  t1 t 2  t 22 ) and   at1t 2 (t1  t 2 ); drawn. Show that the co-ordinates of the vertices of
the triangle formed by these tangents are roots of
Since t1t 2  4 then from (1) we get.
x 3  (h  2a)x 2  ak 2  0, y 3  a(h  2a)y  a 2 k  0 .
2 2 2 2
   a(2  t  4  t )  a(2  t  t )
1 2 1 2 ...(iv) Solution
and   4a(t1  t 2 ) ...(v) The equation of the normal at the point (at2, 2at) is
after eliminating t1, t2 from (iv), (v) we get y  tx  2at  at 3 , which passes through the point (h,
2  16a(  6a) k), so k  th  2at  at 3  at 3  t(2a  h)  k  0
Hence the required locus of (  ,  ) is ...(i)
Since this equation is a cubic in t, so it has three
y2 = 16a(x – 6a) normals from the point (h, k) to the parabola.
Example 73 Let the feet of the normals be
P be a point on the parabola y2 = 4x with the ordinate
y satisfying 1 < y < 2. The normal to the parabola at P  at12 , 2at1  , Q  at 22 , 2at 2 
P intersects the x-axis in N and a line parallel to y-axis
through P intersects the x-axis in M. Find the minimum A P
difference in the areas of s PMN and PMS where S
is the focus of the parabola. Q
B (h,k)
Solution
Let any point on the parabola y2 = 4x is P(t2, 2t) C
R
1
 1  2t  2  1  y  2 ,   t  1 R  at , 2at 3  and tangents at these points intersect at
2
3
2
The equation of the normal at A, B,C.

y A  {at1t 2  a(t1  t 2 )},B  {at1 t 3  a(t1  t 3 )} and


t) C  {at 2 t 3  a(t 2  t 3 )}
P( t , 2
2

We denote them by (1 , 1 ),( 2 , 2 ) and ( 3 , 3 )


0 x
S M N from (i) : t1  t 2  t 3  0 ....(ii)
2a  h
t 1 t 2  t 2 t 3  t 3 t1  ...(iii)
a
P(t 2 , 2t) is y + tx = 2t + t3...
k
(i) which intersects the y-axis at N(2 + t2,0) and t1t 2 t 3  ...(iv)
PM = y co-ordinate of P = 2t. a
Sum of the x-co-ordinates of A, B, C are
1 1
 PMN   MN  PM  (ON  OM)  PM 1   2   3  at1t 3  at1t 2  at 2 t 3
2 2
1 2a  h
2 2
 (2  t  t )  2t  2t  a(t1t 3  t 2 t 3  t 3 t1 )  a   2a  h
2 a
Also,
1 1
and  PMS   SM  PM  (t 2  1)  2t  t(t 2  1) . 1 2   2 3   31
2 2
 at1 t 2  at1 t 3  at1t 3  at 2 t 3  at 2 t 3  at1t 2
Now PMN  PMS  3t  t 3  A (say).
 a 2 t1 t 2 t 3 (t1  t 2  t 3 )  0
dA
For maxm. min.,  0,  3  3t 2  0  t  1
dt k2 3
 ak 2
Finally 1 2 3  at1t 2  at1t 3  at 2 t 3  a 
a2

But t  1 because  t  1 t  1 . The cubic equation whose roots are 3 is
2
2
x 3  x 2  1  x   1 2  1 2 3  0 .
d A
Since  6t  0 , so the minimum difference in  x 3  (2a  h)x 2  ak 2  0 which is the required
dt 2
equation whose roots are abscissae of A, B and C.
their areas  3  1  13  2 sq. units. Similarly, we can prove the second part.

3.50
Parabola

Example 75 Solution
If the three normals from a point to the parabola Any normal to the parabola y2 = 4ax is
y2 = 4ax cut the axis in points whose distance from y = mx – 2am – am3 ...(1)
the vertex are in AP, show that the point lies on the Also any point on the line y = k is (x1, k)
curve 27ay2 = 2(x – 2a)3 If (1) passes through (x1, k) then
Solution am 3  m(2a  x 1 )  k  0 ...(2)
Let (h, k) be the point of intersection of three normals
to the parabola The equation of any normal is If the roots of this equation are m1, m2, m3 then we
get m1+ m2+ m3 = 0
y  mx  2am  am2 ...(1)
(2a  x 1 )
If it passes through (h, k) then m1m 2  m 2m 3  m3 m1 
2m3 + m(2a – h) + k = 0 ...(2) a
let roots of above equation be m1, m2, m3 then k
m1 + m2 + m3 = 0 ...(3) and m1m 2m3  
a
(2a  h) Also the co-ordinates of three points P, Q and R
m1 m2 +m2 m3 + m3 m1 = ...(4)
a are (am12 , 2am1 ) , (am 22 , 2am 2 ) , (am 23 , 2am 3 )
k
m1m 2 m3   ...(5) respectively.
a The equation of the line PQ is
Y (2am 2 )  (2am1 )
y  (2am1 )  (x  am12 )
am 22  am12

2
V B C  y  2am1   (x  am12 )
X’ X (m 2  m1 )
A
P(h,k) or y(m1  m 2 )  2am1 (m1  m 2 )  2x  2am12
or y(m1  m 2 )  2am1m 2  2x
Y’ 2am1m 2 m 3
As normal cuts the axis of parabola at (2a+am2, 0) or y(m1  m 2  m 3  m 3 )   2x
m3
then the normal through (h, k) cuts the axis at
2k
A(2a  am12 , 0) , B(2a  am 22 , 0) and C(2a  am 23 , 0) or y(0  m 3 )   2x
m3
and Let V(0, 0) be the vertex of the parabola.
Then VA  2a  am12 , VB  2a  am 22 and   ym 32  2k  2m 3 x

VC  2a  am 23  ym 32  2m 3 x  2k  0
As VA, VB and VC are in AP which is a quadratic in m3 since PQ will touch it then
2VB = VA+VC B 2  4AC  0
2 2 2
4a  2am  2a  am  2a  am
2 1 3  (2x)2  4  y  2k  0  x 2  2ky  0
2m 22  m12  m 32  2m 22  (m1  m 3 )2  2m1m 3 Make Concepts Clear 3.4
1. Tangents PT and QT to the parabola y2 = 4x intersect
2m1m 2 m 3
2m 22  (m1  m 2  m 3  m 2 )2  at T and the normals at the points P and Q intersect
m2 at the point R (9, 6) on the parabola. Find the
2  k coordinates of the point T and circumcircle of PTQR.
2m 22  (0  m 2 )2   
m2  a  2. A family of chords of the parabola y2 = 4ax is drawn
so that their projections on a straight line equally
2k inclined to both the axes are all of a constant length
m32  ...(6)
a c. Then prove that the locus of their middle point is
As m 2 is root of equation (2), so it satisfy (2) So,, the curve (y2 – 4ax)(y + 2a)2 – 2a2c2 = 0
3. A variable chord of a given parabola passes through
27ak 2  2(h  2a)3 a fixed point. The circle on this chord as diameter
cuts the parabola again at two other points. Prove
Hence locus of (h, k) is 27ay 2  2(x  2a)3 .
that the lines joining these two points passes through
Example 76 another fixed point whose ordinate is equal in
The normals at P, Q, R on the parabola y2 = 4ax magnitude to the ordinate of the first point.
meet in a point on y = k. Prove that the sides of the 4. From a point P on the axis of the parabola y2 = 4ax,
triangle PQR touch the parabola x2 – 2ky = 0.

3.51
Coordinate Geometry

normals PA and PB (other than the axis of the the tangent at a third point R cuts them in P' and
parabola) are drawn to the parabola. Prove that for
every permissible position of the point P, a circle TP ' TQ '
Q ' prove that   1.
circumscribing the triangle formed by the tangents TP TQ
drawn to the parabola at A, B and the vertex subtends 20. Prove that locus of circumcentre of variable triangle
an angle 2 sin–1(1/3) at P. having sides x = 0, y = 2 and its third side touches
5. Normals drawn at points P, Q, R on the parabola the parabola y2 = 4x is also a parabola.
y2 = 4ax meet on the line x = . Show that the sides 21. Prove that the area of the triangle formed by the
of the PQR touch the parabola y2=16a(x+2a –). normals to the parabola y 2 = 4ax at the points
6. Find the locus of the centres of the circles which pass P(at 12 , 2at 1 ), Q(at 22 , 2at 2 ) and R(at 23 , 2at 3 ) is
through the focus of the parabola y2 = 4ax and cut it
orthogonally. 1 2
a (t1  t 2 )(t 2  t 3 )(t 3  t1 )(t1  t 2  t 3 )2
7. Prove that the shortest normal chord of the parabola 2
y2 = 4ax is 6a 3 and that its inclination to the axis 22. Find the locus of the points such that the sum of the
angles which the three normals drawn from them to a
is tan1 2 . parabola make with the axis of the parabola is
8. The normals from (P, 0) are are drawn to the parabola constant, is a straight line.
y2 = 8x, one of them is the axis. If the remaining 23. Find locus of a point P when three normals drawn
normals are perpendicular find the value of P. from it to the parabola y2 = 4ax are such that one
9. Three normals with slopes m1, m2 and m3 are drawn bisects the angle between the other two.
from a point P not on the axis of the parabola 24. If tangents are drawn to y2 = 4ax from any point on
y2 = 4x. If m1m2 = , results in the locus of P being a y2 = a(x + b) then show that normals drawn at either
same parabola, find the value of . point of contact meet on fixed line.
10. Show that normal to the parabola y2 = 8x at (2, 4)
meet it again at (18, –12).
Answer Key
11. Prove that the locus of the middle point of the portion
of a normal intersected between the curve and the 1.T(2, –3) and x 2  y 2  11x  3y  0
axis is a parabola whose vertex is the focus and whose 2 2
latus rectum is one quarter of that of the original 6. y (a  2x )  a(3x  a ) 8.6
parabola. 9.2 13.(0, 0) (–4,3) & (16, 8)
12. A point on a parabola, the foot of the perpendicular
from it on the directrix, and the focus are the vertices 14.343y2 = 48(x – 6)3 15. 

4 2 3 
of an equilateral triangle. Prove that the focal distance 3
of the point is equal to the latus rectum.
17. x 2  2(y  6) 22. 2x  (cot  )y  2a  0
13. Three normals are drawn from the point (7, 14) to the
2 2
parabola x2 – 8x – 16y = 0. Find coordinates of the 23.27ay = (x – 5a) (2x – a)
feet of the normals. Solutions are on Page No. 3.110
14. The ordinates of point P and Q on the parabola
y2 = 12x are in the ratio 1 : 2. Find locus of the point
of intersection of the normals to the parabola at P 3.5 Chord of Contact, Pair of
and Q.
Tangents, Diameter, Pole and
15. If the line y  3x  3 cuts the parabola y2 = x + 2 Polar
at P, Q and if A be the point  
3, 0 , then find value Length of tangent, Subtangent, Normal and
of AP  AQ Subnormal
16. If tangents be drawn from points on the line x = c to
the parabola y2 = 4ax, show that the locus of point Y'
of intersection of the corresponding normals is the
parabola ay2 = c2(x + c – 2a)
17. Find locus of point of intersection of two perpendicular , y)
P(x
2
normals to x  8y . 0
90 - 

18. The normals at P, Q, R are concurrent and PQ meets X' X
T A N G
the diameter on the directrix x = –a. Prove that PQ
touches [ or PQ envelopes] the parabola
y2 + 16a (x + a) = 0.
19. TP and TQ are any two tangents to a parabola and Y

3.52
Parabola

Let the parabola be y2 = 4ax and P(x, y) is a point on


y2
parabola from where length of tangent, normal, sub tangent |TN | 
and sub normal has to be find. 2a

Let tangent and normal at point P meet the axis of parabola d) Length of Subnormal
at T and G respectively and N is foot of perpendicular from Projection of normal on axis of parabola is length of
P on axis of parabola. subnormal i.e., NG.
Let tangent at point P makes angle  with positive direction
PN
of x – axis. In PNG, tan  90    
NG
a) Length of tangent
In figure length PT in length of tangent. dy 2a
 |NG|  PN tan   y.  y.
dx y
PN
In PTN sin  
PT  |NG|  2a
Therefore, for parabola length of subnormal is constant
 |PT|  PN cos ec   y 1  cot 2  and is equal to semi latus rectum.

2
Chord of Contact
 dx 
 y 1   The chord joining the points of contact of two tangents
 dy  drawn from external point to a parabola is known as chord
of contact of tangents drawn from external point.
y2 Equation of chord of contact
|PT|  |y| 1 
4a 2
b) Length of Normal ')
', y
Q(x
Length PG is length of normal.
In PNG X' X
P( x Chord of contact
1 ,y)
PN 1
R (x
sin  90     ", y
PG ")

 |PG|  PN sec   y 1  tan2  Let PQ and PR be tangents to the parabola y2 = 4ax drawn
from any external point P(x1, y1), then QR is called chord of
2
contact of the parabola y2 = 4ax.
 dy 
 y 1  
 dx  Let Q   x ', y '  and R   x ", y "

Equation of tangent PQ is yy '  2a  x  x '  ...(1)


4a 2
| PG | = |y| 1 
y2
the equation of tangent PR is yy "  2a  x  x "  ...(2)
c) Length of Sub tangent
Since lines (1) and (2) pass through (x 1, y 1) then
Projection of tangent on axis of parabola is length of y1 y '  2a  x1  x '  and y1 y "  2a  x1  x " 
subtangent i.e., TN.

PN Hence it is clear Q  x ', y '  and R  x ", y " lie on


In PTN, tan   yy1 = 2a (x + x1)
TN
which is chord of contact QR.
dx y
 |TN|  PN cot   y . y. Hence chord of contact with respect to external point
dy 2a P(x, y) of parabola is S1 = 0.
Above result is valid for all parabolas.

3.53
Coordinate Geometry

Length of Chord of contact


Y'
Given parabola is y2 = 4ax ...(1)

Y'
Q (h1 , k1 )
X' X
A

, y 1) (at 2
1 ,2at1 ) P(x1, y )
P(x
1 1
R (h 2 , k 2 )
A
X' X Y

R (at 22 , 2at 2 )
The ordinates of Q and R are the roots of the equation.
 yy  2ax 1 
Y y 2  4a  1 
 2a 

Let P  (x1 , y1 ) y2 = 2(yy1 – 2ax1) ...(1)

 y 2  2yy1  4ax 1  0
Let the tangents from P touch the parabola at  at12 , 2at1 
k1 + k2 = 2y1 and k1k2 = 4ax1
and R  at 22 , 2at 2  then P is the point of intersection of
2
tangents.   k 2  k1    k1  k 2   4k1 k 2
 x1 = at1t2 and y1 = a(t1 + t2)
  4y 2
1  16ax 1   2 y 2
1  4ax 1 
x y
or t1 t 2  1 and t 1  t 2  1
a a Since Q (h1, k1) and R (h2, k2) lie on the parabola y2 = 4ax
therefore
2 2
Now QR   at 2
1  at 22    2at1  2at 2 
k12  4ah1 and k 22  4ah 2

 k 22  k12  4a(h 2  h1 )
 |a||t1  t 2 |  t 1  t2   4
2

(k2 + k1) (k2 – k1) = 4a (h2 – h1)
 |a |  t 1
2
 t 2   4 t1 t 2  
2
 t 1  t 2   4 
  2y1 (k 2  k1 )  4a(h 2  h1 )

y1  k 2  k 1 
 y 2 4x   y 2    h 2  h1  
 |a |  12  1  .  12  4  2a
a a  a 
2 2
Now, QR   k 2  k1    h 2  h1 
y 2
1  4ax 1  y 2
1  4a 2 
 |a| . 2
|a| |a | 2 y12  k 2  k1 
  k 2  k1  
4a 2
1

|a|
y 2
1  4ax 1  y12  4a 2 
 k 2  k1 
 y 12  4a 2
2|a|
Alternative Method

yy1  2ax1 2 (y12  4ax1 )


The equation of QR is yy1 = 2a(x + x1) or x   y 2
 4a 2 
2a 2|a |
1

3.54
Parabola

Equation of Pair of tangents


1
 (y 12  4ax1 )  y12  4a 2 
|a | Method - 1
Let P(x1, y1) is any point outside the parabola from where
1 two tangents PQ and PR are drawn..
Hence, |QR |  S11  y12  4a 2 
|a | Let T(h, k) be any moving point on either of the tangents
from P(x1, y1), then locus of T is pair of tangents. The
Theorem
equation to the line joining P and T is
Two tangents can be drawn from a point to a parabola.
The two tangents are real and distinct or coincident or k  y1
imaginary according as the given point lies outside, on or
y  y1   x  x1 
h  x1
inside the parabola.
Proof k  y1 hy  kx1
i.e., y  x 1
Let the parabola be y2 = 4ax and P(h, k) be a point. The h  x1 h  x1
equation of any tangent to the given parabola is
If this be a tangent it must be of the form
a
y  mx  . k  y1
m
y  mx 
a
so that,  m and hy1  kx  a .
m h  x1 hx m
If it passes through (h, k), then

k  y1 hy1  kx
Y' Hence, by multiplication a 
h  x 1 h  x1

T1 T(h, k)
X' X
Q
P(x1, y1)
P(h, k)

T2 R
Y
2
i.e., a  h  x 1    k  y 1  hy1  yx 1 
a
k  mh   m2h – mk + a = 0  The locus of the point T(h, k), (i.e., the required pair of
m
tangents) is therefore.
This equation being a quadratic in m, gives two values of
m, corresponding to each of these values of m there is a y 2
 4ax  y12  4ax 1   yy1  2a  x  x 
2

tangent to the parabola.


Thus, two tangents can be drawn from a point to a parabola. SS11  S12
The tangents are real and distinct, coincident or imaginary
according as the roots of quadratic in m are real and distinct, Method - 2
equal or imaginary i.e., according as k2 – 4ah > or = or Let P(x1, y1) be any point outside the parabola. Let a chord
<0 of the point through the point P(x1, y1) cut the parabola at
i.e., (h, k) lies outside or on or inside the parabola Q and let R (h, k) be any point on the line PQ (R inside or
y2 = 4ax. outside).
Pair of tangents Let Q divide PR in the ratio  : 1 then co-ordinates of Q is
If from any external point P two tangents PA and PB are
drawn to parabola, then joint equation of PA and PB are  h  x 1 k  y 1 
 , 
known as pair of tangents with respect to point P.   1  1 

3.55
Coordinate Geometry

Since Q lies on parabola (1) then


Chord bisected at given point

 k  y 1 
2
 h  x1  Equation of chord of the parabola y2 = 4ax which is bisected
   4a   at P(x1, y1) is S1 = S11
  1    1 
Proof
2
or  k  y1   4a  h  x 1    1   0 The equation of a line passing through (x1, y1) is
y – y1 = m (x – x1) ...(1)
or
2
The abscissae of the points of intersection of y = 4ax and
 k 2  4ah   2  2  ky1  2a  h  x1     y12  4ax1   0 ...(2) (1) the roots of the equation.
(y1 – mx1 + mx)2 = 4ax
Line PR will become tangent to parabola (1) then roots of
equation (2) are equal m2x2 + 2 [ – 2a + my1 – m2x1] x + (y1 – mx1)2 = 0 ...(2)
2
 4  ky1  2a  h  x 1    4  k 2  4ah  y12  4ax1   0 let h1 and h 2 be the roots of this equation, Then,
 2  2a  my1  m 2 x 1 
2 h1  h 2 
or ky 1  2a  h  x1    k 2  4ah  y12  4ax1  m2

Hence locus of R (h, k) i.e., equation of pair of tangents Since (x1, y1) is the mid - point of the chord joining the
from P(x1, y1) is points of intersection of (1) and the parabola.

{yy1 – 2a(x + x1)}2 = (y2 – 4ax)  y 12  4ax 1  h1  h 2  2a  my 1  m 2 x 1 


 x1   x1 
2 m2
i.e. SS1 = T2.
Area of triangle formed by pair of tangents and chord 2a
 m
of contact y1
Let P(x1, y1) is any external point with respect to parabola Substituting this value of m in (i), we obtain
y2 = 4ax, from where two tangents PQ and PR are drawn,
then QR is chord of contact with respect to point P. 2a
y  y1   x  x1 
y1
Equation of QR (chord of contact) is
yy1 = 2a(x + x1)  yy1  y12  2a  x  x1 
yy1 – 2a (x + x1) = 0
 yy1  2a  x  x1   y12  4ax1
PM = Length of perpendicular from P on QR
 S1  S11
|y 1 y1  2a  x 1  x 1 | | y  4ax 1 |
2
1
  This is the equation of the chord bisected at (x1, y1)
y 2
1  4a 2
  y12  4a 2  Length of chord bisected at given point
Let P(x1, y1) is mid point of chord AB w.r.t parabola
Since P(x1, y1) lies outside the parabola y 12  4ax 1  0 y2 = 4ax.

1 Let co-ordinates of A and B are A(at12 , 2at 2 ) and


As area of PQR  QR.PM
2
B(at 22 , 2at 2 )

1 1  y  4ax 
2
1 1
a(t12  t 22 )

2| a |
y 2
1  4ax 1   y 12  4a 2  Then x 1  and y1  a(t1  t 2 ) ...(1)
 y  4a 
2
1
2
2
From equation (1) i.e. values of x1 and y1
3 3


y 2
1  4ax1  2
if a  0 i.e.
 S11  2
y 12  2ax 1
t1t 2  ...(2)
2a 2a 2a 2

3.56
Parabola

every value of t1 and t2 which lies on directrix.


Now, | AB |  a 2 (t 12  t 22 )2  4a 2 (t1  t 2 )2
Hence, directrix of parabola act as director circle of it. (OR)
we can also that two perpendicular tangents always intersect
 a | t1  t 2 | (t1  t 2 )2  4 on directrix.
Diameter
 a (t1  t 2 )2  4t1 t 2 (t1  t 2 )2  4
The locus of mid points of system of parallel chords w.r.t
parabola is called its diameter w.r.t that system of parallel
y 12 2(y12  2ax 2 ) y12 chords. Hence, w.r.t particular system of parallel chords
| AB |  a  4
a2 a2 a2 there is unique diameter. And so for any parabola there
exits infinite diameters.
a  y12  4ax1 y12  4a 2 Equation of Diameter
| AB |  2
a The equation of the diameter bisecting chords of slope m
2a
(s11 ) (y12  4a 2 ) of the parabola y2 = 4ax is y  .
| AB |  m
| a|
Proof
Director Circle Let y = mx + c be a system of parallel chords of the
Locus of a point from where two perpendicular tangents parabola y2 = 4ax.
are drawn to the parabola. Here m is a constant and c is a variable.
2
Let y = 4ax be the parabola. The line y = mx + c meets the parabola y2 = 4ax in the
points say, P and Q whose ordinates (say y1 and y2) are the
Q roots of the equation

P y
P
y=mx+c
R
P(h,k)
x
x’ O
Let from external point P(x1, y1) two perpendicular tangents
PA and PB are drawn to parabola.
Q
2 2
Let A(at , 2at1 ) and B(at , 2at 2 )
1 2 y’

x 1  at1 t 2 
  ...(1) yc
y1  a(t1  t 2 ) y 2  4a    my 2  4ay  4ac  0
 m 
equation of tangent PA and PB are t1 y  x  at12
4a
2
y1  y 2 
t 2 y  x  at 2
m

1 1 y1  y 2
slope of PA  and Slope of PB  Let (h, k) be the mid-point of PQ. Then, k 
t1 t2 2
2a
As two tangents are perpendicular, so k
m
Slope of PA  Slope of PB = –1
2a
 t1 t 2  1 Thus, the locus of (h, k) is y  is required equation of
m
Hence, from equation (1) x1 = –a diameter w.r.t system of chords of slope m.

Therefore, the co-ordinates of P are P(a,a(t1  t 2 )) , for Hence, diameter of parabola is parallel to the axis of

3.57
Coordinate Geometry

parabola and the point where it meets the parabola is known


2at 2  2at1 2
as extremity of the diameter. m 2 2
m
at 2  at1 (t 2  t1 )
Corollary 1
Extremity of diameter can be obtained by solving parabola  Equation of diameter is

2a y  2a / m  y  a(t1  t 2 ) ...(1)
y2 = 4ax with diameter y  . Co-ordinates of extremity
m
Now tangents at P(at12 , 2at1 ) and Q(at 22 , 2at 2 ) meet at a
 a 2a 
of diameter are  2 ,  point [at1 t 2 ,a(t1  t 2 )] which lies on (1).
m m 
Alternative Method
Corollary 2
Let equation of any chord PQ be y = mx + c
The tangent at the extremity of a diameter of a parabola is
parallel to the system of parallel chords which it bisects. If tangents at P and Q meet at R(x1, y1), then PQ is the
chord of contact with respect to R(x1, y1).
Proof
Let y = mx + c (c variable) represent the system of parallel 2a 2ax 1
Equation of PQ is yy 1  2a(x  x1 ) or y  x
y1 y1
chords, then the equation of the diameter of y 2  4ax is
which is identical to y = mx + c
2a
y
m 2a 2a
m or y 1 
y1 m
y=mx+c 2a
a 2a Hence locus of R(x1, y1) is y  which bisects the chord
m2 m m
D PQ.
Corollary 4
Equation of parabola whose axes are any diameter and
 a 2a  tangent at the extremity of diameter (i.e. oblique axes)
2
The diameter meets the parabola y  4ax at  2 , 
m m  Proof
Let PVX be the diameter and PY the tangent at P meeting
 a 2a  a the axis in T.
Tangent to the parabola at  2 ,  is y  mx 
m m  m
Take any point Q on the curve, and draw QM perpendicular
which is parallel to y  mx  c . to the axis meeting the diameter PV in L.
Corollary 3
Y
Tangents at the ends of any chord always intersect on
corresponding diameter. Q
P 
X
V L
Proof 
T A SN M
If extremities of the chord be P(at12 , 2at1 ) and Q(at 22 , 2at 2 )
then its slope
Let PV be x and VQ be y.
)
y=mx+c a
2
t ,2at 1
P ( 1
Draw PN perpendicular to the axis of the curve, and let
  YPX  PTM .
R D Then
(x1,y1) y= 2a
m 4AS  AN  PN 2  NT 2 tan 2   4AN 2  tan2  .
Q
(at22,2at2)  AN  AS  cot 2   a cot 2 ,

3.58
Parabola

and PN  4AS  AN  2a cot . note


A line segment from a point P on the parabola and
Now QM 2  4AS  AM  4   AM ...(1) parallel to system of parallel chords is called ordinate
to the diameter bisecting the system of parallel chords
Also, and the chords are called its double ordinate.
QM  NP  LQ  2a cot   VQ sin  Pole and Polar
 2a cot   y sin , Let P be any point lying on the plane containing parabola,
suppose chords (or secants) AB and A1 B1 are drawn through
and AM  AN  PV  VL  a cot 2   x  y cos  P. Then the locus of the point of intersection of tangents at
the extremities of these chords is called polar of point P
Substituting these values in (1), we have
w.r.t parabola and point P is known as pole of the polar.
(2a cot   y sin )2  4a(a cot 2   x  y cos ) , Some important points related to pole and polar

i.e. y 2 sin 2   4ax 1) If point P (i. e. pole) is inside the parabola then polar of
P is outside the parabola.
The required equation is therefore y 2  4px ...(2)
T(h,k) A’
A Pole
a
where P  a(1  cot 2 )  a  AN  SP The equation
sin 2  P(x1,y1)
to the parabola referred to the above axes is therefore of B’
the same form as its equation referred to the rectangular T’
Polar B
axes.

The equation (2) states that QV 2  4SP  PV . 2) If pole is outside the parabola then polar intersect the
parabola at two real distinct points.
Corollary 5
If diameter through any point P of the parabola meets any T(h,k)
chord at A and tangents at the extremity of chord at B and A
Pole
C. then PA 2  PB  PC
P
Proof (x1,y1)
A’
Let the parabola be y2 = 4ax and diameter be axis of B
parabola i.e. x-axis. T’

Let QR be any chord, whose extremities are Q(at12 , 2at1 ) 3) If pole is on the surface of parabola then polar co-incides
with tangent at pole.
and R(at 22 , 2at 2 ) .
4) As in the parabola two tangents can never be parallel, so
Equation of QR  y(t1  t 2 )  2x  2at 1 t 2  0 ...(1) for every pole there exists a unique polar.
Equation of polar if pole is given
Equation of tangent at Q  t1 y  x  at12 ...(2)
Let equation of parabola be y2 = 4ax and given pole be
P(x1, y1). Let AB be any chord through P, and tangents at
Equation of tangent at R  t 2 y  x  at 22 ...(3)
extremities of chord intersect at T(h, k). So, locus of T is
Now, point of intersection of diameter (y = 0) and QR is polar of P.
A(at1 t 2 ,0) Now, for point T, the line AB will act as chord of contact.
Equation of AB is
Similarly, point of intersection of diameter with tangents at
Q & R are B(at12 ,0) and C(at 22 ,0) ky  2a(x  h) ...(1)
As AB passess through P(x1, y1) hence P satisfies line AB
 PA  at1 t 2 , PB  at12 & PC  at 22
 ky1  2a(x1  h) ...(2)
2
 PA  PB  PC Now to determine locus of T replace (h,k) by (x, y) we get

3.59
Coordinate Geometry

yy1  2a(x  x1 ) therefore, P is the middle point of TV.

i.e. S1 = 0 is required polar of point P w.r.t parabola

note V P T
As polar of point P is unique straight line, so locus of
T and T ' in previous figures is same straight line,
hence we can say that line joining T and T ' is polar of
P.  y2 
Also the tangent at P is yy1  2a  x  1  , which is parallel
Corollary  4a 
If pole is at focus then directrix is its polar. to (1).
Proof Hence the polar of T is parallel to the tangent at P.
Let equation of parabola be y2 = 4ax To draw the polar of T we therefore draw a line through T,
parallel to the axis, to meet the curve in P and produce it to
whose polar w.r.t pole P(x1, y1) is yy1 = 2a(x + x1) ...(1) V so that TP = PV; a line through V parallel to the tangent
Now, as focus is pole, hence to determine polar of focus, (x1, y1) at P is the the required polar.
should be replaced by (a, 0) in equation (1). Pole if polar is given
Therefore polar of focus is y  0  2a(x  a) Let the equation of parabola be y2 = 4ax ...(1)

x  a  0 and given polar w.r.t parabola is lx + my + n = 0 ...(2)


Let P(x1, y1) be pole of given line. Polar of P w.r.t parabola
which is directrix of parabola.
is yy1  2a(x  x1 )
Geometrical construction for the polar of a point
(x1, y1) :  2ax  yy1  2ax1  0 ...(3)
Let T be the point (x1, y1), so that its polar is
As polar w.r.t pole is unique, so equation (2) and (3) are
yy1 = 2a (x + x1) ...(1) identical. On comparing these two equations.
Through T draw a straight line parallel to the axis ; its 2a  y1 2ax1
equation is therefore y = y1 ...(2)  
l m n
R
T  n 2am 
P we get (x 1 , y1 )   ,  is required pole w.r.t
V l l 
Q
lx + my + n = 0
note
No need to remember the above result, to determine
Let this straight line meet the polar in V and the curve in P. pole of given polar we have to follow the same
procedure
The coordinates of V, which is the intersection of (1) and
(2), are therefore Corollary 1

 y12   y1 y 2 y1  y 2 
Pole of chord joining (x1, y1) and (x2, y2) is  ,

2a
 x 1 , y1  ...(3)  4a 2 
 
Proof
Also P is the point on the curve whose ordinate is y1, and
Equation of chord joining A(x1, y1) and B(x2, y2) of the
 y12  parabola y2 = 4ax is S1 + S2 = S12
whose coordinates are therefore  4a , y1 
 
 yy1  2a(x  x 1 )  yy 2  2a(x  x 2 )
abscissa of T  abscissa of V  y 1 y 2  2a(x1  x 2 )
Since abscissa of P  ,
2

3.60
Parabola

 y(y1  y 2 )  4ax  y1 y 2  0 ...(1)  x1 y 2  x 2 y1 2a(x 2  x 2 ) 


On solving we get (h,k)   , 
Let pole of chord AB is P(h, k). Then polar of P is given by  y 2  y1 y 2  y1 

...(2) which is same as point of intersection of polars of Q and R.


ky  2ax  2ah  0
Hence, point of intersection of polars of Q and R is pole of
As polar w.r.t a pole is unique, so equation (1) and (2) are QR.
identical. On comparing the two equations, we get Corollary 3
k 2a 2ah If three poles P, Q and R w.r.t parabola are collinear then
 
y1  y 2 4a  y1 y 2 their polars are concurrent and vice versa
Proof
 y y y  y2 
We get (h,k)   1 2 , 1 as the required co-ordinates Let P(x1,y1), Q(x2, y2) and R(x3, y3) are three poles w.r.t
 4a 2 
parabola y2 = 4ax.
of pole of chord AB.
As three poles are collinear, so area of triangle PQR is zero.
In parametric form (x 1 , y1 )  (at12 , 2at 1 ) and i.e.
(x 2 , y 2 )  (at 22 , 2at 2 ) then co-ordinates of pole are
1 x1 y1
(h, k)  (at1 t 2 ,a(t1  t 2 )) 1 x2 y2  0
...(1)
Hence, we can say that pole of chord joining A and B is 1 x3 y3
point of intersection of tangents at A and B.
Corollary 2 Now, polar of P  yy1  2ax  2ax1  0 ...(2)

Point of intersection of the polars of two points Q and R Polar of Q  yy 2  2ax  2ax 2  0 ...(3)
w.r.t parabola y2 = 4ax is pole of QR.
Proof Polar of R  yy 3  2ax  2ax 3  0 ...(4)

Let Q(x1, y1) and R(x2, y2) be two points, such that polars of Area of triangle whose side equations are polars of P, Q
Q & R intersect at P(h, k). and R is

Polar of Q  yy1  2a(x  x1 )  0 ...(1) y1 2a x1


2
1 x1 y1
2

1 4a 2
Polar of R  yy 2  2a(x  x 2 )  0 ...(2)  y2 2a x 2  1 x2 y2
2k 2k
y3 2a x 3 1 x3 y3
Point of intersection of polar of Q & R is
From equation (1)
 x y  y1 x 2 2a(x1  x 2 ) 
P 1 2 ,   (h, k)
 y 2  y1 y 2  y1  4a 2
Area of   0  0
2k
Now, polar of P is ky  2a(x  h)  0 ...(3)
 Three polars are concurrent.
Equation of line joining Q & R is Conjugate Points
y 2  y1 If two points P and Q w.r.t parabola are such that polar of
y  y1  (x  x 1 )
x 2  x1 P passess through Q and polar of Q passess through P then
P and Q are said to be conjugate points w.r.t parabola.
 x(y 2  y1 )  y(x 2  x1 )  x 2 y1  x1 y 2  0 ...(4) Let P(x1, y1) and Q(x2, y2) are two points w.r.t parabola
y2 = 4ax. Then
As QR is polar of P so equation (3) and (4) are identical on
comparing equation (3) & (4) Polar of P  yy1  2a(x  x1 )  0 ...(1)

2a k 2ah Polar of Q  yy 2  2a(x  x 2 )  0 ...(2)


 
y 2  y1 (x 2  x 1 ) x 2 y1  x 1 y 2

3.61
Coordinate Geometry

As polar of P passess through Q, so co-ordinates of Q satisfy Let A(at 12 , 2at1 ) and B(at 22 , 2at 2 )
polar of P and co-ordinates of P satisfy polar of Q.
On putting co-ordinates of P in polar of Q, we get
P(x1,y1)
y1 y 2  2a(x1  x 2 )  0 ...(3)
A
i.e. S12 = 0
Same conditions we get if we put co-ordinates of Q in polar Q(h,k)
of P.
Hence, S12 = 0 is the required condition for two points to B
be conjugate points.
Let Q is a point on secant PAB, such that P and Q are
note harmonic conjugate of A and B.
If P and Q are conjugate points then Q lies on polar of Let P and Q divide AB in  : 1 ratio externally and internally
P. So, Q can be any where on polar of P. Hence for respectively.
fixed point P their can be infinite pair of conjugate
points. at 22  at12 2at 2  2at1
x1  , y1 
Conjugate Lines  1  1
If L1 and L2 are two lines such that pole of L1 lies on L2 and
pole of L2 lies on L1 then the two lines are said to be conjugate at 22  at12 2at 2  2at1
h , k
lines.  1  1
(OR) Polars of conjugate points are conjugate lines. On eliminating  from the above four equation, we have
Theorem ky1  2a(h  x1 )
The lines l1 x  m1 y  n1  0 and l 2 x  m2 y  n 2  0 are
Hence locus of Q is yy1  2a(x  x1 )  0
the conjugate lines w.r.to the parabola y 2 = 4ax if
l1 n 2  l2 n1  2am1 m2 . which is polar of P.

Proof Hence, if P and Q are harmonic conjugates w.r.t chord AB


of parabola then P and Q are also conjugate points w.r.t
Given lines l1 x  m1 y  n1  0 ...(1) same parabola. And also locus of Q is polar of P and vice
versa.
and l 2 x  m2 y  n 2  0 Solved Examples
The pole of line (1) w.r.t to the parabola Example 77
If the chord of contact of tangents from a point P to
the parabola y 2 = 4ax touches the parabola
 n 2am1 
y 2  4ax is P   1 ,  x2 = 4by. Find locus of P
 l1 l1  Solution
P lies on (2) Let P(h,k) be a point. Then the chord of contact of
tangents from P to y2 = 4ax is
n   2am1  ky = 2a (x + h) ...(i)
 l2  1   m 2    n2  0 This touches the parabola x2 = 4by. So, it should be
 l1   l1 
of the form
 l1 n 2  l2 n1  2am1 m2 b
x  my  ...(ii)
Is the required condition for congugate lines. m
Equation (i) can be re-written as
Alternative Definition of Polar
k
Polar of any point P w.r.t any conic is locus of harmonic x yh ...(iii)
2a
conjugate of P w.r.t two points in which any line through P Since (ii) and (iii) represent the same line, we get
cut the conic.
k b
Let P(x1, y1) is any point through which secant PAB is drawn  m and  h
2a m
to parabola y2 = 4ax,
Eliminating m from these two equations, we get

3.62
Parabola

2ab = – hk
Hence the locus P is y 2 (x  2a)  4a 3  0
Hence, the locus of P(h, k) is
xy = –2ab, which is a hyperbola. Example 80
Find the locus of the poles of the chords of the parabola
Example 78
y2 = 4ax which subtends a constant angle at the vertex.
Show that the locus of the middle points of normal
chords of the parabola y2 = 4ax is y4 – 2a(x–2a)y3 + Solution
8a4 = 0. The given parabola is y2 = 4ax ...(1)
Solution Let (x1 , y1) be the pole of a chord of y2 = 4ax
Then the equation of the chord of the polar of (x1, y1)
If (x1 , y1 ) is the middle point of the chord of the
with respect to (1) is
parabola. Then T = S1
yy1  2a(x  x1 )
yy1  2a(x  x 1 )  y12  4ax 1
yy1  2ax
 2ax  yy1  y12  2ax 1  0 ...(i) 1 ...(2)
2ax 1
Equation of normal of the parabola at ‘t’ is
Joint equation of the lines joining the points of
y  tx  2at  at 3 intersection of (1) and (2) with the origin can be
 tx  y  2at  at 3  0 ...(ii) obtained by homogenising (1) by help of (2).We get
Equation (i) and (ii) are identical then  yy  2ax 
y 2  4ax  1 
 2ax 1 
2
2a y1 y  2ax 1
1
 
t 1 2at  at 3
4ax 2  2y1xy  x 1y 2  0 ...(3)
from first two relation.
As chord subtends an angle  at the vertex then
2a
t ...(iii) Y
y1
A
from first & last relation.
yy1=2a(x+x1)
2a(2a  at 2 )  y12  2ax 1 ...(iv) X’ X
O 
Substituting the value of t from (iii) in (iv) then
 4a 3  B
2a  2a  2   y 12  2ax 1
 y1  Y’
 y14  2a(x 1  2a)y12  8a 4  0 angle AOB = 
Hence locus of the middle point (x 1 , y 1 ) is 2
2 h2  ab 2 (y1  4ax1 )
y 4  2a(x  2a)y 2  8a 4  0 tan   
|a  b| |4a  x 1 |
Example 79
Find locus of the poles of the chord which is normal (4a  x 1 )2 tan 2   4(y12  4ax1 )
to the parabola y2 = 4ax. The required locus of (x1, y1) is
Solution
Let P(x1, y1) be the pole of normal chord (4a  x)2 tan 2   4(y 2  4ax)
Example 81
y  mx  2am  am2 Polar of any point P w.r.t the circle x2 + y2 = a2 touches
with respect to y2 = 4ax the parabola y2 = 4ax. Find locus of P.
Equation to pole of P(x1,y1) is Solution
yy1  2a(x  x1 )  0 Let P (x1, y1)
But equations (1) & (3) represents same line its polar w.r.t the circle x2 + y2 = a2 is
xx1 + yy1 = a2
1 m 2am  am3
   x1 a2
y1 2a 2ax1 y x ...(1)
y1 y1
2a 2a  am 2 Since the polar is tangent to y2 = 4ax, equation of
m and 1 
y1 x1 tangent to parabola is
a
 4a 2  y  mx  ...(2)
On eliminating m we get x 1  2a  a  2   0 m
 y1  equation (1) and (2) are identical, on comparing and
 y12  x1  2a   4a 3  0

3.63
Coordinate Geometry

So, one normal is always the x-axis.


a2 ay
eliminating m we get  1 For the other two normals to be perpendicular, we
y1 x1 must have m1 m2 = –1
y12  ax 1  0  locus is y2 + ax = 0  2(k  1)   2(k  1)  1
Example 82
3
Prove that the normals at the points where the straight  2(k  1)  1  k 
line lx + my = 1 meets parabola y2 = 4ax meet on 2
Example 84
 4am 2 am  Tangent is drawn at any point (x1 y1) on the parabola
the normal at the point  l 2 , l  of the parabola.
  y2 = 4ax. Now tangents are drawn from any point on
Solution these tangents to the circle x2 + y2 = a2 such that all
the chords of contact pass through a fixed point (x2,
Let P(at12 , 2at 1 ) and Q(at 22 , 2at 2 ) be the points on the 2
parabola where it is met by the line lx + my = 1. x  y 
y2). Prove that 4  1    1   0
Then,  x 2   y2 
lat 12  2mat1  1  0 and lat 22  2mat 2  1  0 Solution
The equation of the tangent to y2 = 4ax (x1, y1) is
 t1 , t 2 area the roots of the equation
yy1  2a(x  x1 )
lat 2  2mat  1  0
 2a(h  x 1 ) 
2m 1 Let P  h,  be a variable point on this
 t1  t 2  and t1t 2   y1
l la  
The normals at P and Q intersect at tangent. Then, the chord of contact of tangents drawn
from P to the circle x2 + y2 = a2 is
R  2a  a(t12  t 22  t1 t 2 ), at 1t 2 (t 1  t 2 ) 
 h  x1  2
Now, hx  2a  y  a
 y 1 
4m 2 1
t12  t 22  t1t 2  (t1  t 2 )2  t1t 2  
l2 la  h(xy1  2ay)  2ax1 y  a 2 y1  0

 4am 2 1 2m   (2ax 1 y  a 2 y1 )  h(xy1  2ay)  0


 Coordinates of R are  2a  l 2  l , l 2  This equation represents a family of lines passing
 
through the point of intersection of the lines.
 4am 2 4am  2ax1 y  a 2 y 1  0 and xy1  2ay  0
The equation of the normal at  l 2 , l  is
  The coordinates of the point of intersection of these

4am 2m  4am 2   a 2 ay1 


y  x  2  lines are  x , 2x 
l l  l   1 1 

Clearly, R satisfies this equation. Thus, all the chords of contact of tangents pass through
Example 83  a 2 ay1 
Three normals are drawn from (2k, 0) to the parabola the point  x , 2x 
y2 = 4x. Show that k must be greater than 1. One  1 1 

normal is always the x-axis. Find k for which the other


two normals are perpendicular to each other. a 2 ay1
 x2  and y 2  2x
Solution x1 1

The equation of any normal to y2 = 4x is 2 2


y = mx – 2m – m3 ...(i) x  y   x 2   2x 
 4  1    1   4  21    1   0
If it passes through (2k, 0), then  x2   y2   a   a 
m3  2m(1  k)  0 Example 85
From a point A common tangents are drawn to the
 m  0,m   2(k  1)
For three normals to be real and distinct, we must 2 2 a2
circle x  y  and parabola y2 = 4ax. Find the
have 2(k  1)  0  k  1 2
Putting m = 0 in (i), we get area of the quadrilateral formed by the common
y = 0. tangents, the chord of contact of the circle and the

3.64
Parabola

chord of contact of the parabola. by the directrix.


Solution Solution
Let the equation of the parabola be y2 = 4ax and the
a
Let y  mx  be a tangent to the parabola y2 = co-ordinates of the point P be (at12 , 2at1 ) and Q be
m
4ax. If it touches the circle (at 22 , 2at 2 ) .
2
 a  The equation of the line PQ the normal at P, will be
x 2  y2    , then
 2 y   t1x  2at1  at 13 ...(1)
2 2 If T  (, ) be the pole of PQ, then PQ will be the
a  a  2
    (m  1) [Using : c2 = a2(m2+1)] polar of T. Hence the equation of PQ must be
m  2
y  2a(x  ) . ...(2)
 2  m 2 (m 2  1)  m 4  m 2  2  0 As (1) and (2) represent the same line PQ, so the
 (m 2  2)(m 2  1)  0  m 2  1  0  m  1 . coefficients must be proportional, therefore,
Hence, the equations of the common tangents are  2a 2a
  ...(3)
y  x  a and y  x  a . 1  t 1 2at1  at 12
These two intersect at A(–a,0) Let PR be the focal chord through P, where the co-
The equation of the chord of contact of tangents drawn
 a 2a 
a2 ordinates of R are  2 ,  
from A(–a,0) to the circle x  y 
2 2
is  t1 t1 
2
The equation of the diameter through R will be
a2 a 2a
ax  0y  x y
2 2 ...(4)
t1
This is the equation of PQ.
By relation no (3),
Similarly, RS is the chord of contact of tangents drawn
from A(– a,0) to the parabola y 2 = 4ax. So, its 2a
equation is   (2a  at 12 ) and    t
1
0y = 2a ( x – a)
xa. Hence the co-ordinates of T are (2a  at12 , 2a / t 1 ) .
As these co-ordinates satisfy (4), T lies on (4) i.e. the
y diameter through R.
L The equation of the directrix of the parabola
x+a=0
R Abscissa of the mid-point of PT is
P
(-a,0)A M x 1 2
x’ 2
 at1  2a  at12   a .
Q As the abscissa satisfies (5), the equation of the
S
directrix, the line PT is bisected by the directrix.
a2 Example 87
x2+y2= 2
y’ Through each point of the straight line x = my + h a
chord is drawn to the parabola y2 = 4ax which is
bisected at that point. Prove that it always touches
We have to find the area of the trapezium PQSR. The
coordinates of P, Q, R and S are marked in figure. the parabola (y + 2am)2 = 8a(x – h).
Solution
1 Let the equation of the parabola be y2 = 4ax. If
Required are  (PQ  RS)  LM
2 (x1, y1) be the co-ordinates of the middle point of any
chord, then equation of that chord is
1 3a 15a 2
 (a  4a)  sq.units  sq.units yy1  2a  x  x1   y1 2  4ax1
2 2 4
Example 86 y1(y  y1 )  2a(x  x1 ) ...(1)
The normal at a point P of a parabola meets the Again as the point (x1, y1) lies on the line x = my + h
curve again in Q and T is the pole of PQ. show that T (given), so x1 = my1 + h.
lies on the diameter passing through the other end of Substituting the values of x1 in (1), we have
the focal chord passing through P, and PT is bisected

3.65
Coordinate Geometry

y1(y  y1 )  2a(x  my1  h) Solution


Let P(at12 , 2at) and Q(at 22 , 2at 2 ) be two points on the
2a 2a
or (y  2am)  y (x  h)  2a / y ...(2) parabola, then the equation of the chord PQ is
1 1

Changing the origin to (–2am, h) and putting (at12,2at1)


2a/y1 = M, equation (2) becomes Y = MX + 2a/M P
which is always tangent to the parabola
Y2 = 8aX. ...(3)
2c (h,k)
As referred to the original (3) becomes
(y + 2am)2 = 8a (x – h).
Hence (2) always touches the parabola.
Example 88 Q (at 2,2at )
2 2
Two parabolas have the same axis and tangents are
drawn to the second from points on the first, prove
2at 2  2at 1
that the locus of the middle points of the chords of y  2at1  (x  at 12 )
contact with the second parabola lie on a fixed at 22  at 12
parabola. 2x  (t1  t 2 )  2at1t 2  0 ...(1)
Solution
Let (h, k) be the pole of PQ
Suppose y2 = 4ax ...(1)
So, equation of polar of y2 = 4ax wrt (h, k) is
and y = 4b (x + c) ...(2)
ky = 2a(x + h)
be the equations of the parabolas and let PQ be the
2ax – ky + 2ah = 0 ...(2)
polar, the co-ordinates of ends P and Q to be (at12 , 2at1 ) Equation (1) and (2) are identical and comparing them
and (at 22 , 2at 2 ) respectively; then the equation of polar 2 t 1  t 2 t1 t 2
 
PQ is 2a k h
(t1  t 2 )y  2x  2at1t 2 ...(3) h  at1 t 2 and k  a(t1  t 2 )
and if (x1, y1) be the pole, then polar of (x1,y1) w.r.t (1) (PQ)2  [a(t 12  t 22 )]2  [2a(t 1  t 2 )]2
is
yy1  2a(x  x1 ) ...(4) 4c 2  a 2 (t1  t 2 )2 {t 1  t 2 )2  4}
Since (3) and (4) represent the same line, the  4c 2  a 2 {(t 1  t 2 )2  4t 1t 2 }{(t1  t 2 )2  4}
coefficient will be proportional. So comparing the
coefficients of x, y and constant term, we get  k 2 4h   k 2 
 4c 2  a 2  2    2  4
y1 2a 2ax1  a a  a 
 
t1  t 2 2 2at1t 2
 4a 2c 2  (k 2  4ah)(k 2  4a 2 )
or x1  at1t 2 and y1  a(t1  t 2 ) Hence the required locus of (h, k) is
Hence the co-ordinates of the pole are {at1t2, a(t1 + t2)}. (y 2– 4ax)(y2 + 4a2) = 4a2c2
As this point lies on (2), we have Example 90
If a triangle is such that its vertices are the poles of the
2b  2a 2 t 1t 2  2ac  opposite sides w.r.t a parabola, then show that the
a 2 (t1  t 2 )2  4b  at1t 2  c   ...(5) lines joining the middle points of the sides of the triangle
a
touch the parabola.
If (h, k) be the co-ordinates of the middle point of the
Solution
chord PQ, then 2h / a  t 12  t 22 and k / a  t1  t 2 Let the given parabola be y2 = 4ax ...(1)
Hence k2 – 2ah = 2at1t2 Let A  (x1, y1 ), B  (x 2 , y 2 ), C  (x 3 , y 3 )
Substituting the values in (5)
Let D, E, F be the middle points of BC, CA and AB
 k 2   2b  2 respectively.
a2  2      k  2ah  2ac  Since BC is the polar of point A w.r.t parabola (1),
a   a  therefore, equation of BC will be
Generalising, we have ay2 = 2b (y2 – 2ax + 2ac) yy1 = 2a(x + x1) ...(2)
which is a fixed parabola. Since line (2) passes through B(x 2 , y 2 )
Example 89
 y 2 y1  2a(x 2  x1 ) ...(3)
In the parabola y2 = 4ax, prove that the locus of the
poles of all chords of constant length 2c is x 1  x 2 y1  y 2 
also F   , 
(y 2  4ax)(y 2  ka 2 )  4a 2c 2  2 2 

3.66
Parabola

2a  
Slope of line BC  or tan 3  tan    
y1 2 
since FE || BC, therefore, equation of EF will be Solving, we have

y1  y 2 2a  x1  x 2         
y 3      ,         or  2      
 x    2    2    2 
2 y1  2 
If 1, 2 and 3 be three values of , then
 y  y2 
y1  y  1   2ax  a(x1  x 2 ) 1   1 
 2  1      , 2       and
32  3 3  2 
A(x1,y1)
2 1   
3     
F E 3 32 

So 2  1    3  2
B (x2 ,y2) D C (x3,y3) 3
Therefore the three normals are inclined at 600. So
y12  2yy1  4ax  0 ...(4) the angles between the tangents are also 600.
Envelope of line (4) is Therefore these form an equilateral triangle.
4y 2  4  4ax  0 or y 2  4ax Make Concepts Clear 3.5
2
Hence line FE touches parabola y = 4ax. 1. A rod of length 2L slides on y2 = 4x. Find locus of
Similarly, we can show that DE and DF touch the mid point of rod.
parabola y2 = 4ax. 2. Find the mid-point of the chord intercepted on the
Example 91 line 4x – 3y + 4 = 0 by the parabola y2 = 8x.
Show that three circle can be drawn to touch a
3. Show that the locus of the poles of the tangents to
parabola and also to touch at the focus a given straight
the parabola y2 = 4ax with respect to the parabola y2
line passing through the focus, and also prove that
= 4bx is the parabola, ay2 = 4b2 x.
the tangents at the point of contact with the parabola
form an equilateral triangle. 4. Find the locus of middle point of the chord of the
Solution parabola y2 = 4ax whose slope is ‘m’.
The equation to the circle which passes through the 5. Show that the locus of poles of chords of parabola y2
focus (a, 0) and also touches the parabola y2 = 4ax = 4ax which are at a constant distance “a” from the
at (at2,2at) is focus is y2 = 4x(2a + x)
x2 + y2 – ax (3t2 + 1) – ay(3t – t3) + 3a2t2 = 0 6. Find the locus of the mid points of the chords of the
and equation of any line through (a, 0) is parabola y2 = 4ax, subtend a right angle at the vertex
sin  of the parabola.
y (x  a) i.e x sin   y cos   a sin  and
cos  7. From a variable point on the tangent at the vertex of
equation of perpendicular line will be a parabola y2 = 4ax, a perpendicular is drawn to its
chord of contact. Show that these variable
x cos   y sin   a cos  ...(1) perpendicular lines pass through a fixed point on the
Co-ordinates of the centre of the circle are axis of the parabola.
[a(3t 2  1) / 2, a(3t  t 3 ) / 2] . 8. Show that if tangents be drawn from any point on the
Since (1) passes through the centre, we have line x + 4a = 0 to the parabola y2 = 4ax, prove that
their chord of contact will subtend a right angle at the
a  3t 2  1 a vertex.
2
cos  
2
 3t  t 3  sin   a cos  9. Find locus of middle point of the chord of parabola y2
= 4ax which are such that the normals at their
 (3t  t 3 )sin   (1  3t 2)cos  extremeties meet on parabola.
3t  t 3 10. Find locus of middle point of the chord of parabola y2
 cot  
1  3t 2 = 4ax which subtends a constant angle  at the vertex.
Putting t  tan , we get 11. Find the locus of the poles of tangents to the parabola
y2 = 4ax with respect to the circle x2 + y2 – 2ax = 0.
3 tan   tan3  12. If the perpendicular drawn from P on the polar of P
cot    tan 3
1  3 tan2  with respect to the parabola y2 = 4ax touches the
parabola x2 = 4ax. Find locus of P.

3.67
Coordinate Geometry

13. Perpendiculars are drawn from points on the tangent


1
at the vertex on their polars with respect to the  m1  Slope of PR 
parabola. Prove that the locus of the feet of these t1
perpendiculars is a circle. Find its centre and radius.
1
14. If from the vertex of a parabola a pair of chords be m 2  Slope of QR 
drawn at right angles to one another and with these t2
chords as adjacent sides a rectangle be made, prove
that the locus of the further angle of the rectangle is 1
 m1 m 2   1  t1t 2  1
the parabola y2 = 4a (x – 8a). t1 t 2
Hence, the tangents at P and Q intersect at right-angles on
Answer Key the directrix.
2a 2) The portion of the tangents to a parabola cut off between
2.(5/4, 3) 4. y  6. y 2  2ax  8a 2  0
m the directrix and the curve subtends right angle at the focus.
9.y2 = 2a(x+a) Proof
10. (8a 2  y 2  2ax )2  tan 2   16a 2 (4ax  y 2 )
Let P(at2, 2at) be any point on the parabola y2 = 4ax such
2 2 2
11. x  y  ax 12. 2ax  by  4a  0 that the tangent at P cuts the directrix at Q.
13.(a, 0), a 14.y2 = 4a(y – 8a).
Solutions are on Page No. 3.114 y 2
ty=x+at
Q R P
x
3.6 Properties of Parabola x’ S

1) The tangents at the extremities of a focal chord of


x=-a
parabola intersect at right angles on the directrix. y’
Proof
Equation of tangent at P is ty = x+at2 and equation of
2 2
Let P(at , 2at1 ) and Q(at , 2at 2 ) be the extremities of a
1 2
directrix is x+a=0. On solving these two equations we get
focal chord of the parabola y2 = 4ax.  a
coordinates of Q are  a,at   .
 t
y
2t
P Now, m1  Slope of SP  2
t 1
x’ x
O S(a,0)
R a
Q at  0  1  t2 
m2 = Slope of SQ  t
x=-a   .
2a  2t 
y’
Clearly, m1m2 = –1.
R  at1 t 2 , a(t1  t 2 ) is the point of intersection of tangents
Hence, PQ subtends a right angle at the focus of the
at P and Q. parabola y2 = 4ax.
Since PQ is a focal chord of the parabola. Therefore, 3) The tangent at any point on the parabola bisects the
t1 t 2  1 angle between the focal distance of the point and the
perpendicular on the directrix from the point.
Thus, the coordinates of R are  a,a(t1  t 2 ) . Clearly, it Proof
lies on the directrix. Let the tangent at P(at2, 2at) to the parabola y2 = 4ax
meets the axis of the parabola i.e., x–axis or y = 0 at T.
The equations of tangents PR and PQ are t1 y  x  at12
The equation of tangent to the parabola y2 = 4ax at P is
and t 2 y  x  at 22 ty  x  at 2

3.68
Parabola

y 2at12 t 2  (t12  1)a(t1  t 2 )  2at 1


p1   a | t1  t 2 |
P (t12  1)2  4t12
M
x
T Z V S similarly p 2  a | t 2  t1 |    

ii) SP  SQ  (a  at12 )(a  at 22 )  a 2 (1  t12 )(1  t 22 )


Directrix
Also (ST)2  a 2 (t1 t 2  1)2  a 2 (t1  t 2 )2
For co-ordinates of T solve it with y = 0  a 2 [t12 t 22  1  t 22  t 22 ]
T(at 2 ,0)
 a 2 (1  t12 )(1  t 22 )
ST  SV  VT  a  at 2  a(1  t 2 )
Hence (ST)2= (ST)2  SP  SQ
Also SP  PM  a  at 2  a(1  t 2 )
This is conclusive that product of the focal radii of two
points P and Q is equal to the square of the distance of
 SP  ST,i.e., STP  SPT
focus from the point of intersection of the tangents drawn
But STP  MPT (alternate opposite angles) at P and Q.

STP  MPT ST SQ
(iii) again  and .
SP ST
And tangent at P bisects angle SPM.
4) If tangents at P and Q meet at T, then hence the two triangles SPT and SQT are similar.

i) TP and TQ will subtends equal angle at focus S. 5) If N is foot of perpendicular from focus S on tangent at
point P to the parabola then N lies on tangent at vertex and
ii) ST 2  SP  SQ SN 2  SA  SP (where A is vertex).
iii) The triangles STP and STQ are similar Proof
Proof Equation of tangent at P(at 2 , 2at) is given by
To prove that , it will be sufficient to prove that ‘T’ lies
x  yt  at 2  0 ...(i)
on the angle bisector of the angle  PSQ i.e. perpendicular
distance of ‘T’ from the line SP is equal to the perpendicular foot of perpendicular drawn from focus S(a, 0) to (i) is
distance of T from SQ. xa y0 (a  0  at 2 )
given by,   2
1 t (1)  ( t)2
y
y
M2 T Q P
M
P N
M1 x
x A S
S

Directrix

y
 x a   a
t
2at1
Equation of SP is y  (x  a)
at12  a  x  0;y  at
 Co-ordinate of N are (0, at) which lies on y-axis i.e,
2t1 x  (t12  1)y  2ab1  0 tangent at vertex.

3.69
Coordinate Geometry

Also SP  PM  a(1  t 2 ) ; Proof


Let the equation of parabola be y 2 = 4ax. Let
SA  a and SN  a 2  a 2 t 2  a 1  t 2 P  (at 2 , 2at) be a point on parabola

 (SN)2  a 2 (1  t 2 )  a.a(1  t 2 )  (SA).(SP)  Equation of tangent to parabola at point P is given by


yt = x + at2 ...(1)
Hence locus of foot of perpendicular from focus on any
tangent to the parabola is tangent at vertex. (1) intersects axis, of parabola, at T(at 2 ,0)
6) Circle drawn on focal chord as diameter touches the
directrix. y
Proof P
Equation of circle for which focal chord acts as diameter is
 a  2a  T x
given by (x  at 2 )  x  2   (y  2at)  y  0
 t   t  L

Solving with x = –a, we get

   2a 
a 2 (1  t 2 ) 1  2   y 2   2at  y  0 Clearly, length of sub tangent = TL = 2at2 = 2 (Abscissa
 t   t 
of point P).
 1  1 8) If tangent and normal at any point P on the parabola
 y 2  2a  t   y  a 2  t 2  2   2a 2  0
 t  t  meet its axis at T and G respectively then ST = SG = SP
where S is focus of parabola.
 1   1 Proof
D  4a 2  t 2  2  2   4a 2  t 2  2   8a 2  0
 t   t  Let P(at2, 2at) be any point on the parabola y2 = 4ax, then
Therefore, the circle touches the directrix x = –a tangent and normal at P(at2, 2at) are ty = x + at2 and
y = – tx + 2at + at2 respectively since tangent and normal
meet its axis in T and G  Co-ordinates of T and G are
(–at2, 0) and (2a + at2, 0) respectively.
A
 SP  PM  a  at 2

R SG  VG  VS  2a  at 2  a  a  at 2
(0,0)
(a,0)
and ST  VS  VT  a  at 2
Hence, SP = SG = ST
B

Directrix P
M
Alternate method
As we know that tangents at the extremities of any focal T Z V S
chord intersect at right angle on directrix. So angle
APB  90 0
Directrix
Hence, the circle which is drawn considering focal chord as
diameter always touches directrix. 9) If the tangent at any point P of a parabola intersects the
7) The length of sub tangent to a parabola at a point P axis of parabola at any point T and M is the foot of
intercepted between the point of contact and axis of perpendicular from point P on directrix and S is focus of
parabola is twice the abscissa of point. the parabola, then the quadrilateral SPMT is rhombus.

3.70
Parabola

Proof  SPT  MPT ( STP  MPT , alternate angles)


2
Let the equation of parabola be y = 4ax .We already proved
and SPN  QPN ( SNP  QPN , alternate angle)
TS = SP = PM
Also MT = Hence the result.
11) Area of triangle formed by three points on a parabola,
( a  at 2 )2  4a 2 t 2  a 2  a 2 t 4  2a 2 t 2  a(1  t) is twice the area of triangle formed by the tangents at these
points.
y Proof

Let A(at12 , 2at),B(at 22 , 2at 2 ) and C(at 23 , 2at 3 ) be three points


M P
Y on the parabola y2 = 4ax. Then,
x Area of ABC | 1 |, where
T V S

at 12 2at1 1 t2
2 1
t1 1
1 2a 2
1  at 22 2at 2 1  t2 t2 1
2 2 2 2
at 3 2at 3 1 t3 t3 1
 M  (a, 2at) and T  (at ,0) 2

and PM  at 2  a  a(1  t 2 ) t12 t1 1


 a 2 t 22  t12 t 2  t1 0  Applying : R 2  R 2  R 
 
 TS  SP  PM  MT t 32  t12 t 3  t1 0  and, R 3  R 3  R1 

 TSPM is a rhombus.
10) Reflection Property of Parabola t12 t1 1
 a 2 (t 2  t1 (t 3  t1 ) t 2  t1 1 0
Tangent PT and normal PN are internal and external bisector
t 3  t1 1 0
of angle SPM

i.e., QPN  SPN and SPT  MPT t 2  t1 1


 a 2 (t 2  t1 )(t 3  t1 )
i.e., any ray of light coming parallel to the axis of parabola t 3  t1 1
then after reflection through parabola it passes through focus
of the parabola and conversely. i.e., ray after passing  a 2 (t 2  t1 )(t 3  t1 )(t 2  t 3 )
through focus becomes parallel to axis of parabola after
reflection through parabola.  a 2 (t1  t 2 )(t 2  t 3 )(t 3  t 1 )

Suppose the tangents at A, B, C taken in pairs intersect at


Y
P, Q and R. Then, coordinates of P, Q and R are
P P(at1 t 2 ,a(t1  t 2 )), Q(at 2 t 3 ,a(t 2  t 3 )) and
M Q
Y
X R(at 3 t1 ,a(t 3  t1 )) , respectively..
Z T O S N
Area of PQR  |  2 |, where
Directrix 2
y =4ax
at1 t 2 a(t1  t 2 ) 1 t1 t 2 t1  t 2 1
1 1 2
 2  at 2 t 3 a(t 2  t 3 ) 1  a t 2 t 3 t 2  t3 1
2 2
Proof at 3 t1 a(t 3  t1 ) 1 t 3 t1 t 3  t1 1
We already proved ST = SP = SN
t1t 2 t1  t 2 1
 SPT  STP and SNP  SPN (angles opposite 1 2
 a t 2 (t 3  1) t 3  t1 0  Applying R 2  R 2  R 3 
2 and R  R  R 
to equal sides of  are equal) t1 (t 3  t 2 ) t 3  t 21 0  3 3 1 

3.71
Coordinate Geometry

Clearly, it lies on x = –a i.e. the directrix of the parabola.


t1 t 2 t1  t 2 1
1 2 13) If tangents at points Q and R intersect at P. If P1, P2, P3
 a (t 3  t1 )(t 3  t 2 ) t 2 1 0
2 are length of perpendiculars from P, Q and R respectively
t3 1 0 on any tangents to parabola, then P2, P1, P3 are in GP.
Proof
1 2
 a (t 3  t1 )(t 3  t 2 )(t 2  t1 )
2 Let the coordinates of Q and R are (at12 , 2at1 ) and
Q(at 22 , 2at 2 ) respectively. Then, the equations of the
1
 a 2 (t1  t 2 )(t 2  t 3 )(t 3  t1 ) tangents at Q and R are
2
t1 y  x  at12
Clearly, 1  2 2

| 1 |  2|  2 | t 2 y  x  at 22

respectively.
 Area of ABC  2(Area of PQR)
12 ) orthocentre of any triangle formed by three tangents to The coordinates of P are  at1 t 2 ,a(t1  t 2 ) .
a parabola lies on directrix.
Let ty  x  at 2 ...(i)
Proof
be any tangent to the parabola y2 = 4ax
2 2 2
Let P(at , 2at1 ),Q(at , 2at 2 ) and R(at , 2at 3 ) be three
1 2 3 Then, p 1 = Length of the perpendicular from
points on the parabola y2 = 4ax. The equations of tangents
at P, Q and R are P(at1 t 2 ,a(t1  t 2 )) on (i)

t1 y  x  at12 ...(i) at1 t 2  a(t1  t 2 )t  at 2 (t  t1 )(t  t 2 )


 a
2
t 1 t2  1
t 2 y  x  at 22 ...(ii)
p2 = Length of the perpendicular from Q(at12 , 2at1 ) on (i)
t 3 y  x  at 32 ...(iii)
2 2 2
respectively.  2at t1  at1  at  a(t  t1 )
 p2    
Suppose these tangents form ABC . The coordinates of  t2  1  t2  1
the vertices are A  at 2 t 3 ,a(t 2  t 3 ) , B  at1 t 3 ,a(t 1  t 3 ) , and,
and C  at1 t 2 ,a(t1  t 2 ) . p3 = Length of the perpendicular from R(at 22 , 2at 2 ) on (i)
The equation of altitude through the vertex A is
2at t 2  at 22  at 2 a(t  t 2 )2
y  a(t 2  t 3 )   t1 (x  at 2 t 3 )  p3  
t2  1 t2  1
or, y  t1 x  a(t 2  t 3  t 1 t 2 t 3 ) ...(iv)
Clearly, p12  p 2 p3
Similarly, the equations of altitudes through B and C are
Hence, p1 is the geometric mean of p2 and p3.
y  t 2 x  a(t 3  t1  t 1 t 2 t 3 ) ...(v)
14) No normal other than axis of parabola can pass through
and, y  t 3 x  a(t1  t 2  t 1 t 2 t 3 ) ...(vi) the focus of parabola.
Proof
Solving (iv) and (v), we get x  a and
y  a(t1  t 2  t 3  t1 t 2 t 3 ) Let the equation of normal to parabola y2 = 4ax be y + xt
= 2at + at3
Thus, the coordinates of the orthocentre of ABC are
If (1) passes through the focus (a, 0) then, 0  at  2at  at 3
 a,a(t1  t 2  t 3  t1 t 2 t 3   at 3  at  0

3.72
Parabola

 at(t 2  1)  0 Solving this equation with y-axis (x = 0) we get


y 2  2aty  a 2 t 2  0  (y  at)2  0
 t  0 or t2 + 1 = 0 (impossible)
Thus we get two repeated roots y = at. So we conclude
 t = 0 is the only possibility that tangent drawn at the vertex of parabola. (i. e., y-axis)
Thus axis is the only normal to parabola passing through touches the circle drawn on focal distance (SP) as diameter.
focus. Aliter :
15) The tangent at one extremity of focal chord of parabola As we know that foot of perpendicular from focus on any
is parallel to normal at other extremity of focal chord. tangent at point P lies on tangent at vertex. Let this foot of
Proof perpendicular be A then angle PAS is right angled and so if
we draw any circle on PS as diameter then it touches tangent
Let the equation of parabola be y2 = 4ax at vertex.
Let PQ be focal chord, then by property of tangents, 17) circle on any focal radii of point P(t) of parabola
tangents drawn at P and Q are perpendicular to each other y2 = 4ax intercepts a chord of length on normal at point P.
and intersect at directrix at R.
Proof
Also PG is normal to parabola at P
Let the normal at P(at2, 2at) cut the axis of parabola at G
 PG  RP and circle at K then PK is the required length of intercept.
 from (i) and (ii) RQ || PG SP = PM = a + at2
i.e., normal at P is || to tangent at Q. As angle in a semi-circle being right angle,

Y
P
P(t) M

90 K
0
R
0
90 A
X T S G
S G

SKP  90 0
16) Circle drawn on focal radius as diameter touches tangent
at vertex Normal at P(at 2 , 2at) is tx  y  2at  at 3  0 ...(1)
Proof SK is perpendicular distance from S(a, 0) to the normal (1)
2
Let equation of parabola be y = 4ax, whose focus is
S(a ,0) and P(t) is any point on parabola, then |at  0  2at  at 3 |
SK   a|t| 1  t2
t2  1

P  In SPK (PK)2 = (SP)2 – (SK)2

A = a2(1 + t2)2 – a2t2(1 + t2) = a2(1 + t2)


C
X PK  a (1  t 2 )
C S
18) Circumcircle of triangle formed by three tangents to the
parabola always passess through focus of parabola.
Proof

Equation of circle with SP as diameter is given by Let P(at12 , 2at1 ),Q(at 22 , 2at 2 ) and R(at 23 , 2at 3 ) be any three
2
(x  a)(x  at )  y(y  2at)  0 points on the parabola y2 = 4ax. The equations of the
tangents at these three points are

3.73
Coordinate Geometry

t1 y  x  at12 Proof

t 2 y  x  at 22 D y
L1
and t 3 y  x  at 32 22a
Suppose these three tangent lines form the sides BC, CA x
and AB of ABC . Then the coordinates of the vertices of T O S G
ABC are
L2
A(at 2 t 3 ,a(t 2  t 3 )),B(at1 t 3 ,a(t1  t 3 ))

and, C(at1 t 2 ,a(t1  t 2 )) respectively.. Let extremities of lotus rectum be L1 and L2 and their co-
ordinates are L1(a, 2a) and L2(a, –2a)
Let x2 + y2 + 2gx + 2fy + c = 0
Equation of tangent at L1  y = x + a ...(1)
be the equation of the circumcircle of ABC .
Equation of tangent at L2  y = – x – a ...(2)
It passes through A, B and C. Therefore,
On solving equation (1) and (2) we get co-ordinates of
2 2 2 2 2
a t t  a (t 2  t 3 )  2ag t 2 t 3  2af(t 2  t 3 )  c  0 ...(i) T(–a, 0)
2 3

Equation of normal at L1  x + y = 3a ...(3)


a 2 t 12 t 23  a 2 (t1  t 3 )2  2ag t1 t 3  2af(t1  t 3 )  c  0 ...(ii)
Equation of normal at L2  x – y = 3a ...(4)
2 2 2 2 2
a t t  a (t1  t 2 )  2ag t1 t 2  2af(t1  t 2 )  c  0 ...(iii)
1 2 On solving equation (3) and (4) we get co-ordinates of
Subtracting (ii) from (iii) and (iii) from (iv), we get N(3a, 0).
Now, L1L2 = 4a = GT
a  t 23 (t 2  t 1 )  (t1  t 2  2t 3 )  2gt 3  2f  0 ...(iv)
Also L 1L 2  GT
2
a  t (t 2  t 3 )  (t 2  t 3  2t 1 )  2gt1  2f  0
1 ...(v) Figure GL1TL2 is square whose area is 8a2 sq. units.
20) If difference of ordinates of end points of chord of
Solving (vi) and (v), we get 2g  a(1  t1 t 2  t 2 t 3  t 3 t1 )
parabola is twice the length of latus rectum, then circle
and, 2f  a(t1  t 2  t 3  t1 t 2 t 3 ) drawn on this chord as diameter will touch the parabola.
Substituting the values of g and f in (ii), we get Proof
2
c  a (t1 t 2  t 2 t 3  t 3 t 1 ) Let P(at 2 , 2at) and Q(at12 , 2at1 ) be the coordinates of the
Putting the values of 2g, 2f and c in (i), we obtain the end-points of a chord PQ of the parabola y2 = 4ax.
equation of the required circle as
It is given that 2at1  2at  2  4a
2 2
x  y  ax(t1 t 2  t 2 t 3  t 3 t1  1)
 t1  t  4
ay(t1  t 2  t 3  t1 t 2 t 3 )  a 2 (t1 t 2  t 2 t 3  t 3 t1 )
The equation of the circle with chord PQ as diameter is
Clearly, it passes through S (a, 0) (x  at 2 )(x  at12 )  (y  2at)(y  2at1 )  0
Hence, the circumcircle of ABC passes through the focus
 x 2  y 2  a(2t 2  8t  16)x  4a(t  4)y
of the parabola.
a 2 t(t  4)(t 2  4t  4)  0
note
We can also prove this result by applying Ptolemy  x 2  y 2  2a(t 2  4t  8)x  4a(t  4)y
theorem on quadrilateral ABCS.
a 2 t(t  4)(t  2)2  0
19) Tangents and normals at the extremities of the latus
rectum of parabola y2 = 4ax forms a square and points of Now, to show that this circle touches the parabola . For
intersection of tangents and normals are (–a, 0) and (3a, 0) this, it is sufficient to prove that it intersects the parabola at
respectively. coincident points.

3.74
Parabola

Ordinates of the points of intersection of the above circle Proof


and parabola are the roots of the equation
Let O be the point (h, 0), and let P and P' be the points
y4 y2
 y 2
 (t 2
 4t  8) (at12 , 2at1 ) and (at 22 , 2at 2 ) .
16a 2 2
4a(t  4)y  a t(t  4)(t  2)2  0
2
Equation of PP' is (t 2  t1 )y  2x  2at1 t 2
If this pass through the point (h, 0), we have
y2
[ On putting x  in the equation of the circle ] h
4a 2h  2at1 t 2  t 1 t 2  
a
or, y 4  0y 3  8a 2 y 2 (t 2  4t  6)  64a 3 (t  4)y
h2
16a 4 t(t  4)(t  2)2  0 Hence AM  AM '  at12 ,at 22  a 2   h 2  AO 2 and
a2
This equation has four roots, such that two of them are the
 h
ordinates of P and Q i.e. 2at and 2at1 = 2a(t + 4). Let the PM  PM '  2at1 .2at 2  4a 2     4a  AO
other two roots be  and . Then,  a
23) From a point P a tangent is drawn, if Q is any point on
Sum of roots is 2at  2a(t  4)      0 the tangent such that QR and QL are perpendicular from
Q on PS and directrix respectively, then QL = SR (where S
Product of roots is
is focus).
2at  2a(t  4)    16a 4 t(t  4)(t  2)2 Concept of parabola of the form
     4a(t  2) and   4a (t  2)2 2
(x   y)2  2gx  2fy  c  0

 (  )2  (  )2  4  The given equation can be written as

(x   y)2  2gx  2fy  c


 (  )2  16a 2 (t  2)2  16a 2 (t  2)2
Now add an arbitrary constant  in the square root of the
 (  )2  0     second terms. Then the equation will be of the form
Thus, the circle cuts the parabola at coincident points Hence, i.e., (x   y  )2  xf1 ()  yf2 ()  f3 () ...(1)
the circle touches the parabola.
21) Portion of tangent intercepted between axis and point Now choose  such that the lines x  y    0 and
of contact is bisected by tangent at vertex. xf1 ()  yf2 ()  f3 ()  0 are perpendicular
Proof
 f ()
Given parabola is y2 = 4ax ...(1)    1  1
 f2 ()
The tangent at any point P(at2, 2at) to parabola (1) is
ty = x + at2 ...(2)  f1 ()  f2 ()  0 ...(2)
Equation of the axis of the parabola (1) is y = 0 Now substitute the value of  in (1) from (2)
Let the line (2) intersect the axis of the parabola. So Multiply and divide (2 + 2) in LHS of (1)
coordinates of Q are (– at2, 0).
2

2
 x   y   
2 2
 at 2  (at 2 ) 2at  0  i.e., (x  y  )  (   )   and RHS of
The mid-point of QP is  ,   2  2 
 i.e.(0,at)  
 2 2 
(1) by ( 2   2 )
which lies on y-axis (i.e., on x = 0) which is the tangent at
the vertex.
i.e., xf1 ()  yf2 ()  f3 ()  ( 2   2 )
22) Let O be any point on the axis and POP ' be any
chord passing through O, and if PM and P ' M ' be the  xf ()  yf ()  f () 
 1 2 3

ordinates of P and P ' , then AM.AM '  AO 2 and  ( 2
  2
) 
 
PM.P ' M '  4a.AO

3.75
Coordinate Geometry

Then equation (1) reduce in the form


 b2 
2 yb  2a  x  
 x  y      x  gay     4a 
   4  
  2  2   ( 2  2 )  2a
   
Slope of tangent is
b
Which is of the form Y2 = 4X
b
x   y   x  y   Hence slope of normal    tan(1800  )
Y ,X  2a
( 2  2 ) ( 2  2 ) b
 tan  
2a
1
4   slope of reflected ray  tan(1800  2)
(  2 )
2

 2 tan  
  tan 2    
1 2
 1  tan  
Latus rectum is 4 
( 2  2 )
 b 
Axis is Y = 0 or x  y    0  2   4ab
  2a 
2  2
1  b  (4a  b2 )
Equation of tangent at vertex is X = 0 or x  y    0  2
4a 
Vertex is the point of intersection of X = 0 and Y = 0 Hence equation of reflected ray is

i.e. x  y    0 and x  y    0 4ab  b2 


yb  x  
(4a 2  b 2 )  4a 
Equation of directrix : is X    0
 (y  b)(4a 2  b 2 )  (4ax  b 2 )
Equation of latus rectum : is X    0 which obviously passes through the focus S(a, 0)
Example 93
Focus Since axis and latus rectum intersect at the focus is A parabola of latus rectum 4a, touches a fixed equal
its co-ordinates can be obtained by solving X    0 and parabola the axes of the two curves being parallel;
prove that the locus of the vertex of the moving curve
Y = 0.
is a parabola of latus rectum 8 a.
Solved Examples Solution
Example 92 Let the given parabola is y2 = 4ax
A ray of light is coming along the line y = b from the If the vertex of moving parabola is () then equation
positive direction of X-axis strikes a concave mirror of moving parabola is
whose intersection with the x-y plane is a parabola
y2 = 4ax. Find the equation of the reflected ray and
show that it passes through the focus of the parabola.
Both a and b are positive.
Solution (0,0)
Given parabola is y2 = 4ax
 b2 
Equation of tangent at P  4a , b  is ()
 

Y b2 ,b
4a (y  )2  4a(x  )
P y=b

 y2
Normal Substituting the value of x i.e., x  in (2) w e
2  1800- 4a
X’ A S 0
180 -2 X
(a,0) 2  y2 
get (y  )   4a   
 4a 

Y’ y 2  2y  2   y 2  4a

3.76
Parabola

2y 2  2y  2  4a  0 ...(3)  (m1  m 2 )2 (m 2  m 3 )2 (m 3  m1 )2  C ...(1)


Since two parabolas (1) and (2) touch each other consider
Hence roots of (3) are equal i.e., discriminant = 0
y
" B 2  4AC  0 " (am12,2am1)
A
(2)2  4  2  ( 2  4a)
P(h,k)
4 2  32a   2  8a (am22,-2am2)
x
The required locus is y 2  8ax B
which has latus rectum double that of given parabola. C
Example 94 (am32,-2am 3)
Three normals are drawn to the parabola y2 = 4ax
cos from any point on the straight line y = b sin .
Prove that the locus of the orthocentre of the triangle (m1  m 2 )2  m12  2m1m 2  m 22
formed by the corresponding tangent is the ellipse
 [m1 (m2  m 3 )  2m1m2  m 2 (m1  m 3 )
x2 y2
  1 , the angle  being variable. h  2a 3k
a2 b2  [( m1m2 )  3m1m2 ]  
a am 3
Solution
y2 = 4Ax where A = a cos 
y + tx = 2A t + at3 passes through , bsin    
 3k   3k   3k 
b sin  + t = 2At + At3 (h 2a)

   m  (h  2a)  m  (h  2a)  m 
At3 + (2A – )t – b sin  = 0  x 3   2   1 
 l  m  n 

y hence equation (1) becomes


P
(x  l )(x  m)(x  n)  constant
(,b sin )
x 3  (l  m  n)x 2  (lm  mn  nl)x  lmn  0
O x
Q  1  1 
(h  2a)3  3k     (h  2a)
2

 m1 m 2 m3 
R
 1 1 1  27k 3
9k 2     (h  2a) 
 m1m 2 m 2m 3 m3 m4  m1m 2 m3

b sin  = constant.
 t1  t 2  t 3  0 ; t 1 t 2 t 3  Now replace h by x and k by y.
A
Example 96
Let orthocentre be (h , k), as orthocentre lies on Find the length of latus rectum of the parabola
directrix. So, (a2 + b2) (x2 + y2) = (bx + ay – ab)2.
h = – A = – a cos  ...(1) Solution
and k  A(t1  t 2  t 3  t1 t 2t 3 ) The given equation may be written as
 b sin   a 2 x 2  a 2 y 2  b2 x 2  b 2 y 2
k  A0  
 A   b2 x 2  a 2 y 2  a 2 b 2  2abxy  2a 2 by  2ab 2 x
k = b sin  ...(2)
 a 2 x 2  2abxy  b2 y 2  2ab2 x  2a 2 by  a 2 b2
x2 y2
from (1) and (2) locus is 2  2  1
a b  ab 
Example 95  (ax  by)2  2ab  bx  ay  
 2 
Locus of a point P when the 3 normals drawn from it
are such that area of the triangle formed by their feet ab
is constant. Since ax  by  0 and bx  ay  0 are
2
Solution
perpendicular
Area of ABC = constant
 ab 
 bx  ay  2 
2
am12 2am1 1  ax  by 
(a 2  b 2 )     2ab (a 2
 b 2
)  
am22 2am2 1  C 2
 a b 
2 2 2
 (a  b ) 
 
am23 2am3 1

3.77
Coordinate Geometry

 ab  y
2 bx  ay 
 ax  by  2ab  2  P
    
2 2 2 2 2 2
 a b  (a  b )  (a  b )  Q
 
x
which is of the form Y 2  4 x O S C

2ab
Therefore the latus rectum  4   G(t’)
(a 2  b 2 )
Example 97 Shortest distance between the two curves are along
Find the area of quadrilateral TSPM, where P(t) is the common normal
any point on parabola, y2 = 4ax, where S is focus, M Let P(x1, y1) be the point on parabola through which
is perpendicular drawn from point P on directrix and the minimum distance line passes.
T point of intersection of tangent at P and axis of
parabola. y1
Therefore, slope of shortest distance line 
Solution 2
We know that TSPM is a rhombus
 y2 
1  Now, P(x 1 , y1 )  P  1 , y1  and shortest distance line
area of TSPM  4  (SY)  (PY)   4 
2 
passes through the centre of circle C
Also we know that portion of tangent between the
parabola and its axis is bisected at tangent at vertex. y1  0
 Slope of shortest distance line 
 y12 
  3
y  4 

 y1 4y
M Y P(at2,2at)   2 1  y13  4y1  0
2 y 1  12
x
T V S(a,0)  y1  0 or y1  2
(-at2,0)
P(x1 ,y 1 ) can have co-ordinate (0, 0) ; (1, 2) or
(1, –2) Shortest distance = min {PC – 1}
 min{2, 2 2  1, 2 2  1}  2 2  1
 Y is mid-point of TP and YS  rTP equation of normal along the shortest distance has
y1  2 
1  slopes      1
Area of  TSPM  2  2 TP  YS   TP  YS 2  2
equation of normal along shortest distance are given
by y = mx – 2am – am3 is y = x – 3 and y + x = 3
Also co-ordinates of T are (at 2 0)  co-ordinates of
Example 99
Y are (0, at) (By mid-point of formula) Find locus of point of intersection of tangents to the
parabolas y2 = 4(x + 1) and y2 = 8 (x + 2) which are
TP  (2at 2 )  (2at)2  2at t 2  1 and perpendicular to each other.
Solution
YS  a 2  a 2 t 2  a t 2  1 Equation of tangent to first parabola is
From (i) are y = m (x + 1) + (1/m)
1
  
Area of TSPM  2at t 2  1 a t 2  1  2at 2 t  t 2  1 
or y  mx   m  
 m
...(1)

Example 98 Equation of tangent to second parabola is


Find the equation of normals to parabola y2 = 4x
along the minimum distance between the parabola 2
and y  m '(x  2) 
and the circle x2 + y2 – 6x + 8 = 0 m'
Solution
 1 
Equation of parabola y2 = 4x  m' x  2  m '  ...(2)
and equation of circle is x2 + y2 – 6x + 8 = 0  m' 
having its centre at (3, 0) and radius = 1 Now, m  m '  1

3.78
Parabola

Then from equation (2)


y
x  1 
y  2   m
m  m 
O
A x
x  1
 y    2 m   ...(3)
m  m B

Subtracting equation (3) from (1), we get.


Equation of a normal to the two parabolas can be
 1  1
x  m    3  m    0 or x + 3 = 0 written as
 m  m y = m (x + a) – 2am – am3 ...(3)
Example 100 and x  m'(y  a)  2am'  am '3 ...(4)
P is a point which moves in the x – y plane such that
the point P is nearer to the centre of square then any respectively.
of the sides. The four vertices of the square are If these normals be perpendicular to each other, then
( a,  a) . The region in which P will be bounded by m '  m . Putting this result in equation (4), it reduces
to
parts of parabolas of which one has the equation
a) y2 = a2 + 2ax b) x2 = a2 + 2ay x  m(y  a)  2am  am3 ...(5)
2 2
c) y + 2ax = a d) none of these If P(h, k) be the intersection point of the two normals,
Solution then (h, k) must satisfy both equation (3) and (5)
(a, b, c) i.e. mh  k  am  am 3 ...(6)
If P (x, y) lie in region, then OP < PM, RP, NP, PQ and h  mk  am  am 3
...(7)
Subtracting equation (7) from equation (6), we have
 x 2  y 2  |a  x |, x 2  y 2  |a  x | m(h  k)  (h  k)  0
hk
x 2  y 2 |a  y| and x 2  y 2  |a  y| gives m 
hk
Putting this value in any one of the equation (6) or
y
(7), we have
B N A 3
R M h k h k h k
h   k  a   a 
P hk hk hk
x’ x
O
(h  k)2  2a(h  k)
C Q D i.e. h 2  2hk  k 2  2ah  2ak  0
Putting (x, y) in place of (h, k) gives the equation of
y’ 2 2
the desired locus as x  2xy  y  2ax  2ay  0
On squaring, we get which is the equation of a parabola
 The region bounded by the curves Example 102
x2 + y2 = (a – x)2 ; x2 + y2 = (a + x)2 Prove that the line ax + by + c = 0 touches the
x2 + y2 = (a – y )2 ; x2 + y2 = (a + y)2
2 2 (lx  my  n)2
Example 101 parabola (x  a)  (y  b)  if
Two equal parabolas have the same focus and their l 2  m2
axis at right angles ; a normal to one is perpendicular (A 2  B 2 )| la  mb  n|  2| Al  Bm)(Aa  Bb  c)|
to the normal to the other. Prove that the locus of the Solution
intersection points of these normals is another
The equation of the parabola is
parabola.
Solution (lx  my  n)2
(x  a)2  (y  b)2 
If we choose the common focus O as the origin and l 2  m2
the vertex of the parabola as A(– a, 0) and B(0, –a),
then equation of the two equal parabolas can be lx  my  n
 (x  a)2  (y  b)2 
written as l 2  m2
y2 = 4a(x + a) ...(1)
and x2 = 4a (y + a) ...(2)  SP  PM
Where, S (a, b) is the focus and lx + my + n = 0 is
the directrix of the given parabola.

3.79
Coordinate Geometry

Let AY be the tangent at the vertex and Z be the


point of intersection of the tangent at any point P and l A
 
AY. Then, 0
 tan(90  )  m B Am  Bl
lA  cot  
SZ  PT 1 lA  mB
Suppose the directrix BM cuts PT at Q. mB
STP   SPT  (say) 1 | lA  mB|
0
 sin   
 ASZ  PQM  90    Am  Bl 
2
(l  m 2 )(A 2  B 2 )
2

1 
Now, in ASZ we have  lA  mB 
lx+my+n=0

y From (i) and (ii), we get


0
+ C= | lA  mB| A 2  B2 la  mb  n
+ By   
M P Ax 2 2
(l  m )(A  B ) 2 2 2
l m 2 Aa  Bb  c
900-  Z
  (A 2  B 2 )| la  mb  n|  2
Q S(a,b)
x’ T A x |(lA  mB)(Aa  Bb  C)|
B
 0
90 -  Example 103
If the tangents at points P and Q on the parabola
y2 = 4ax meet at T1 whereas tangents at R and S on
y’ the parabola meet at T2 such that the points P, Q, R,
S, T1 and T2 are concyclic. Prove that the mid-point
AS
cos(ASZ)  of T1 and T2 is the focus of the given parabola.
SZ Solution
1 SB Let P(at12 , 2at 1 ),Q(at 22 , 2at 2 ), R(at 32 , 2at 3 ) and
 cos(900  ) 
2 SZ
S(at 42 , 2at 4 ) be four points on the parabola. The
1 tangents at P and Q intersect at T1 (at1t2,a(t1 + t2))
[ AS = AB  AS  SB ]
2 and the tangents at R and S intersect at
1 SB
T(at3 t4, a(t3 + t4)).
 sin   It is given that points P, Q, R, S, T1 and T2 are concylic.
2 SZ So, let these points lie on the circle.
But, x2 + y2 + 2gx + 2fy + c = 0 ...(i)
SB = Length of the perpendicular from S(a, b) on Let (at2,2at) be an arbitrary point on the parabola.
the directrix lx + my + n = 0 Then the values of parameter for different points of
la  mb  n intersection of the parabola y2 = 4ax and the circle
 are the roots of the equation
l 2  m2 a2 t4 + 4a2 t2 + 2agt2 + 4aft + c = 0 ...(ii)
And, Since P, Q, R and S are the points of intersection of
SZ = Length of the perpendicular from S (a, b) on the parabola and the circle (i). Therefore, t1, t2 , t3
the directrix Ax + By + C = 0 and t4 are the roots of the equation (ii) such that
t1 + t2 + t3 + t4 = 0 ...(iii)
Aa  Bb  C
 2g
A2  B2 (t1  t 2 )(t 3  t 4 )  t1t 2  t 3t 4  4  ....(iv)
a
1  SB  4f
  sin     t1t 2 (t 3  t 4 )  t 3 t 4 (t1  t 2 )  
2  SZ  ....(v)
a
1 A 2  B2 la  mb  n c
 sin    ...(i) and, t1t 2 t 3 t 4  ....(vi)
2 2
l m 2 Aa  Bb  c a2
Since T1 (at1t2, a(t1 + t2)) and T2(at3t4, a(t3 + t4) lie on
l circle (i). Therefore,
Now, Slope of BM  
m
a 2 (t 1t 2 )2  a 2 (t 1  t 2 )2  2agt1 t 2  2af(t 1  t 2 )  c  0
A ....(vii)
and, Slope of PT  
B and
a 2 (t 3 t 4 )2  a 2 (t 3  t 4 )2  2agt 3 t 4  2af(t 3  t 4 )  c  0

3.80
Parabola

....(viii)
1
Adding (vii) and (viii), we get and, y  0   (x  a) ...(ii)
m
a 2 {(t 1t 2 )2  (t 3 t 4 )2 }  a 2 {(t1  t 2 )2  (t 3  t 4 )2 } These two lines meet the directrix x = –a at T and
2ag(t1 t 2  t 3 t 4 )  2af(t 1  t 2  t 3  t 4 )  2c  0 T ' . Solving these two equations with x = – a, we
obtain that the coordinates of T and T ' are
 a 2 {(t 1t 2  t 3 t 4 )2  2t 1t 2 t 3 t 4 }  2a 2 (t 1  t 2 )2
(–a, –2am) and (–a, 2a/m) respectively. Therefore,
2ag(t1t 2  t 3 t 4 )  2c  0
the coordinates of the mid-point of TT ' are
 2c    1 
 a 2 (t 1t 2  t 3 t 4 )2  2   2a 2 (t1  t 2 )2   a, a  m   
 a    m 
2ag(t1 t 2  t 3 t 4 )  2c  0 The equation of the tangent to y2 = 4ax parallel to
the lines (i) and (ii) are
 a(t 1t 2  t 3 t 4 )2  2a(t 1  t 2 )2  2g(t1 t 2  t 3 t 4 )  0
a x
...(ix) y  mx  and y    am
From (iii) and (iv), we get m m

2g  1 
(t1  t 2 )2  t1 t 2  t 3 t 4  4  These two tangents intersect at  a, a  m  m  
a   
 a(t 1  t 2 )2  a(t 1t 2  t 3 t 4 )  4a  2g ...(x) which is the mid-point of TT ' .
Example 105
Eliminating a(t 1  t 2 )2 from (ix) and (x), we get The tangent and normal at P(t), for all real positive t,
a(t1 t 2  t 3 t 4 )2  2a(t1 t 2  t 3 t 4 )  8a  4g  2g(t1t 2  t 3 t 4 )  0 to the parabola y2 = 4ax meet the axis of the parabola
in T and G respectively, then find the acute angle at
 a(t1t 2  t 3 t 4 )2  2(t 1t 2  t 3 t 4 )(a  g)  (8a  4g)  0 which the tangent at P to the parabola is inclined to
the tangent at P to the circle passing through the points
2(a  g)  4(a  g)2  4a(8a  4g) P, T and G.
 t1 t 2  t 3 t 4 
2a Solution
Slope of tangent of parabola at point
2(a  g)  2 (3a  g)2 P(t) = m1 = 1/t.

2a We know that if tangent to parabola at a point P
meets the axis of parabola at T and normal at P meets
2(a  g)  2(3a  g)
 the axis at G, then SP = ST = SG.
2a  the circle passing through T, P and G must have its
2g centre at S (focus)
 2, 4   PS will be normal at P
a
 Slope of tangent to circle at point
 t1 t 2  t 3 t 4  2 ...(xi)
The coordinates of the mid-point of T 1 T 2 are y
 P
a a 
 (t1t 2  t 3 t 4 ), (t1  t 2  t 3  t 4 ) 
 2 2 
x
= (a, 0) T S G
Hence, the mid-point of T1T2 is the focus of the given
parabola.
Example 104
Two perpendicular straight lines through the focus of
the parabola y2 = 4ax meet its directrix in T and 1 1 1  t2
P    m2 (say)
T ' respectively. Show that the tangents to the parabola Slope of PS  2t  2t
parallel to the perpendicular lines intersect in the mid-  2 
 t 1
point of TT ' .
 Angle between tangent at P to circle and tangent
Solution at P to parabola is given by
The equations of two perpendicular lines through the
focus of the parabola are 1 1  t2

y – 0 = m (x – a) ...(i) m1  m 2 t 2t 2t  t  t 3
tan    
1  m1m 2 1 (t 2  1)
1  2 (1  b 2 )
2t

3.81
Coordinate Geometry

|t|  t as t > 0    tan 1(t) Example 107


Find the equation of a parabola having its vertex on
Example 106 the Y-axis, concavity towards the +ve Y-axis and
A parabola is drawn touching the x-axis at the origin touching the equal parabola y2 = ax.
and having its vertex at a distance k from the X-axis. Solution
Prove that the axis of this parabola is a tangent to the Equation of the required parabola can be written as
parabola x2 = – 8k(y – 2k) x2 = a(y – k) ...(1)
Solution where (0, k) are the coordinates of its vertex. Solving
The figure shows a parabola touching the X-axis at for the intersection points of the given parabola
the origin O. V is the vertex, VX ' is the axis of the y2 = ax ...(2)
parabola and VC = k (given). and the required parabola, we have
The tangent and the normal at O intersect the axis of y4 – a 3 y + a 3 k = 0 ...(3)
the parabola at A and B respectively and a which shows that the two parabolas will have four
perpendicular from O to the axis of the parabola meets intersection points, real or imaginary.
it at N.
Y
y

x’ required
parabola given
y’
parabola
N B O X
V
x
 A C D O
If the parabolas touch at only one point, then two
roots of equation (3) must be identical and the other
two roots must be imaginary.
we have AV = VN = x(let). Thus, if the ordinate of the intersection point be b
Also, VAC  DNO  (let) then the bi-quadratic expression (3) must be divisible
From similar triangles AVC and AND, we have by (y – b)2. Dividing, we have
N ND y 2  2by  b 2  y 4  a 3 y  a 3 k  y 2  2by  3b 2
2
AV VC
gives ND = 2VC = 2k. y 4  2by 3  b2 y 2
Similarly, we have
CD = AC = VC cot  = k cot  2by 3  b 2 y 2  a 3 y  a 3 k
and DO = ND tan  = 2k tan . 2by 3  4b 2 y 2  2b 3 y
Therefore, we have
CO = CD + DO = k cot  + 2k tan . 3b 2 y 2  (2b3  a 3 )y  a 3 k
The coordinates of V is (–CO, k). Substituting m for
3b 2 y 2  6b3 y  3b 4
 1
tan , we have CO  k  2m   (4b3  a 3 )y  (a 3 k  3b 4 )
 m 
whose remainder must be zero
Hence, the equation of the axis of the parabola is
i.e. 4b3 – a3 = 0 ...(4)
 k(2m  1) 2 and a3k – 3b4=0 ...(5)
y  k  m x   Solving the two equations, we have
 m 
3a
i.e. y  2k  mx  2km 2 which can be written as k
44 / 3
2k Putting the value of k in equation (1) gives the equation
x  (y  2k)  ...(1)
 of the required parabola.
Example 108
1 Prove that the orthocentres of the triangles formed by
where   .
m three tangents and the corresponding three normals
we know that this equation is a general tangent to the to a parabola are equidistant from the axis.
Solution
parabola x 2  8k(y  2k) .
Let the parabola be y2 = 4ax ...(1)

3.82
Parabola

and (am12 , 2am1 ) , (am 22 , 2am 2 ) and (am 23 , 2am 2 ) Solution


Let the parabola y2 = 4ax touch the sides BC, CA
be the co-ordinates of the points P, Q and R and AB of a triangle ABC at D, E and F respectively,
respectively. Then the equations of the tangents at P, and suppose the co-ordinates of the points D, E and
Q and R w.r.t. the parabola (1) are
F be (at12 , 2at1 ),(at 22 , 2at 2 ) and (at 23 , 2at 3 ) respectively..
 m1y  x  am12 ...(2)
So the tangents at D, E and F will represent the sides
 m 2 y  x  am 22 ...(3) BC, CA and AB respectively.
Tangents at the point D, E and F are respectively
 m 3 y  x  am 22 ...(4)
t 1y  x  at 12 ...(1)
Solving (2) and (3), we have the co-ordinates of the
point of intersection say A as {am1m2, – a(m1 + m2)}. t 2 y  x  at 2
2 ...(2)
Now equations to the line through A and perpendicular
and t 3 y  x  at 32 ...(3)
to (3) will be y  a(m1  m 2 )  m3 (x  am1m 2 ) , ...(5)
On solving (2) & (3), (3) & (1) and (1) & (2) in pairs,
Similarly the equation to the line through B the point
we get the co-ordinates of A, B and C as
of intersection (3) and (4) and perpendicular to (2) is
{at 2 t 3 ,a(t 2  t 3 )} ; (at1t 3 ,a(t1  t 3 ) & {at1t 2 , a(t1  t 2 )}
y  a(m 2  m3 )  m1 (x  am 2m 3 ) ...(6)
respectively. Now the diameter through the point A
On solving (5) and (6), we get the ordinate of the will be y = a(t 1 + t 2 ) and equation of DE is
orthocentre of the triangle formed by (2), (3) and (4)
which is y(t1  t 2 )  2x  2at1t 2 .
a(m1  m2  m 3  m1m 2m 3 ) Solving these equations, we have the co-ordinates of
point of intersection b as
Equations of normals at P, Q and R are
y  m1x  2am1  am13 ...(7) F
B
3
y  m 2 x  2am 2  am 2 ...(8)
D b
c
and y  m 3 x  2am 3  am 32 ...(9) A
Solving (8) and (7), we have the co-ordinates of one
of the vertices of the triangle formed by these three
C
E
normals as
{2a  a(m12  m 22  m1m 2 ),am1m 2 (m1  m 2 )}
Equation to the line through this point and {(a / 2) (t 22  t 2 t 3  t1 t 3  t 1t 2 ), a(t1  t 2 )}
perpendicular to (9) is Again the equation of DF is
y(t1+t3)=2x+2at1t3; this cuts the diameter through A
1 in c
y  am1m 2  m1  m 2   
m2
 x  2a  am12  am 22  am1m 2  By solving the two equations the co-ordinates of c
...(10) are {(a / 2) (t 22  t 2 t 3  t1 t 3  t 1t 2 ), a(t1  t 2 )}
Similarly the equation to the other perpendicular will If slope of Bb be m1 and that of Cc be m2, then
be
2a(t 2  t1 )
m1 
y  am 2m 3  m2  m 3   a  t  t 2 t 3  t1 t 3  t 1t 2 
2
2
1

m1
 x  2a  am22  am 23  am 2m 3  ...(11) 2a  t 2  t1  2
 
a  t 2  t 3  t 2  t1   t 2  t 3 
On solving (10) and (11), we get the ordinate of the
point of intersection as – a(m1 + m2 + m3+ m1m2m3). 2
This is also the ordinate of the point of intersection or Similarly slope of Cc is m 2 
ortho-centre of the triangle formed by tangents.
 t 2  t3 
Hence the ortho-centres are equidistant from the axis Since m1= m2 hence the lines Cc and Bb are parallel.
i.e. x-axis. Example 110
Example 109 Prove that the sum of the angles which the three
A parabola touches the sides of a triangle ABC in the normals, drawn from any point O, make with the axis
points D, E and F respectively; if DE and DF cut the exceeds the angle which the focal distance of O makes
diameter through the point A in b and c respectively, with the axis by a multiple of .
prove that Bb and Cc are parallel. Solution
Let y = mx – 2am – am3 be the normal passing

3.83
Coordinate Geometry

through O, whose co-ordinates are (h, k) (say); then  lh  mk  n  0 ...(5)


am3 + m(2a – h) + k = 0 , Then
Substituting the values of h and k from (3) and (4) in
2a  h k
m1  m2  m 3  0,  m1m 2  and m1m 2m3  . y12  2ax 1
a a (5) we get l.  my1  n  0
2a
If 1 , 2 and  3 be the angles of inclination of these
i.e., l(y 12  ax 1 )  2a(my1  n)  0
normals with axis, i.e., x-axis, then
m1  tan 1 , m 2  tan  2 and m3  tan 3 . i.e., l(y12  4ax 1 )  2a(lx 1  my 1  n)  0
If S is the focus (a, 0), then the slope of  Locus of (x1, y1) is l(y2 – 4ax)+2a(lx + my + n)=0.
Example 112
k Show that three circles can be drawn to touch a
SO   tan (say) ...(1)
ha parabola and also to touch at the focus, a given
Again tan(1  2   3 ) straight line passing through the focus. Also prove that
the tangents at the points of contact with the parabola,
tan 1  tan 2  tan 3  tan 1 tan 2 tan 3 form an equilateral triangle.
 Solution
1   tan 1 tan  2  tan  2 tan  3  tan 1 tan 3 
Let us choose the equation of the parabola as y2 =
 m1  m 2  m 3  m1m 2m 3  0  (k / a) k 4x. Then its focus is S(1, 0).
   Any line through S can be chosen as y = m (x– 1)
1   m1m 2  m2 m3  m3 m1   2a  h  h  a
1   and equation of a circle touching this line at S can be
 a  written as
or tan(1  2  3 )  tan 
Hence 1   2  3  n   . S
Example 111
The polar of a point P with respect to the parabola x2 P
= 4ax meets the curve in Q and R. Show that if P lies
on the line lx + my + n = 0, then the middle point of (x  1)2  y 2  {y  m(x  1)}  0
QR lies on the parabola i.e. x 2  y 2  (2  m)x  y  1  m  0 ...(1)
2
l(y  4ax)  2a(lx  my  n)  0 . Any point on the parabola can be chosen as P(t2,2t).
Solution
x
Now equation of the tangent at P, is y   t and
t
Q equation of a circle touching the parabola at P can
lx+my+n=0

be written as
P(h,k) T
 x 
R (x  t 2 )2  (y  2t)2    y   t   0
 t 

Let P be (h, k) and T the middle point of QR be i.e. x 2  y 2   2t 2    x  (  4t)y  t 4  4t 2  t  0


(x1, y1),  t
The equation of the line QR is ...(2)
yy1  2a(x  x 1 )  y12  4ax 1 Now, if equations (1) and (2) represent the same circle,
then comparing the coefficients, we have
i.e., yy1  2ax  y12  2ax 1 ...(1)
1
QR is the polar of P. m    2  2t 2  0 ...(3)
t
yk  2ax  2ah ...(2)
    4t  0 ...(4)
Equations (1) and (2) represent the same line.
4 2
and m  t  1  t  4t  0 ...(5)
y1 y 2  2ax1
 1 1 Eliminating  from the above equations, we have
k 2ah
 y1  k ...(3) m 1 / t 2  2t 2
1 1 4t 0
and y12  2ax 1  2ah ...(4) 4 2
m t t  4t
The point P lies on lx  my  n  0

3.84
Parabola

If P be the mid-point of the chord AB (see figure),


m 1 / t 2  2t 2 then OP  AB and hence equation of AB is
i.e. 1 1 4t 0 [R3  R3–R1]
t
m t  1 / t t 4  2t 2  1 y  2at   (x  at 2 ) ...(3)
2
If this chord cuts the X-axis at the point Q(h,0), then
4 2  3 1 putting in equation (3) we have 4at = t(h–at2)
i.e. m(t  2t  3)   3t  2t    0
 t
i.e. t(at 2  h  4a)  0
2 2 2 2
i.e. mt(t  1)(t  3)  (3t  1)(t  1)
h
2 2
i.e. mt(t  3)  3t  1 [ t 2  1  0] gives t  0,  4
a
i.e. mt 3  3t 2  3mt  1  0 ...(6) For t to be real and different from zero, we have
which proves that there are three values of t and hence h > 4a ...(4)
three circles satisfying the given condition. Also, since P lies within the circle, therefore we have
Now, the slope of a tangent to the parabola at point
(at 2 )2  (2at 2 )  a 2
P(t), is equal to 1/t. Thus, rewriting equation (6), as
i.e., t 4  4t 2  1  0
1 3m 3
 2  m0 [ Roots of the equation t 4 + 4t 2 – 1 = 0 are
t3 t t
1 (2  5) ]
and putting  tan  , we have
t i.e. (2  5)  t 2  ( 5  2)
3 2
tan   3m tan   3 tan   m  0 i.e. t 2  5  2 [  t2 is always positive]
3 tan   tan3  h
i.e. m   tan 3 i.e. 4 5 2
1  3 tan 2  a
i.e. 3  n  tan 1 m(n  I) i.e. h  ( 5  2)a . ...(5)
n 1 From inequalities (4) and (5), we have
i.e.    tan 1 m
3 3 4a  h  ( 5  2)a
which gives three distinct values of  , as Make Concepts Clear 3.6
1
tan m  tan m 2 tan m 1 1 1. Find the locus of centres of a family of circles passing
 ,  ,  through the vertex of the parabola y2 = 4ax and cutting
3 3 3 3 3
the parabola orthogonally at the other point of
which proves that the tangents are equally inclined to
intersection.
each other by /3 and hence form an equilateral
2. Equilateral triangles are circumscribed to the parabola
triangle.
y2 = 4ax. Prove that their angular points lie on the
Example 113
conic y2 = (3x + a)(x + 3a).
Find the points on the X-axis from where exactly three
3. Let S() be the area included between the parabola y
distinct chords of the circle x2 + y2 = a2 can be drawn
= x2 + 2x – 3 and the line y = x + 1. Find the least
which are bisected by the parabola y2 = 4ax, a > 0.
value of S().
Solution
4. A normal to the curve x2 + kx – y + 2 = 0 at the
Let P(at 2 , 2at) be a point on the given parabola point whose abscissa 1, is parallel to the line, y = x.
y2 = 4ax ...(1) find the area in the first quadrant bounded by the
and lying inside the given circle curve, normal and the axis of x.
x2 + y2 = a 2 ...(2) 5. Prove that the locus of the middle points of all tangents
Slope of the line joining P and the centre O of the drawn from points on the directrix to the parabola y2
2at 2 = 4ax is y 2 (2x  a)  a(3x  a)2
circle is  
6. Find radius of the largest circle which passes through
at 2 t
the focus of the parabola y2 = 4x and contained in it
7. Two tangents to a parabola intersect on a fixed tangent
at point P. Such that product of segments of these
tangents is constant. Prove that the locus of their point
Q
O x of intersection of two variable tangents is a straight
B
line.
P 8. Two parabola P1 and P2 , intersect at two different
A

3.85
Coordinate Geometry

points, where P1 is y = x2 – 3 and P2 is y = kx2. The that the common chord of these circles passes through
abscissa which is positive is designated point A and the centroid of the triangle PQR.
its value is ‘a’. The tangent line l at A to the curve P2 14. If a parabola, whose latus rectum is 4c, slide between
intersects curve P1 at point B, other than A. If abscissa two rectangular axes. Prove that the locus of its focus
of point B is 1, then find value of a. is x2y2 = c2(x2 + y2), and that the curve traced out by
9. Prove that the foot of any perpendicular from the 2 2
 2 2

point (0, – c), c > 0, to any normal to the parabola x2 its vertex is x 3 y 3  x 3  y 3   c
2

= 4ay lies on the curve whose equation is x4 = (y +  


c) {x2 (2a – y) + a(y + c)2}. 15. Parabolas are drawn to touch two given rectangular
10. The normal at a point P to the parabola y2 = 4ax axes and their foci are all at a constant distance c
meets its axis at GQ is another point on the parabola from the origin. Prove that the locus of the vertices of
such that QG is perpendicular to the axis of the 2 2 2
parabola. Prove that QG2 – PG2 = constant. these parabolas is the curve x 3  y 3  c 3 .
11. Circles are drawn through the vertex of the parabola
16. If (x1, y1), (x2, y2) and (x3, y3) be three points on the
to cut the parabola orthogonally at the other point of
parabola y2 = 4ax, the normals at which meet at a
intersection. Prove that the locus of the centres of the
circles is the curve x1  x 2 x 2  x 3 x 3  x1
point, then evaluate  
2y2(2y2 + x2 – 12ax) = ax(3x – 4a)2. y3 y1 y2
12. If the normals at P and Q meet on the parabola,
prove that the point of intersection of the tangents at
P and Q lies either on a certain straight line, which is Answer Key
parallel to the tangent at the vertex, or on the curve 1. 2y 2 (2y 2  x 2  12ax )  ax(3x  4a )2
whose equation is y2(x + 2a) + 4a3 = 0.
3.32/3 4.7/6 6.4 8.3
13. Through a point P are drawn tangents PQ and PR to
16.0
a parabola and circles are drawn through the focus to
Solutions are on Page No. 3.116
touch the parabola in Q and R respectively ; prove

3.86
Practice Session-1
for
Single Choice
JEE MAIN
C) independent of the latus rectum and lies between
1. If y  m1x  c and y  m 2x  c are two tangents to 3 5
and
4 6
the parabola y 2  4a  x  a   0 , then
D) independent of the latus rectum and lies between
A) m1  m 2  0 B) 1  m1  m 2  0 2 3
and
C) m1m 2  1  0 D) 1  m1m 2  0 3 4
2. A line bisecting the ordinate PN of a point 6. Two mutually perpendicular tangents of the parabola

P  at 2 , 2at  , t  0 , on the parabola y 2  4ax is drawn y 2  4ax meet its axis in P1 and P2. If S is the focus

parallel to the axis to meet the curve at Q. If NQ 1 1


meets the tangent at the vertex at the point T, then of the parabola then SP  SP is equal to :
1 2
the coordinates of T are
4 2 1 1
A)  0,  4 / 3  at  B)  0, 2at  A) B) C) D)
a a a 4a
C) 1 / 4  at , at 
2
D) (0, at) 7. If parabola y 2  x and
3. Which one of the following equations represented 2 2 2
25  x  3    y  2     3x  4y  2  are equal,
parametrically equation to a parabolic curve ?  
A) x  3 cos t ;y  4 sin t then value of  is
A) 9 B) 3 C) 7 D) 6
t
B) x 2  2  2cos t ; y  4 cos 2 8. The condition that the two tangents to the parabola
2 y 2 = 4ax become normal to the circle
C) x  tan t; y  sec t x2 + y2 – 2ax – 2by + c = 0 is given by
A) a2 > 4b2 B) b2 > 2a2
t t 2
C) a > 2b 2
D) b2 > 4a2
D) x  1  sin t ; y  sin  cos
2 2 2

4. A parabola y = ax2 + bx + c crosses the x  axis at (

2
 2
9. min  x 1  x 2   5  1  x 1  4x 2   x , x
1 2  R is
, 0) ( , 0) both to the right of the origin. A circle also
passes through these two points. The length of a A) 4 5  1 B) 4 5  1
tangent from the origin to the circle is :
C) 5 1 D) 5  1
bc b c 10. If the normal to a parabola y2 = 4ax at P meets the
A) B) ac2 C) D)
a a a curve again in Q and if PQ and the normal at Q
makes angles  and  respectively with the x-axis then
5. In a parabola y 2  4ax the angle  that the latus
tan (tan  + tan ) has the value equal to
rectum subtends at the vertex of the parabola is :
A) dependent on the length of the latus rectum 1
A) 0 B) – 2 C) – D) – 1
B) independent of the latus rectum and lies between 2
5 11. A parabola y  ax 2  bx  c crosses the x – axis at
and 
6  ,0  ,0  both to the right of the origin. A circle
also passes through these two points. The length of a
Coordinate Geometry

tangent from the origin to the circle is : A) real and distinct B) real and equal
C) imaginary D) none of these
bc b c
A) B) ac2 C) D) 20. From a point A(t) on the parabola y 2  4ax , a focal
a a a
chord and a tangent is drawn. Two circles are drawn
12. The angle between the tangents drawn from the point
in which one circle is drawn taking focal chord AB as
(1, 4) to the parabola y2 = 4x is
diameter and other is drawn by taking intercept of
A) /6 B) /4 C) /3 D) /2
tangent between point A and point P on the directrix,
13. A ray of light travels along a line y = 4 and strikes the
as diameter. Then the common chord of the circle is
surface of a curve y 2  4  x  y  , then equations of A) line joining focus and P
the line along which reflected ray travel is B) line joining focus and A
A) x = 0 B) x = 2 C) tangent to the parabola at point A
C) x + y = 4 D) 2x + y = 4 D) none of these
14. A line is drawn form A(–2, 0) to intersect the curve 21. The point(s) on the parabola y2 = 4x which are closest
to the circle, x2 + y2  24y + 128 = 0 is/are
y 2  4x in P and Q in the first quadrant such that
1 1 1
A) (0, 0)  
B) 2 , 2 2 C) (4, 4) D) none
  , then slope of the line is always.
AP AQ 4 22. Maximum number of common normals of y 2  4ax

1 1 and x 2  4by is equal to


A)  3 B)  C)  2 D) 
3 3 A) 3 B) 4 C) 6 D) 5
15. Minimum area of circle which touches the parabolas 23. If the line x  y  1  0 is a tangent to a parabola
y  x 2  1 and y  x 2  1 is with focus (1, 2) at A and inersects the directrix at B
and tangent at vertex at C respectively, then AC.BC
9 9 is equal to :
A) sq.unit B) sq.unit
16 32 A) 2 B) 1 C) 1/2 D) 1/4

9 9 24. From a point  sin , cos   if three normals can be


C) sq.unit D) sq. unit
8 4 drawn to the parabola y 2  4ax , then the value of
16. Two parabolas have the same focus. If their directrices ‘a’ is
are the x – axis and the y – axis, respectively, then the
slope of their common chord is 1   1 
A)  , 1 B)   , 0 
4 3 2   2 
A)  1 B) C) D) none
3 4 1  1   1
17. The locus of the point of intersection of normals drawn C)  , 1 D)  , 0    0, 
2  2   2
to a parabola y2 = 4ax at the extremities of a chord
which subtends a right angle at the vertex is 25. Two mutually perpendicular tangents of the parabola
y2 = 4ax meet the axis in P1 and P2. If S is the focus
A) y2 = 16a(x – 6a) B) y2 = 16a(x + 6a)
2
C) y = –16a(x – 6a) D) y2 = –16a(x + 6a) 1 1
of the parabola then l(SP )  l(SP ) is equal to
18. The locus of the middle points of chords of the parabola 1 2

y 2  4x , which are of constant length ‘2l’ is 4 2 1 1


A) B) C) D)
A)  4x  y
2
 y 2
 4   4l 2 a a a 4a
26. If the tangents & normals at the extremities of a focal
B)  4y  x  x  4   4l chord of a parabola intersect at (x 1 , y 1 ) and
2 2 2

(x2, y2) respectively, then :


C)  4y  x 2  x 2  4   4l 2 A) x1 = x2 B) x1 = y2
C) y1 = y2 D) x2 = y1
D)  4x  y 2  y 2  4   4l 2 27. A tangent to the parabola y2 = 4ax is inclined at  /
3 with the axis of the parabola. The point of contact
19. If a and c are the lengths of segments of any focal
is
chord of the parabola y 2  2bx  b  0  , then the roots
A) (a/3, -2a / 3 ) B) (3a, -2 / 3 a)
2
of the equation ax  bx  c  0 are
C) (3a, 2/ 3 a) D) (a/3, 2a/ 3 )

3.88
Parabola

28. The straight line joining any point P on the parabola 35. Statement – 1 : If end points of two normal chords
y2 = 4ax to the vertex and perpendicular from the AB and CD (normal at A and C) of a parabola
focus to the tangent at P, intersect at R, then the
y 2  4ax are concyclic, then the tangents at A and C
equaiton of the locus of R is
will intersect on the axis of the parabola.
A) x2 + 2y2 – ax = 0 B) 2x2 + y2 – 2ax = 0
Statement – 2 : If four point on the parabola
C) 2x2 + 2y2 – ay = 0 D) 2x2 + y2 – 2ay = 0
y 2  4ax are concyclic, then sum of their ordinates is
Assertion Reason zero.
Following questions has to be answered according to 36. Statement -1: Normal chord drawn at the pont (8,
the codes mentioned below 8) of the parabola y2 = 8x subtends a right angle at
A) Statement –1 is True, Statement - 2 is True ; the vertex of the parabola.
Statement -2 is a correct explanation for statement–1 Statement-2: Every chord of the parabola y2 = 4ax
B) Statement –1 is True, Statement–2 is True ; passing through the point (4a, 0) subtends a right angle
Statement –2 is NOT a correct explanation for at the vertex of the paraobola.
Statement –1 37. Statement-1 : Slope of tangents drawn from
C) Statement –1 is True, Statement –2 is False
D) Statement –1 is False, Statement –2 is True 1 9
(4, 10) to parabola y2 = 9x are , .
29. Statement-1: The normals at the point (4, 4) and 4 4
Statement-2 : Every parabola is symmetric about
1 
 ,  1  of the parabola y2 = 4x are perpendicular.. its directrix.
4  38. Statement – 1 : The point of intersection of the
Statement-2: The tangents to the parabola at the tangents at three distinct points A, B, C on the parabola
end of a focal chord are perpendicular.
30. Statement – 1 : Circumcircle of a triangle formed by y 2  4x can be collinear..
the lines x = 0, x  y  1  0 and x  y  1  0 also Statement – 2 : If a line L does not intersect the
passes through the point (1, 0) parabola y 2  4x , then from every point of the line
Statement – 2 : Circumcircle of a triangle formed by two tangents can be drawn to the parabola.
three tangents of a parabola passes through its focus. 39. Statement-1: The point (sin ,cos ) does not lie
31. Statement-1 : A is a point on the parabola y2 =
outside the parabola y 2 + x – 2 = 0 when
4ax. The normal at A cuts the parabola again at point
B. If AB subtends a right angle at the vertex of the   5   3 
   ,    , 
1 2 6   2 
parabola, then slope of AB is
2 Statement-2: The point (x1 , y1) lies outside the
Statement-2 : If normal at (at12 , 2at 2 ) cuts again the parabola y2 = 4ax if y12 – 4ax1 > 0.

2 Comprehension linked Passages


parabola at (at 22 , 2at 22 ) then t 2   t 1  .
t1 Passage 1
40. The locus of point of concurrency of the chord of
32. Statement – 1 : Line x  y  5  0 cannot be normal
to parabola contact AB of the circle x 2  y 2  4 is :

 5x  15 
2 2
  5y  10    3x  4y  2  .
2
A) y 2  2x  0 B) y 2  x 2  4

Statement – 2 : Normal to parabola never passes C) y 2  2x  0 D) y 2  2x 2  4


through its focus.
41. The locus of circumcentre of AQB if P   8, 8  is
33. Statement-1: The equation of the common tangent
to the parabolas y 2 = 4x and x 2 = 4y is A) x  2y  4  0 B) x  2y  4  0
x+y+1=0 C) x  2y  4  0 D) x  2y  4  0
Statement-2: Both the parabolas are reflected to
Passage 2
each other about the line y = x.
The locus of a moving point P such that its distance
34. Statement -1: The perpendicular bisector of the line
from a fixed points S called focus bears a constant
segment joining the point
ratio ‘e’ to its distance from a fixed line MZ called
(–a, 2at) and (a, 0) is tangent to the parabola y2 =
directrix is called conic section. The ratio ‘e’ is the
4ax, where t  R .
eccentricity.
Statement-2: Number of parabolas with a given i.e., PS = e. PM
point as vertex and length of latus rectum equal to 4, Equation of a parabola in standard form y2 = 4ax.
is 2.

3.89
Coordinate Geometry

48. The curve C is symmetric about the line


Y P(x,y)
M 3 3
L A) x  B) y  
K 2 2
x = -a
z A S(a, 0) Axis y = 0
X 3 3
(0, 0) C) x   D) y 
L’ (a, -2a) 2 2
y2 = 4ax Passage 5
From a point p(h, k) in general three normals can be
42. The length of the latus-rectum of the parabola 169{(x
– 1)2 + (y – 3)2} = (5x – 12y + 17)2 is drawn to the parabola y 2  4ax . If t1, t 2 , t 3 are
the parameters associated with the feet of normals,
12 14 28 31
A) B) C) D) then t1, t 2 , t 3 are the roots of the equation
13 13 13 13
43. The co-ordinates of a point on the parabola y2 = 8x at 3  (2a  h)t  k  0 . Moreover from the line x  a
whose focal distance is 4 is two perpendicular tangents can be drawn to the
A) (2, 4) B) (4, 2) C) (2, –4) D) (4, –2) parabola.
44. Co-ordinate of the focus of the parabola x2 – 4x – 8y
– 4 = 0 are 49. If the feet Q(at1 2 , 2at 1 ) and R(at 2 2 , 2at 2 ) are the ends
A) (0, 2) B) (2, 1) of a focal chord of the parabola, then the locus of
71  p(h, k) is

C)  3,  D) none of these A) y 2  a(x  2a) B) y 2  a(x  a)
 10 
Passage 3 C) y 2  a(x  3a) D) y 2  3a(x  a)
Consider
50. If the tangents at the feet Q(at1 2 , 2at 1 ) and
C : y  x 2  3,D : y  kx 2 , L 1 : x  a,L 2 : x  1 a  0
45. If the parabola C and D intersect at a point A on the R(at 2 2 , 2at 2 ) to the parabola meet on the line x  a
line L1, then equation of the tangent line L at A to the
then t1 , t 2 are the roots of the equation
parabola D is
A) t 2  t 3 t  1  0 B) t 2  t 3 t  1  0
A) 2  a 3  3  x  ay  a 3  3a  0
C) t 2  t 3 t  1  0 D) t 2  t 3 t  1  0
B) 2  a  3  x  ay  a  3a  0
3 3
51. If p(h, k) is a vertex of the square comprising normals
to the parabola from p and tangents from the directrix
C)  a  3  x  2ay  2a  6a  0
3 3
then (h, k) is the same as
D) None of these A) (a , 0) B) (2a, 0)
46. If a > 0, the angle subtended by the chord AB at the C) (3a, 0) D) (4a, 0)
vertex of the parabola C is Passage 6
Consider one side AB of a square ABCD, (read in
1  5  1  1 
A) tan   B) tan   order) on the line y  2x  17 , and the other two
7  2
vertices C, D on the parabola y  x 2
1  1  52. Minimum intercept of the line CD on y – axis, is :
C) tan1  2 D) tan  
8 A) 3 B) 4 C) 2 D) 6
Passage 4 53. Maximum possible area of the square ABCD can be
If the locus of the circumcentre of a variable triangle A) 980 B) 1160 C) 1280 D) 1520
Passage 7
having sides y – axis, y = 2 and lx  my  1 , where
Two tangents on a parabola are x  y  0 and
(l, m) lies on the parabola y 2  4x is a curve C, then
x  y  0 . If (2, 3) is focus of the parabola, then
47. Coordinates of the vertex of this curve C is 54. The equation of tangent at vertex is
 3  3 A) 4x  6y  5  0 B) 4x  6y  3  0
A)  2,  B)  2,  
 2  2 C) 4x  6y  1  0 D) 4x  6y  3 / 2  0
55. Length of latus rectum of the parabola is
 3  3
C)  2,  D)  2,  
 2  2

3.90
Parabola

neither touches nor intersects the abscissa axis and is


6 10 2 symmetric about the line x = 1. The coefficient of the
A) B) C) D) none
3 13 13 leading term of the polynomial is unity. A point A(x1,
Passage 8 y1) with abscissa x1 = 1 and a point B(x2, y2)
with ordinate y 2 = 11 are given in a cartisian
Consider the circle x 2  y 2  9 and the parabola
rectangular system of co-ordinates OXY in the first
y 2  8x . They intersect at P and Q in the first and quadrant on the curve y = f (x) where 'O' is the origin.
the fourth quadrants, respectively. Tangents to the circle Now answer the following questions:
at P and Q intersect the x – axis at R and tangents to 59. Vertex of the quadratic polynomial is
the parabola at P and Q intersect the x – axis at S. A) (1, 1) B) (2, 3) C) (1, 2) D) none
56. The ratio of the areas of the triangles PQS and PQR 
60. The scalar product of the vectors OA and OB is

is
A) –18 B) 26 C) 22 D) –22
A) 1: 2 B) 1:2 C) 1:4 D) 1:8 61. The area bounded by the curve y = f(x) and a line y
57. The radius of the circumcircle of the triangle PRS is = 3 is
A) 4/3 B) 5/3 C) 7/3 D) 28/3
A) 5 B) 3 3 C) 3 2 D) 2 3 62. The graph of y = f(x) represents a parabola whose
58. The radius of the incircle of the triangle PQR is focus has the co-ordinates
A) 4 B) 3 C) 8/3 D) 2 A) (1, 7/4) B) (1, 5/4)
Passage 9 C) (1, 5/2) D) (1, 9/4)
A quadratic polynomial y = f (x) with absolute term 3

3.91
Coordinate Geometry

Practice Session-2
for
Single Choice
JEE ADVANCED
Y
2
1. Parabolas y    4a  x    and E F
2 A B
y    4a '  x   '  will have a common normal
(other than the normal passing thorugh vertex of
parabola if :
X
2  a  a ' 2  a  a ' D C G
A) 1 B) 1
 '    '
5 1 3 1
2  a ' a  2  a ' a  A) B)
1 1 2 2
C) D)
  '   '
5 1 3 1
2. The number of points with integral coordinates that C) D)
lie in the interior of the region common to the circle 4 4
6. The graph of the curve
x 2  y 2  16 and the parabola y 2  4x , is
2 2
A) 8 B) 10 x  y  2xy  8x  8y  32  0 falls wholly in the
C) 16 D) None of these A) first quadrant B) second quadrant
C) third quadrant D) none of these
3. Consider a parabola y 2  4ax , the length of focal
chord is l and the length of the perpendicular from 7. The line x – y = 1 intersects the parabola y 2  4x at
vertex to the chord is p then A and B. Normals at A and B intersect at C. If D is
the point at which line CD is normal to the parabola,
A) l.p is constant B) l.p 2 is constant
then coordinates of D are
C) l 2 .p is constant D) none of these A) (4, –4) B) (4, 4)
4. Consider a circle with its centre lying on the focus of C) (–4, –4) D) none of these
the parabola y2 = 2px such that it touches the directrix 8. From an external point P, pair of tangent lines are
of the parabola. Then a point of intersection of the drawn to the parabola, y2 = 4x. If 1 & 2 are the
circle and parabola is inclinations of these tangents with the axis of x such

p  p   p p  that, 1 + 2 = , then the locus of P is :
A)  , p  or  , p B)  ,  4
2   2  2 2 
A) x  y + 1 = 0 B) x + y  1 = 0
 p   p p C) x  y  1 = 0 D) x + y + 1 = 0
C)   , p  D)   ,   9. The locus of a point P(h, k) such that the slopes of
 2   2 2
three normals drawn to the parabola
5. ABCD and EFGC are squares and the curve y  k x y2 = 4ax from P be connected by the relation
passes through the origin D and the points B and F. tan 1 m12  tan 1 m 22  tan 1 m 23  . is
FG
The ratio is : A) x 2 tan   y 2  2a(1  2 tan )x  a 2 (3 tan   4)  0
BC
B) x 2 tan   y 2  2a(1  2 tan )x  a 2 (3 tan   4)  0

C) x 2 tan   y 2  2a(1  2 tan )x  a 2 (3 tan   4)  0

3.92
Parabola

D) x 2 tan   y 2  2a(1  2 tan )x  a 2 (3 tan   4)  0 A) 4x + y = 39 B) 2x + y = 19


C) x + y = 9 D) x + 2y = 8
10. The number of common chords of the parabolas
19. In a square matrix A of order 3, a ii  mi  i where
x  y 2  6y  11 and y  x 2  6y  1 are
A) 1 B) 2 C) 4 D) 6 i = 1,2,3 and m i ’s are the slopes (in increasing order
2
11. A t tangent is drawn to the parabola y  4x at the of their absolute value) of the 3 normals concurrent
point ‘P’ whose abscissa lies in the interval [1,4]. The at the point (9, –6) to the parabola y 2  4x . Rest all
maximum possible area of the triangle formed by the other entries of the matrix are one. The value of det.(A)
tangent at ‘P’, ordinate of the point ‘P’ and the is equal to :
x – axis is equal to : A) 37 B) – 6 C) – 4 D) – 9
A) 8 B) 16 C) 24 D) 32
20. If A1B1 and A 2B 2 are two focal chords of the
12. If a  0 and the line 2bx  3cy  4d  0 passes
through the points of intersection of the parabola parabola y 2  4ax , then the chords A1 A 2 and B1B 2
intersect on
y 2  4ax and x 2  4ay , then
A) directrix B) axis
2 2
A) d 2   2d  3c   0 B) d 2   3d  2c   0 C) tangent at vertex D) none of these
21. The tangent at P to a parabola y2 = 4ax meets the
2
C) d 2   2d  3c   0 D) None of these directrix at U and the latus rectum at V then SUV
(where S is the focus) :
13. The locus of the centre of a circle which cuts
A) must be a right triangle
orthogonally the parabola y 2  4x at (1, 2) will pass B) must be an equilateral triangle
through points C) must be an isosceles triangle
A) (3, 4) B) (4, 3) C) (5, 3) D) (2, 4) D) must be a right isosceles triangle.
14. The triangle PQR of area ‘A’ is inscribed in the parabola 22. The distance between a tangent to the parabola
y 2  4ax such that the vertex P lies at the vertex of y 2  4Ax  A  0  and the parallel normal with
the parabola and the base QR is a focal chord. The gradient 1 is :
modulus of the difference of the ordinates of the points
Q and R is : A) 4A B) 2 2A C) 2A D) 2A
2
A A 2A 4A 23. The mirror image of the parabola y  4x in the
A) B) C) D) tangent to the parabola at the point (1, 2) is
2a a a a
2 2
15. TP & TQ are tangents to the parabola, y2 = 4ax at P A)  x  1  4  y  1 B)  x  1  4  y  1
& Q. If the chord PQ passes through the fixed point 2 2
C)  x  1  4  y  1 D)  x  1  4  y  1
( a, b) then the locus of T is :
A) ay = 2b (x  b) B) bx = 2a (y  a) 24. The end points of two normal chords of a parabola
C) by = 2a (x  a) D) ax = 2b (y  b) are concyclic, then the tangents at the feet of the
normals will intersect at
16. Let S be the focus of y 2  4x and a point P is moving A) tangent at vertex of the parabola
on the curve such that is abscissa is increasing at the B) axis of the parabola
rate of 4 units/sec, then the rate of increase of C) directrix of the parabola
projection of SP on x + y = 1 when P is at (4, 4) is D) None of these
25. Let S be the focus of y2 = 4x and a point P is moving
3
A) 2 B) – 1 C)  2 D)  on the curve such that it's abscissa is increasing at the
2 rate of 4 units/sec, then the rate of increase of
17. The straight line joining any point P on the parabola projection of SP on x + y = 1 when P is at (4, 4) is
y 2  4ax to the vertex and perpendicular from the 3
focus to the tangent at P, intersect at R, then the A) 2 B) – 1 C) – 2 D) –
2
equation of the locus of R is
26. In a square matrix A of order 3, aii = mi + i where i
A) x 2  2y 2  ax  0 B) 2x 2  y 2  2ax  0 = 1, 2, 3 and mi's are the slopes (in increasing order
of their absolute value) of the 3 normals concurrent
C) 2x 2  2y 2  ay  0 D) 2x 2  y 2  2ay  0
at the point (9, – 6) to the parabola y2 = 4x. Rest all
18. An equation for the line that passes through other entries of the matrix are one. The value of det.
x2 (A) is equal to
(10, –1) and is perpendicular to y =  2 is A) 37 B) – 6 C) – 4 D) – 9
4

3.93
Coordinate Geometry

27. The equation of a straight line passing through the through a fixed point  x 2 , y 2  then
point (3, 6) and cutting the curve y = x orthogonally
A) x1, a, x 2 are in G.P.
is
A) 4x + y – 18 =0 B) x + y – 9 = 0 y1
C) 4x – y – 6 = 0 D) none B) , a, y 2 are in G.P.
2
28. C is the centre of the circle with centre (0, 1) and
radius unity. P is the parabola y = ax2. The set of y1 x1
values of 'a' for which they meet at a point other than C) 4, , are in G.P.
y2 x 2
the origin, is
D) x 1x 2  y 1y 2  a 2
 1
A) a > 0 B) a   0,  35. The focus and directrix of the parabola
 2 9x2 – 24xy + 16y2 – 20x – 15y – 60 = 0 is

 1 1 1   43 129   43 129 
C)  ,  D)  ,   A)   ,  B)  , 
 4 2 2   25 100   25 100 

Multiple Choice 53 53
C) 4x  3y  0 D) 4x  3y  0
4 4
29. If two distinct chords of a parabola y 2  4ax passing
thorugh the point (a, 2a) are bisected by line 36. If P1P2 and Q1Q 2 , two focal chords of a parabola
x + y = 1, then the length of the latus rectum can not are at right angles, then :
be :
A) area of the quadrilateral P1Q1P2Q 2 is minimum
A) 2 B) 4 C) 5 D) 7
30. The focal chord of y 2 = 16x is tangent to 
(x – 6)2 + y2 = 2, then the possible values of the slope when the chords are inclined at an angle to the
4
of this chord, are
axis of the parabola
1 1 B) minimum area is twice the area of the square on
A) 1 B)  C) D) –1 the latus rectum of the parabola
2 2
C) minimum P1Q1P2Q 2 cannot be found
31. Let PQ be a chord of the parabola y 2  4x . A circle
D) minimum area is thrice the area of the square on
drawn with PQ as a diameter passes through the vertex
the latus rectum of the parabola
V of the parabola. If area of triangle PVQ = 20 sq
37. The locus of the midpoint of the focal distance of a
units, then the coordinates of P are
A) (16, 8) B) (16, –8) variable point moving on the parabola, y 2  4ax is a
C) (–16, 8) D) (–16, –8) parabola whose
32. y  mx bisects two distinct chords drawn from A) latus rectum is half the latus rectum of the original
parabola
(4, 4) on y 2  4x if
A) m = –1/2 B) m = 0 a 
B) vertex is  , 0 
C) m = 1/2 D) m = 1 2 
33. Let there be two parabolas with the same axis, C) directrix is y – axis
focus of each being exterior to the other and the latus D) focus has the co–ordinates (a, 0)
recta being 4a and 4b. The locus of the middle points 38. If the normal at P(18, 12) to the parabola y2 = 8x
of the intercepts between the parabolas made on the cuts it again at Q.
lines parallel to the common axis is a :
A) straight line if a = b A) 9PQ = 80 (10) B) 9PQ = 70 (10)
B) parabola if a  b
C) parabola a, b  R 10 10
C) 80 D) 80
D) none of these 81 91

34. Tangent is drawn at any point  x1, y1  other than 39. P is a point on the parabola y 2  4x and Q is a point

vertex on the parabola y 2  4ax . If tangents are drawn on the line 2x  y  4  0 . If the line x  y  1  0 is
from any point on this tangent to the circle the perpendicular bisector of PQ, then the co–ordinates
of P can be :
x 2  y 2  a 2 such that all the chords of contact pass A) (1, –2) B) (4, 4) C) (9, –6) D) (16, 8)

3.94
Parabola

40. A quadrilateral is inscribed in parabola, then


A) quadrilateral may be cyclic 
a,  
3 3 a  3 3 a a
, , 
 
B) diagonal of the quadrilateral may be equal C)  2 a   2 2
C) all possible pairs of adjacent sides may be    
perpendicular
D) none of these 
 a , 
32 3 a  32 3 a
, ,


a  
41. Whatever be the value of , the line D)  2 a   2 2
y  (x  11)cos   cos 3 is a normal to the parabola    
of Passage 2
A) y2 = 16x B) x2 = 16y The equation of the curve representd by
C) y2 = 4x D) x2 = –4y C  9x 2  24xy  16y 2  20x  15y  60  0 , then
42. Variable circle is described to pass thorugh point (1,0) 46. The locus of the curve C given in above statement is
and tangent to the curve y  tan  tan x  . The locus
1 A) a circle B) a pair of straight lines
C) a parabola D) an ellipse
of the centre of the circle is a parabola whose: 47. The equation of the axis of the curve C is
A) length of the latus rectum is 2 2 A) x  4y B) 3x  y
B) axis of symmetry has the equation x + y=1 C) 3x  4y D) 3x  4y
C) vertex has the co–ordinates (3/4, 1/4) 48. The equation of directrix of the curve C is
D) None of these
A) 16x  9y  53 B) 16x  12y  53  0
43. P is a point on the parabola y2 = 4ax (a > 0) whose
vertex is A. PA is produced to meet the directrix in D C) 16x  12y  53 D) 16x  9y  53  0
and M is the foot of the perpendicular from P on the Passage 3
directrix. If a circle is described on MD as a diameter
y = f(x) is a parabola of the form f  x   x 2  bx  1 ,
then it intersects the xaxis at a point whose
coordinates are : b is a constant. The tangent line is drawn at the point
A) ( 3a, 0) B) ( a, 0) where f(x) cuts y – axis, also touches
C) ( 2a, 0) D) (a, 0) x 2  y 2  r 2  r  0  . It is also given that at least one

Comprehension Linked Passages tangent can be drawn from point P to y = f(x) where
Passage 1 P is a point at which y  x   is non differentiable
2 2
Let the two parabolas y  4ax and y  4ay, a  0   R .
intersect at O and A(O being origin). Parabola P whose 49. For maximum value of b, the area of circle is :
directrix is the common tangent to the two parabolas
 
and whose focus is the point which divides OA A) B) C)  D) 5
10 5
 
internally in the ratio 1  3 : 7  3  rmax .  r
44. The equation of the parabola P is : 50. lim 
b0 sin b
2
  
A)  x  y   2  3 a x  y  1  3 a    1 1 1
A) B)  C) D) Not exist
2 2 2
2
  
B)  x  y   2  3 a 2x  2y  2  3 a    51. Locus of vertex of parabola is :
A) y  1  x 2, x   1, 1, y   1, 0
2
  
C)  x  y   2  3 a 2x  2y  1  3 a   
B) y  1  x 2,x   2, 2 , y  0, 1
2
  
D)  x  y   2  3 a x  y  1  3 a   
C) y  1  x 2,x   2, 2 , y   3, 1
45. Etremities of latus rectum of P are :
D) y  1  x 2, x   1, 1, y   0, 1

a,  
32 3 a  32 3 a a
, , 
 
A)  2   Passage 4
a 2 2
    If l, m are variable real numbers such that
  
3 3 a  3 3 a    5l 2  6m 2  4 lm  3l  0 , then variable line
 a , , , 
a
B)  2 a   2 2 lx  my  1 always touches a fixed parabola, whose
    axes is parallel to x – axis.

3.95
Coordinate Geometry

52. Vertex of the parabola is Passage 7

 5 4  7 3 Consider the inequality, 9 x  a, 3 x  a  3  0 , wheree


A)   ,  B)   ,  ‘a’ is a real parameter.
 3 3  4 4
59. The given inequality has at least one negative solution
5 7 1 3 for a 
C)  ,   D)  ,  
6 6 2 4 A)  , 2  B)  3,  
53. Focus of the parabola is
C)  2,   D) (2, 3)
1 7 1 4 60. The given inequality has at least one positive solution
A)  ,   B)  , 
6 6 3 3 for a 

3 3  3 3
A)  ,  2  B)  3,  
C)  ,   D)   , 
 2 2  4 4 C)  2,   D)  2,  
54. Directrix of the parabola is 61. The given inequality has at least one real solution for
A) 6x  7  0 B) 4x  11  0 a
C) 3x  11  0 D) none of these
A)  , 3  B)  2,  
Passage 5
C)  3,   D)  2,  
4x 2  2y 2  6xy
The limiting value of expression is Passage 8
6x 2  2y  8xy
A parabola whose focus is S(3, 4) is touching the
A as point (x, y) on curve x 2  y 2  1 approaches the coordinates axes.
62. The equation of the circle whose diameter is the portion
 1 1 
position  ,  where A is such that of tangent at vertex of the parabola between the
 2 2
coordinate axis is
(5A, 0) is a point as focus of parabola S having axis A) x2 + y2 – 3x – 4y = 0
parallel to x – axis, vertex at origin. B) x2 + y2 + 6x + 8y = 0
55. The two common tangents can be drawn to both circle C) x2 + y2 – 6x – 8y = 0
and parabola from external point whose co–ordinates D) x2 + y2 + 3x + 4y = 0
are 63. The equation of axis of the parabola is
A) 4x – 3y + 7 = 0 B) 3x – 4y = 0
 4   4 
A)  ,0 B)  ,0  C) 4x – 3y = 0 D) 3x – 4y + 7 = 0
 15  1   17  1  64. If P, Q are ends of focal chord of the parabola then
 4   4  1 1
C)  ,0 D)  ,0   =
 17  1   15  1  SP SQ
56. Locus of midpoints of chords of parabola, which 12 5 6 5
subtend a right angle at vertex of parabola is : A) B) C) D)
5 12 5 6
A) y 2  4x  32  0 B) y 2  4x  32  0
C) y 2  32x  4  0 D) y 2  32x  4  0 Subjective and Integer Type
Passage 6 65. If  x 1 , y 1  ,  x 2 , y 2  and  x 3 , y 3  be three points one
2
y 2  4x and y  8  x  a  intersect at points A and parabola y 2  4ax and the normals at these points
C. Points O(0, 0), A, B(a, 0), C are concyclic. x1  x 2 x 2  x 3 x 3  x1
57. The area of cyclic quadrilateral OABC is meet in a point, then   is
y3 y1 y2
A) 24 3 B) 48 2 C) 12 6 D) 18 5 equal to
58. Tangents to parabola y 2  4x at A and C intersect at 66. Prove that line joining the orthocentre to the centroid
of a triangle formed by the focal chord of a parabola
point D and tangents to parabola y 2  8  x  a  and tangents drawn at its extremities is parallel to the
intersect at point E, then the area of quadrilateral axis of the parabola
DAEC is 67. If the locus of centres of a family of circles passing
through the vertex of the parabola
A) 96 2 B) 48 3 C) 54 5 D) 36 6 y2 = 4ax and cutting the parabola orthogonally at the

3.96
Parabola

other point of intersection is 76. If the parabolas y2 = 4c ( x – d ) and y2 = 4ax have


a common normal other then x-axis (a > 0, c > 0),
2y 2 (2y 2  x 2  12ax)  ax(kx  4a)2 , then find the
value of k. 2c  d
then find the least integral value of
68. Tangents to the parabola at the extremities of a 2a
common chord AB of the circle x 2  y 2  5 and the 77. If the normals at the pionts where the straight line

parabola y 2  4x intersect at the point T. A square


e lx  my  1 meet the parabola y 2  4ax , meet on

ABCD is constructed on this chord lying inside the  am 2 kam 


2 2 2 the normal at the point  k l 2 , l  of parabola
parabola then  TC    TD   is equal to  

69. A parabola is drawn touching the axis of x at the y 2  4ax , then k is equal to
origin and having its vertex at a given distance k from 78. Normal at a point P (a, - 2a) intersects the parabola
this axis. Prove that the axis of the parabola is a y2 = 2x at point Q. If the tangents at P and Q meet at
point R, then prove that the area of triangle PQR is
tangent to the parabola x 2  8k  y  2k  .
70. Prove that the normals at the extremities of each of a 4a 2 (1  m 2 )3
.
series of parallel chords of a parabola intersect on a m3
fixed line itself normal to the parabola. 79. Three normal to y2 = 4x pass through the point
71. Find the locus of the point of intersection of two (15, 12). Show that one of the normal is given by
mutually perpendicular normals to the parabola y = x - 3 and find the equations of the others.
y2 = 4ax and show that the abscissa of the point can 80. A series of chords are drawn so that their projections
never be smaller than 3a. What is the ordinate when on the straight line, which is inclined at an angle 
the abscissa is smallest? to the axis, are of constant length c. Prove that the
72. Let PG is the normal at point P to a parabola cuts its locus of their middle point is the curve
1 y 2 2
 4ax   y cos   2a sin    a 2c 2  0
axis in G and is produced to Q so that GQ  PG .
2
The other normals which pass through Q intersect at 81. ‘O’ is the vertex of parabola y 2  4x and L is the
upper end of latus rectum. If LH is drawn

an angle of , then k = perpendicular to OL meeting in H, then length of double
k
ordinate through H is N , then N =
73. If a leaf of a book be folded so that one corner moves
along an opposite side, then prove that the line of 82. The abscissae of any two points on the parabola
crease will always touch parabola. y2 = 4ax are in the ratio  : 1. Prove that the locus
74. If the normals at the points P, Q, R on the parabola y2 of the point of intersection of tangents at these points
= 4ax meet in the point (h, k). is y2 = (  1/4 +  –1/4 ) ax.
If the centroid and orthocentre of the triangle PQR is 83. Find the area of the trapezium whose vertices lie on
(x 1 , y 1 ) and (x 2 , y 2 ), then find the value of
3x1 – 2x2| the parabola y 2  4x and its diagonals pass through
75. Prove that the locus of intersections of tangents to the 25
parabola y2 = 4ax which intercept a fixed length ‘l ’ (1, 0) and having length unit each
4
on the directrix is (y2 – 4ax) (x + a)2 = l 2x2.

Matrix Type Match the Following


84. If y = x + 1 is axis of parabola, y + x = 4 is tangent of same parabola at its vertex and y = 2x+3 is one of its
tangent, then
Column – I Column – II
A) If equation of directrix of parabola is ax + by – 29 = 0, then a + b = 0 P) 9

a 2
B) If length of latus rectum of parabola is where a and b are Q) 18
b
relatively prime natural numbers, then a + b =
C) Let extremities of latus rectum are  a1 , b1  and  a 2 , b 2  , then R) 23

 a1  b1  a 2  b2   (where[.] denote greatest integer function)

3.97
Coordinate Geometry

2
D) If equation of parabola is a  x  y  1  b  x  y  4  S) 37
where a and b are relatively prime natural numbers then a + b =
85. Column – I Column – II
A) Parabola y2 = 4x and the circle having its centre at (6, 5) intersects P) 13
at right angle, at the point (a, a) then one value of a is equal to
B) The angle between the tangents drawn to (y – 2)2 = 4(x + 3) Q) 8
at the points where it is intersected by the line 3x – y + 8 = 0 is
4p/p, then p has the value equal to
C) If the line x – 1 = 0 is the directrix of the parabola R) 10 5
2
y – kx + 8 = 0, then one of the value of k is/are
D) Length of the normal of the parabola y2 = 8x at S) 4
the point where abscissa & ordinate are equal is
T) – 8
86. Column – I Column – II
A) Radius of the largest circle which passes through the focus of the P) 16
parabola y2 = 4x and contained in it, is
B) Two perpendicular tangents PA & PB are drawn to the parabola Q) 5
y2 = 16x then length AB may be
C) The shortest distance between parabolas y2 = 4x and y2 = 2x – 6 R) 8
is d then d2 = 0
D) The harmonic mean of the segments of a focal S) 4
chord of the parabola y2 = 8x
T) 20
2
87. Normals are drawn from point (4, 1) to the parabola y  4x . The tangents at the feet of normals to the parabola

y 2  4x from a triangle ABC.


Column – I Column – II
2
A) The distance of focus of parabola y  4x from centroid of ABC is P) 5/3

B) The distance of focus of parabola y 2  4x from orthocentre of ABC is Q) 10


2

C) The distance of focus of parabola y 2  4x from circumcenter of ABC is R) 7


2

D) Area of ABC is S) 5
2
T) 5

3.98
Brain Booster
Single Choice rectum, then the area of the trapezium PL1L2Q is
1. The axis of a parabola is along the line y = x and the
A) 3 2a 2 B) 2 2a 2
distances of its vertex and focus from origin are 2
2 2 2
and 2 2 respectively. If vertex and focus both lie in C) 4a2 D)  2  a
the first quadrant, then the equation of the parabola  
is 6. Through the vertex O of the parabola y 2  4ax two
A) (x – y)2 = 8(x + y – 2)
chords OP & OQ are drawn and the circles on OP &
B) (x + y)2 = 2 (x + y – 2)
OQ as diameter intersect in R. If 1, 2 &  are the
C) (x – y)2 = 4 (x + y – 2)
angles made with the axis by the tangents at P & Q
D) (x + y)2 = 2(x – y + 2)
2. Which one of the following equations represented on the parabola & by OR, then cot 1  cot  2 is equal
parametrically, represents equation to a parabolic to
profile ? A) –2tan  B) –2tan(– )
A) x = 3 cos t ; y = 4 sin t C) 0 D) 2 cot 
t 7. If the locus of middle point of point of contact of
B) x2  2 =  2 cos t ; y = 4 cos2 tangent drawn to the parabola y2 = 8x and foot of
2
perpendicular drawn from its focus to the tangent is a
C) x = tan t ; y = sec t conic then length of latusrecturm of this conic is
A) 9/4 B) 9 C) 18 D) 9/2
t t 8. If the chord of contact of tangents from a point P to
D) x = 1  sin t ; y = sin + cos
2 2 the parabola y2 = 4ax touches the parabola x2 = 4 by
3. Locus of the feet of the perpendiculars drawn from ,the locus of P is
vertex of the parabola y2 = 4ax upon all such chords A) circle B) parabola
of the parabola which subtend a right angle at the C) ellipse D) hyperbola
vertex is 9. Let P and Q be Points (4, – 4) and (9, 6) of the
A) x2 + y2 – 4ax = 0 B) x2 + y2 – 2ax = 0 parabola y2 = 4a(x – b). Let R be a point on the arc
C) x2 + y2 + 2ax = 0 D) x2 + y2 + 4ax = 0 of the parabola between P & Q. Then the area of
4. A circle is described whose centre is the vertex and PRQ is largest when
whose diameter is three-quarters of the latus rectum A) PRQ  900 B) the point R is (4,4)
of the parabola y2 = 4ax. If PQ is the common chord
of the circle and the parabola and L1 L2 is the latus 1 
rectum, then the area of the trapezium PL1 L2Q is : C) the point R is  ,1  D) None
4 
 2 1  2 10. If a focal chord of y 2 = 4ax makes an angle
A) 3 2 a2 B)  2  a
   
,    0,  with the positive direction of x-axis, then
 4
2  2 minimum length of this focal chord is
C) 4 a2 D)  2  a2
  A) 4a B) 6a C) 8a D) None
5. A circle is described whose centre is the vertex and 11. If the lines (y  b)  m1 (x  a) and (y  b)  m2 (x  a)
whose diameter is three-quarters of the latus rectum are the tangents to y2 = 4ax, then
of the parabola y 2  4ax . If PQ is the common chord 2
(y  b)  m1(x  a)y  b  m 2 (x  a), y  4ax
of the circle and the parabola and L1L2 is the latus
Coordinate Geometry

A) m1 + m2 = 0 B) m1m2 = 1 A) Latus rectum is half the latus rectum of the original


C) m1 + m2 = 1 D) m1m2 = – 1 parabola
12. The parabola y2 = 4ax and circle (x – 6)2 + y2 = r2 B) Vertex is (a/2, 0)
will have no common tangent if ‘r’ is C) Directrix is y-axis
A) r  20 B) r  20 D) Focus has the co-ordinates (a, 0)
19. Let y2 = 4ax be a parabola and x2 + y2 + 2 bx = 0
C) r  18 D) r  ( 20, 28) be a circle. If parabola and circle touch each other
13. If A & B are points on the parabola y2 = 4ax with externally then :
vertex O such that OA perpendicular to OB & having A) a > 0, b > 0 B) a > 0, b < 0
C) a < 0, b > 0 D) a < 0, b < 0
4 /3 4/3
1 r 2r 20. Let V be the vertex and L be the latusrectum of the
lengths r1 & r2 respectively, then the value of 2/3 2/ 3
r
1 r
2 parabola x2 = 2y + 4x – 4. Then the equation of the
is parabola whose vertex is at V, latusrectum is L/2 and
A) 16a2 B) a2 C) 4a D) None axis is perpendicular to the axis of the given parabola.
A) y2 = x – 2 B) y2 = x – 4
Multiple Choice C) y2 = 2 – x D) y2 = 4 – x
21. The locus of the mid point of the focal radii of a
14. If from the vertex of a parabola y 2  4ax a pair of variable point moving on the parabola, y2 = 4ax is a
chords be drawn at right angles to one another and parabola whose
with these chords as adjacent sides a rectangle be A) Latus rectum is half the latus rectum of the original
made, then the locus of the further angle of the parabola
rectangle is : B) Vertex is (a/2, 0)
A) an equal parabola C) Directrix is y–axis
B) a parabola with focus at (8a, 0) D) Focus has the co–ordinates (a, 0)
C) a parabola with directrix as x – 7a = 0
D) not a parabola Assertion Reason
15. Through a point P(–2, 0), tangents PQ and PR are Following questions has to be answered according to
drawn to the parabola y 2  8x . Two circles each the codes mentioned below
A) Statement –1 is True, Statement - 2 is True ;
passing through the focus of the parabola and one
Statement -2 is a correct explanation for statement–1
touching parabola at Q and other at R are drawn.
B) Statement –1 is True, Statement–2 is True ;
Which of the following point(s) with respect to the
Statement –2 is NOT a correct explanation for
triangle PQR lie(s) on the common chord of the two
Statement –1
circles ?
C) Statement –1 is True, Statement –2 is False
A) centroid B) orthocentre
D) Statement –1 is False, Statement –2 is True
C) incentre D) circumcentre
22. Statement – 1 : If straight line x = 8 meets the
16. If (xr, yr); r = 1, 2, 3, 4 be the points of intersection of
parabola y2 = 8ax at P & Q then PQ subtends a right
the parabola y 2 = 4ax and the circle
angle at the origin.
x2 + y2 + 2gx + 2fy + c = 0, then
Statement – 2 : Double ordinate equal to twice of
A) y1 + y2 + y3 + y4 = 0
latus rectum of a parabola subtends a right angle at
B) x1  x 2  x 3  x 4  0 the vertex.
C) y1 – y2 + y3 – y4 = 0 23. Statement – 1 : Area of triangle formed by pair of
D) y1 – y2 – y3 + y4 = 0 tangents drawn from a point (12, 8) to the parabola
17. Consider a circle with its centre lying on the focus of y2 = 4ax and their corresponding chord of contact is
the parabola, y2 = 2 px such that it touches the 32 sq. units
directrix of the parabola. Then a point of intersection Statement – 2 : If from a point P(x1, y1) tangents are
of the circle & the parabola is : drawn to a parabola y2 = 4ax then area of triangle
formed by these tangents and their corresponding chord
p  p 
A)  , p  B)  ,  p  3
2  2 
of contact is
y 2
1  4ax 1  2
sq.units
4a
 p   p 
C)   , p  D)   ,  p  24. Statement – 1 : The perpendicular bisector of the
 2   2 
line segment joining the point (–a, 2at) and (a, 0) is
18. The locus of the mid point of the focal radii of a
tangent to the parabola y2 = 4ax, where t  R
variable point moving on the parabola, y2 = 4ax is a
Statement – 2 : Number of parabolas with a given
parabola whose

3.100
Parabola

point as vertex and length of latus rectum equal to 4 27. The minimum area bounded by y  g  x  & y  f  x 
is 2.
is :
x 2 A) 1/3 B) 1/2 C) 2/3 D) 5/6
25. Statement-1 : The curve y   x  1 is symmetric
2 28. Let y = g(x) intersects y = f(x) at two distinct points
with respect to the line x = 1 A, B, then the slope of g(x) if length of segment AB is
Because 4 units is :
Statement-2 : A parabola is symmetric about its A)  1 B)  2 C)  3 D)  4
axis. Passage 2
y = f(x) is a parabola of the form y = x2 + ax + 1, ,
Comprehension Linked Passages its tangent at the point of intersection of y–axis and
Passage 1 parabola also touches the circle x2 + y2 = r2. It is
The function f satisfies known that no point of the parabola is below x –axis.
f  x   f  2x  y   5xy  f  3x  y   2x 2  1 for all 29. The radius of circle when a attains its maximum value
1 1
real numbers x, y. Let a chord to parabola x 2  4y , A) B) C) 1 D) 5
10 5
normals to parabola at ends of which satisfy the 30. The slope of the tangent when radius of the circle is
relation, m1m2  2 where m1 , m 2 represent slope maximum
of normals, passes through a fixed point ‘P’ on axis A) 0 B) 1
of parabola. Let y = g(x) represent line passing through C) – 1 D) not defined
31. The minimum area bounded by the tangent and the
point P.
26. The value of f(10) is equal to : coordinate axes
A) –61 B) – 49 C) – 21 D) – 10 1 1 1
A) B) C) D) 1
4 3 2
Matrix Type Match the Following
32. Column – I Column – II
A) Parabola y2 = 4x and the circle having its centre at (6, 5) P) 13
intersects at right angle, at the point (a, a) then one value of
a is equal to
B) The angle between the tangents drawn to Q) 8
2
 y  2  4  x  3  at the points where it is intersected by the

4
line 3x  y  8  0 is , then p has the value equal to
p

C) If the line x  1  0 is the directrix of the parabola R) 10 5

y 2  kx  8  0 , then one of the value of k is


D) Length of the normal chord of the parabola y 2  8x at the S) 4
point where abscissa & ordinate are equal is
T) 12
33. Column – I Column – II
A) Radius of the largest circle which passes through the P) 16
focus of the parabola y 2  4x and contained in it, is
B) Two perpendicular tangents PA & PB are drawn to the Q) 5
2
parabola y  16x then minimum value of AB is
C) The shortest distance between parabola y 2  4x and R) 8

y 2  2x  6 is d then d 2 
D) The harmonic mean of the segments of a focal chord S) 4
2
of the parabola y  8x
T) 12

3.101
Coordinate Geometry

34. Column – I Column – II


A) The equation of the common normals to the parabolas P) y = 1
y2 = 4ax and x2 = 4ay is/are
B) A pair of tangents drawn from a point P to the parabola y2= 4ax Q) x =a
intersects the coordinate axes in concyclic points. The locus of P is
C) The locus of point from which tangents drawn to parabolas R) y = x – 3a
y2 = 4a(x + a) and y2 = 8a(x + 2a) are mutually perpendicular is
D) y = 2x + c, ‘c’ being variable is a chord of the parabola S) x + 3a = 0
y2 = 4x, meeting the parabola at A and B. Locus of a point
dividing the segment AB internally in the ratio 1 : 1 is
T) x = 0

3.102
ANSWER KEY TO PS-1
1. D 2. A 3.B 4.D 5.D 6.C 7.D 8.D 9.B 10.B 11.D
12.C 13.A 14.A 15.B 16.A 17.A 18.D 19.C 20.C 21.C 22.D
23.A 24.D 25.C 26.C 27.A 28.B 29.A 30.A 31.B 32.C 33.B
34.C 35.A 36.D 37.C 38.D 39.A 40.C 41.A 42.C 43.A 44.B
45.B 46.B 47.A 48.D 49.C 50.D 51.C 52.A 53.C 54.A 55.B
56.C 57.B 58.D 59.C 60.B 61.A 62.D

ANSWER KEY TO PS-2


1.A 2.A 3.B 4.A 5.A 6.A 7.B 8.C
9.A 10.D 11.A 12.D 13.A 14.C 15.C 16.C
17.B 18.D 19.C 20.A 21.C 22.B 23.C 24.B
25.C 26.C 27.A 28.D 29.B,C,D 30.A,D 31. A,B 32. B,C,D
33.A,B 34.B,C,D 35.A,C 36. A,B 37.A,B,C,D 38.A,C 39. A,C 40.A,B
41.A,B 42.B,C 43.A,D 44.C 45.D 46.B 47.A 48.D
49.B 50.D 51.D 52.A 53.B 54.C 55.C 56.A
57.B 58.A 59.D 60.C 61.B 62.A 63.D 64.B
2
65. 0 67. 3 68. 6400 71. a(x – 3a) = y , 0 72. 2 74.8a 76. 1

77.4 79. y   4x  72 and y = 3x – 33 81. 80 83. 75/4

84. A – Q ; B – R ; C – P ; D – S 85.A – S ; B – Q ; C – S ; D – R
86. A – S ; B – P ; C – Q ; D – S 87. A – P ; B – T ; C – Q ; D – S

BRAIN BOOSTER
1.A 2.B 3.A 4.D 5.D 6.A 7.B 8.D
9.C 10.C 11.D 12.B 13.A 14.A,C 15.A,B,C,D 16.A,B
17.A,B 18.A,B,C,D 19.A,D 20.A,B 21.AB,C,D 22.A 23.C 24.C
25.A 26.B 27.C 28.A 29.B 30.A 31.A
32. A – S ; B – Q ; C – S ; D – R 33. A – S ; B – P ; C – Q ; D – S
34. A – R ; B – Q ; C – S ; D – P

You might also like